Download as pdf or txt
Download as pdf or txt
You are on page 1of 98

1 | Page

KENDRIYA VIDYALAYA SANGATHAN


JAIPUR REGION

Sh. B L MORODIA
Deputy Commissioner
KVS JAIPUR REGION

Sh. D R MEENA Sh. MADHO SINGH Sh. G S MEHTA


Assistant Commissioner Assistant Commissioner Assistant Commissioner
KVS JAIPUR REGION KVS JAIPUR REGION KVS JAIPUR REGION

Sh. P K TAILOR Sh. D K DEHRAN


COORDINATOR COORDINATOR
PRINCIPAL, KV NO.2 JAIPUR PRINCIPAL, KV NO.2 AJMER
2 | Page
KENDRIYA VIDYALAYA SANGTHAN JAIPUR REGION
FIVE DAYS WORKSHOP OF PGT CHEMISTRY W.E.F. 25.08.2023 TO 27.08.2023
DETAIL OF PARTICIPANTS
S. No. NAME OF TEACHER NAME OF KV
1 VIJAY KUMAR SETHI KV NASIRABAD
2 RAKESH KUMAR KV JHUNJHUNU
3 POOJA YADAV KV BHILWADA
4 MANOJ KUMAR SAIN KV LALGARH JATAN
5 RAKESH KUMAR KV NO. 2 JAIPUR
6 YOGESH KUMAR SWAMI KV NO. 2 AJMER
7 ANUJ KUMAR SHARMA KV BHARATPUR
8 SHIV PRAKASH OLA KV JHALAWAD
9 RAKESH KUMAR MEENA KV BARAN
10 HIMMAT SINGH KV NO.1 AFS JODHPUR
11 ASHUTOSH SHARMA KV NO.1 AFS JODHPUR
12 SURENDRA KUMAR KV ANUPGARH
13 DIMPLE ARORA KV NO. 2 BIKANER
14 NEKA RAM KV NO. 2 AFS JODHPUR
15 BHUPENDRA KR. VERMA KV SRIGANGANAGARCANTT
16 S C JAT KV BSF DABLA
17 SHALU SOLANKI KV UTTARLAI
18 DR. ANJANA KV NO. 6 JAIPUR
19 CHANDRAKALA KV NO. 1 UDAIPUR
20 GOPAL SAINI KV CHITTORGARH
21 GURVINDER KAUR KV ANTA
22 RENU JAIN KV NO. 3 JAIPUR
23 SUSHILA KUMARI KV AFS JAISALMER
24 ARJUN SINGH RAJPUROHIT KV NO. 1 JAIPUR
25 RAKHI GOYAL KV STPS SURATGARH
26 DHARAMRAJ MEENA KV NO. 1 KOTA
27 GOPAL SAIWLIYA KV HANUMANGARH
28 BANSHI DHAR KHATIK KV JALIPA CANTT BARMER
29 RAJESH VERMA KV NO. 4 JAIPUR
30 K. K. MEENA KV NO. 5-1st (shift) JAIPUR
31 DEVENDRA KUMAWAT KV PHULERA
32 S. C. SEERVI KV BSF JODHPUR
33 VINOD KUMAR MEENA KV KHETRI NAGAR
34 BABLU SINGH KV NO. 1 ALWAR
35 BHUPENDRA PRAKASH KV BEAWER
36 NAVEEN KR. VYAS KV BANAR JODHPUR
37 VIKAS KUMAR SUMAN KV NO. 1 KOTA
38 JITENDRA SINGH KV JALORE
39 AMIT KUMAR KV NAGAUR
40 SHER SINGH KV NO.1 ALWAR
41 ARUN KUMAR KV KARAULI
42 MOHAN LAL KV SURATGARH CANTT.
43 SANDEEP KUMAR KV NO. 1 AFS SURATGARH
44 SHRAVAN KUMAR KV NO. 1 AJMER
45 SURESH KUMAR KV CTPP CHHABRA
46 BADRI PRASHAD PRAJAPAT KV DHOLPUR
47 HARI NARAYAN KV NO. 2 KOTA
48 MAHIPAL SINGH KV SWRI AVIKANAGAR
49 NARAYAN LAL BAIRWA KV EKLINGGARH UDAIPUR
3 | Page
50 BANWARI LAL SAINI KV DEOLI
51 REKHA SHARMA KV NO. 2 JAIPUR
52 BHARAT BHUSHAN KV GANGAPUR CITY
53 SEEMA KUMARI SHARMA KV CHURU
54 PREETI KAUSHIK KV DUNGARPUR
55 JITENDRA KUMAR SAINI KV ITARANA ALWAR
56 AKSHAY KUMAR KV NO. 1 JAIPUR
57 BHARAT KUMAR PANDYA KV BANSWARA
58 VIDHYDHAR SINGH KV SIKAR
59 SUNITA GUPTA KV TONK
60 DHEERAJ KUMAR KV UTTARLAI
61 PREM BAI KV NO. 3 JAIPUR
62 J. K. GUPTA KV NO. 5 SHIFT-1 JAIPUR
63 SUMAN TALWARA KV NO. 2 AFS JODHPUR
64 MOOLCHAND MAHALA KV INDERPURA
65 RAJESH RAM SINGH KV NO. 1 ARMY JODHPUR
66 BALESHWAR PRASAD KV NO. 1 UDAIPUR
67 ASHOK KUMAR RAJAWAT KV SAWAI MADHOPUR
68 VIJAY SINGH SAINI KV BHARATPUR
69 RITU KUMARI KV DEOGARH
70 SUREKHA VERMA KV NO. 7 JAIPUR
71 NEHA SUMAN KV NO. 1 AJMER
72 AJAY KUMAR KV KAHJUWALA

SYLLABUS FOR SESSION 2023-24


CLASS-XII
Time: 3 Hours 70 Marks

4 | Page
SOLUTIONS
QUICK REVISION POINTS:
It is a homogenous mixture of solute and solvent. Solute + Solvent = Solution
EXPRESSING CONCENTRATION OF SOLUTIONS
Some basic formulae:-
Mass percentage. Mole fraction Molarity (M) =
w/w% 𝑛𝐴 No. of moles of solute/volume
𝑋𝐴 =
𝑀𝑎𝑠𝑠 𝑜𝑓 𝑠𝑜𝑙𝑢𝑡𝑒 𝑊𝐵 𝑛𝐴 + 𝑛𝐵 of solution (L)
= × 100 𝑛𝐴
𝑀𝑎𝑠𝑠 𝑜𝑓 𝑠𝑜𝑙𝑢𝑡𝑖𝑜𝑛 𝑊𝐴 + 𝑊𝐵 𝑋𝐵 = Unit=mole/litre [mol/L]
𝑛𝐴 + 𝑛𝐵
XA+XB=1
For liquid solutions Molality (m) =
ppm (parts per million)
%v/v = no. of moles of solute/mass of
𝑉𝑜𝑙𝑢𝑚𝑒 𝑜𝑓 𝑠𝑜𝑙𝑢𝑡𝑒(𝑉𝐵) 𝑊𝐵 solvent (kg)
𝑉𝑜𝑙𝑢𝑚𝑒 𝑜𝑓 𝑠𝑜𝑙𝑢𝑡𝑖𝑜𝑛(𝑉𝐴 + 𝑉𝐵 ) =𝑊 ×106 Unit=mole/Kilogram [mol/kg]
𝐴 +𝑊𝐵

NOTE: Mole Fraction, Mass Percentage, ppm and Molality are independent of temperature,
whereas molarity depends on temperature. This is because volume depends on temperature.
SOLUBILITY (maximum amount of solute present in a specific amount of solvent at a specified
temperature) of gas depends on:
Nature of gas Pressure Temperature and Solubility of gas
Easily Increases with Increases with decrease in temperature, aquatic species are
liquefiable, increase in pressure more comfortable in cold water rather thanin warmwater
Easy to (HENRY’S LAW) and prefer to stay at lower level away from the upper layer
dissolve of water in summers.
Henry’s law: - Mole fraction of gas in the solution is directly proportional to partial pressure of gas at
constant temperature in the vapour phase.
p  XB p= KHXB ( KH= Henry’s constant )(greater the KHvalue lower the solubility)
APPLICATIONS: Soft drinks are sealed under high pressure to increase solubility of CO2.
To avoid BENDS, the tanks used by scuba divers are filled with air diluted with helium (less soluble
than N2)
ANOXIA problem at higher altitudes due to low pressure, low concentration of O2 in Blood cells.

Vapour pressure: -The pressure exerted by the vapours in the equilibrium with liquid at a given
temperature.
For two volatile liquids For one non-volatile and one volatile liquid
PA = P0AΧA PB = P0BΧB PA = P0AΧA PB =0
Dalton’s partial pressure Dalton’s partial pressure
PTotal= P0AΧA+ PB0ΧB PTotal= P0A(1-XB)+ PB0ΧB PTotal= P0AΧA+0 PTotal= P0AΧA
Composition of vapour phase in equilibrium with the solution: PA = PtotalYA PB = PtotalYB

Raoult’s law: - The partial vapour pressure of each component in the solution is directly proportional
to its mole fraction PA  XA PB  XB

Ideal solutions Non-Ideal solutions


Obeys Raoult’s law over the entire rangeof Don’t Obey Raoult’s law over the entire range
concentration. of concentration.
e.g. Solution of n-hexane and n-heptane
A-A, B-B = A-B interactions A-A, B-B ≠ A-B interactions
PTotal= PA+ PB PTotal= P0AΧA+ PB0ΧB PTotal≠ PA+ PB PTotal≠ P0AΧA+ PB0ΧB
ΔmixH=0 ΔmixV=0 ΔmixH≠0, ΔmixV≠ 0
Can be separated by fractional distillation Cannot be separated by fractional distillation

5 | Page
Non-Ideal solutions
Positive deviation Negative deviation
PTotal > PA+ PB, ΔmixV=+ve, ΔmixH=+ve PTotal< PA+ PB, ΔmixV=-ve , ΔmixH=-ve
A – B interaction is weaker than A – A or B – B A – B interaction is stronger than
interactions. A – A and B – B interaction.
Minimum boiling Azeotrope* Maximum boiling Azeotrope
e.g. Water+ Alcohol, Ethanol + Acetone, e.g. Water+ HNO3, Chloroform + Acetone,
Carbon disulphide + Acetone Phenol + Aniline
Azeotrope* (Greek: boiling without change) Mixtures having same composition in liquid and
vapour phase, boil at a constant temperature and cannot be separated by fractional distillation.

Colligative properties: - The properties of solutions which depend only on the number of particles of
the solute (molecules or ions) dissolved in a definite amountof the solvent and do not dependonthe
nature of solute.
Relativelowering in Elevation in boiling Depression in Osmosis
vapourpressure point freezing point
po-p/po=XB ΔTb =Tb -𝑇𝑏𝑜 ΔTf =𝑇𝑓0 −Tf π = CRT
𝑷𝟎𝑨 − 𝑷𝑺 ΔTb=Kb×m ΔTf =Kf ×m 𝒏
𝜟𝑷𝑨
= = 𝑿𝑩 π = 𝒗RT
𝑷𝟎𝑨 𝑷𝟎𝑨
[for a very dilute 𝑲𝒃 × 𝒘𝟐 × 𝟏𝟎𝟎𝟎 𝑲𝒇 × 𝒘𝟐 × 𝟏𝟎𝟎𝟎 π=osmotic
𝚫𝐓𝒃 = 𝚫𝐓𝒇 = pressure
solution nB << nA ] 𝑴𝟐 × 𝒘𝟏 𝑴𝟐 × 𝒘 𝟏
Kb = molal elevation or Kf = Molal depression For Isotonic
EBULLIOSCOPIC or CRYOSCOPIC solutions
𝑷𝟎𝑨 −𝑷𝑺 𝒏𝑩 𝒘𝟐/𝑴𝟐
= = constant. constant. π1 = π2
𝑷𝟎𝑨 𝒏𝑨 𝒘𝟏/𝑴𝟏
It is the elevation in It is the depression in
boiling point of 1 molal freezing point of 1 and under
solution. molal solution. identical conditions
𝑴 × 𝑹 × 𝑻𝟐𝒃 𝑴 × 𝑹 × 𝑻𝟐𝒇
𝐊𝒃 = 𝐊𝒇 =
𝟏𝟎𝟎𝟎 × 𝒗𝒂𝒑 𝑯 𝟏𝟎𝟎𝟎 × 𝒇𝒖𝒔 𝑯 C1 = C2

6 | Page
IMPORTANT POINTS:
 Condition for Reverse osmosis: pressure higher than the osmotic pressure is applied on the
solution.The solvent will flow from the solution into the pure solvent through the semi-
permeable membrane (cellulose acetate).It is used in desalination of sea water.
 Antifreeze agent: ethylene glycol used in car radiators at hill stations (Depression in
freezing point)
 De-icing agents: common salt (NaCl) or calcium chloride (CaCl2) is scattered on the roads to
melt ice in cold countries (Depression in freezing point)
Biological and Industrial importance of osmosis:
 Edema: Due to excess intake of salt by a person, the tissues become puffy, it is called edema. It
is due to retention of water in the tissue owing to osmosis.
 The preservation of meat by salting and of fruits by adding sugar protects against bacterial
action. Through the process of osmosis, a bacterium on salted meat or candid fruit loses water,
shrivels and dies.
 The osmotic pressure method has got advantages over other colligative
properties:
 The measurement of osmotic pressure is around the room temperature
 gives large measurable values for molar masses and
 Molarity of the solution is used instead of molality.
Conditions for normal molar mass (Accurate value)
Solute must be non-volatile, Solution must be dilute and solute should not undergo any association or
dissociation in the solution (non- electrolyte solution).
Abnormal molar mass: Electrolytic Solutions: - When the molar mass of a solute become higher or
lower after its association or dissociation in the solution it is called abnormal molar mass.
Van’t Hoff factor: i = normal molar mass/abnormal (observed) molar mass
No. of Colligative Molar mass ⅈ (Van’t  (extent) If =100% Examples
PROPERTY molecules property Hoff factor) (=1)
Acetic Acid
Greater than ⅈ−𝟏 𝟏
ASSOCIATION in benzene
Less Lowers theoretical ⅈ˂1 𝟏 ⅈ= 𝟏
−𝟏 𝒏
value 𝒏 ⅈ=
𝟐
ⅈ−𝟏
DISSOCIATION more Increases lesser ⅈ˃1 ⅈ=𝒏 AlCl3 (ⅈ = 𝟒)
𝒏−𝟏

Q.NO. MULTIPLE CHOICE QUESTIONS (1 MARK)


1 12 g of urea is dissolved in 1 litre of water and 68.4 g of sucrose is dissolved in 1 litre of water.
The lowering of vapour pressure in the first case is-
(a) equal to second (b) greater than second (c) less than second (d) double that of second
2 Which solution is isotonic to the blood?
(a)0.75 % by weight of NaCl approximately (b) 0.99 % by weight of NaCl approximately
(c) 0.90 % by weight of NaCl approximately (d) 0.54 % by weight of NaCl approximately
3 An unknown gas X is dissolved in water at 2.5 bar pressure & has mole fraction 0.04 in solution.
The mole fraction of X gas, when the pressure of gas is doubled at the same temperature, will
be-
(a) 0.08 (b) 0.04 (c) 0.96 (d) 0.88
4 The mass of (COOH)2.2H2O, oxalic acid needed to prepare 500 mL of 0.1 molar solution is
(a) 12.6 g (b) 6.3 g (c) 4.2 g (d) 5 g
5 X is dissolved in water. Maximum boiling point is observed when X (0.1 M each) is ………
(a) ZnSO4 (b) CaCl2 (c) NaCl (d) urea
6 The values of KH (in kbar), For gases A, B, C & D are 76, 1.67, 0.41 & 56 respectively –
Which one is least soluble in H2O-
(a) A (b) B (c) C (d) D
7 Identify the law which is stated as: “For any solution, the partial vapour pressure of each
volatile component in the solution is directly proportional to its mole fraction.”
7 | Page
(a) Henry’s law (b) Raoult’s law (c) Dalton’s law (d) Gay-Lussac's Law
8 Four liquids P, Q, R and S boil at 73 C, 87 C, 80 C and 94 C respectively. Which one has the
° ° ° °

highest vapour pressure at 50° C.


(a) P (b) Q (c) R (d) S
9 In reverse osmosis:
(a) a pressure greater than osmotic pressure is applied on pure water side
(b) a pressure less than osmotic pressure is applied on pure water side
(c) a pressure greater than osmotic pressure is applied on salt water side
(d) a pressure less than osmotic pressure is applied on salt water side
10 An azeotropic solution of two liquids has boiling point lower than either of them when solute-
solvent interactions are:
(a) Equal to solute- solute and solvent- solvent interactions
(b) Stronger than solute -solute and solvent -solvent interactions
(c) Weaker than solute -solute and solvent -solvent interactions
(d) None of the above
11 Which of the following pairs have same units?
(a) Molality & Molarity (b) Molarity & Mole fraction
(c) Cryoscopic constant & Ebullioscopic constant (d) Henry constant & Cryoscopic constant
12 Colligative properties depend on
(a) the nature of the solute particles dissolved in solution
(b) the number of solute particles in solution
(c) the physical properties of the solute particles dissolved in solution
(d) the nature of solvent particles
13 Calculate the mass % of aspirin (C9H8O4) in acetonitrile (CH3CN) when 6.5 g of C9H8O4 is
dissolved in 450 g of CH3CN.
(a) 3.8 % (b)7.2 % (c) 1.4 % (d)6.5%
14 In some cases of saturated solution,the dissolution process is endothermic (∆sol H > 0) then-
(a) solubilty increases with rise in temperature
(b) solubilty decreases with rise in temperature
(c) solubilty first decreases then increases with rise in temperature
(d) solubilty remains same with rise in temperature
15 Low concentration of oxygen in the blood and tissues of people living at high altitude is due to
(a) low temperature (b) low atmospheric pressure (c) high atmospheric pressure
(d) both low temperature and high atmospheric pressure
ANS. 1 (a) 2(c) 3(a) 4(b) 5(b) 6(a) 7(b) 8(a) 9(c) 10(c) 11(c) 12(b) 13(c) 14(a) 15(b)
ASSERTION REASON TYPE QUESTIONS (1MARK)
Note: In the following questions (1-5) a statement of assertion followed by a statement
of reason is given. Choose the correct answer out of the following choices.
(a) Assertion and reason both are correct statements and reason is the correct explanation for
assertion.
(b) Assertion and reason both are correct statements but the reason is not a correct
explanation for assertion.
(c) Assertion is a correct statement but the reason is the wrong statement.
(d) Assertion is a wrong statement but the reason is a correct statement.
1 Assertion: Azeotropic mixtures are not formed only by non-ideal solutions and they may have
boiling point either greater than both the components or less than both the components.
Reason: The composition of vapour phase is same as that of the liquid phase of Azeotropic
mixture.
2 Assertion: In solution, Amalgam of mercury with sodium is an example of solid solutions.
Reason: Mercury is solvent and sodium is solute in the solution.
3 Assertion: When a solution is separated from the pure solvent by a semipermeable membrane,
the solvent molecules pass through it from solution side to pure solvent side.
Reason: Diffusion of solvent occours from a region of low concentration solution to region of
high concentration solution.
8 | Page
4 Assertion: Ethanol + Acetone solution does not obey Raoult’s law.
Reason: Raoult’s law is a special case of Henry’s law.
5 Assertion: Molar mass of acetic acid in benzene calculated using colligative property is almost
double the actual value.
Reason: Acetic acid dimerises in solution.
ANS 1 (d) 2(c) 3 (d) 4(b) 5(a)
SHORT ANSWER TYPE QUESTIONS (2 MARKS)
1. What is the value of van’t Hoff factor
(i) For a solution when the solute undergoes association.
(ii) For a solution when the solute undergoes dissociation.
Ans (i) i< 1 (ii) i>1
2. Suman took two glasses of water from a water filter. She cools one glass in a fridge and warms
the other glass on a stove. Which glass of water will hold more dissolved oxygen? Explain using
Henry's law.
Ans Glass kept in a fridge will contain more oxygen. Because according to Henry’s law the partial
pressure of the gas in vapour phase (p) is proportional to the mole fraction of the gas (x) in the
solution. p = KH x
Henry constant is directly proportional to temperature. If temperature increases solubilty of gas
will be decreases.
3. When 2.56 g of sulphur was dissolved in 100 g of CS2, the freezing point lowered by 0.383 K.
B Calculate the formula of (SX). (KF for CS2=3.83 Kkg mol-1, atomic mass of sulphur =32 g mol-1)
Ans ∆TF= Kf Wb x 1000/Mb x Wa
0.383=3.83 x 2.56 x 1000/Mb X 100
Mb=256 S × x =256, 32 × x =256, x=8
4 Give reasons for the following-
(i) Mixture of ethanol and acetone shows positive deviation from Raoult’s Law.
(ii) The preservation of fruits by adding concentrated sugar solution protects against bacterial
action.
Ans (i) The intermolecular interactions between ethanol and acetone are weaker/ the escaping
tendency of ethanol and acetone molecules increases on mixing / the vapour pressure
increases.
(ii) Due to osmosis, a bacterium on fruit loses water, shrivels and dies.
5. Why NaCl solution freezes at lower temperature than water but boils at higher temperature
than water?
Ans When a solute is dissolved in a solvent, the vapour pressure decreases. As a result, the solution
boils at a high temperature while solvent freezes at lower temperature.
6 a) How the osmotic pressure of 5 % aqueous solution of glucose (π1) is related to that of 5 %
aqueous solution of urea (π2)?
b) Why do salt water fish die when they are suddenly transferred to a fresh water aquarium?
Ans a) π1 = π2 (isotonic solutions)
b) Water from aquarium enters in cell causing them to expand and get ruptured.
7 a) Why the colligative property of an electrolyte solution is always greater than that of a non-
electrolyte solution?
b) Ravi wants to put an egg with outer shell removed in a bottle but he is unable to do so as
mouth of bottle is slightly smaller.Suggest one method to help him putting the peeled egg in
the bottle.
Ans a) Due to dissociation of electrolyte the number of ions increases
b) Peeled egg shrinks in saturated brine solution.
8 Why is Benzene insoluble in water but soluble in Toluene?
Ans Polar solute dissolve in polar solvent & non-polar (Benzene) solute dissolved in non-polar
(Toluene) solvent
9 i) How de-icing agent works?
ii) A 500 gm toothpaste sample has 0.18 gm fluoride concentration. What is its concentration in

9 | Page
ppm?
Ans i) It lowers the freezing point of water.
ii) Ppm= parts per million= 0.18 x 106/500 =360 ppm
10. Which type of deviation is shown by Carbon tetrachloride and chloroform mixture? Give reason.
Ans Positive Deviation, A-A & B-B interactions are stronger than those of the A-B interactions,
increase in vapour pressure.
SHORT ANSWER TYPE QUESTIONS (3 MARKS)
1 a) Give reasons.
i) Scuba divers develop bends.
ii) Feeling of weakness and discomfort in breathing at high altitude.
b) Why does soda water bottle kept at room temperature fizz on opening?
Ans i) Scuba divers cope with high concentrations of dissolved gases, when divers come towards
surface, pressure decreases. This releases the dissolved gases and leads to the formation of
bubbles of nitrogen in the blood & develops bends.
ii) At high altitude pressure is decreases, solubility of gases also decreases. This causes person
to become weak & discomfort in breathing.
iii) Soda water bottles are sealed under pressure.
2 a) What is the degree of dissociation for 0.1M Ba(NO3)2 if i (Van’t Hoff factor) is 2.74.
b) Arrange the following solutions in increasing order of Van’t Hoff factor.
0.1M CaCl2, 0.1M KCl, 0.1M C12H22O11, 0.1 M Al2(SO4)3
Ans a) α = i- 1/n-1 2.74-1/3-1 1.74/2=0.87where,n=3
b) 0.1M C12H 22O11< 0.1M KCl < 0.1M CaCl2 < 0.1 M Al2(SO4)3
3 A glucose solution which boils at 101.04oC at 1 atm. What will be relative lowering of vapour
pressure of an aqueous solution of urea which is equimolal to given glucose solution? (Given: Kb
for water is 0.52 K kg mol-1)
Ans ΔTb = Kb m ΔTb = 101.04-100 = 1.04oCor
m= 1.04 /0.52 = 2 m
2 m solution means 2 moles of solute in 1 kg of solvent. 2 m aqueous solution of urea means 2
moles of urea in 1kg of water. No. of moles of water = 1000/18 = 55.5
Relative lowering of V.P. = X2 (where X2 is mole fraction of solute)
Relative lowering of V.P. = n2/n1+n2 (n2 is no. of moles of solute, n1 is no. of moles of solvent)
= 2/ 2+55.5 = 2/57.5 = 0.034
4 i) Why freezing/melting point of a substance used as a criterion for testing the purity of a
substance?
ii) Measurement of osmotic pressure is preferred for the for the determination of molar masses
of macromolecules such as protein & polymers. Give two reasons.
Ans i) Freezing/Melting point changes due to impurities.
ii) Its value is large even for very dilute solutions, so polymers having poor solubility can use
this method to find their molar mass. Proteins & polymers are unstable at high temperatures.
5. Find the vapour pressure of water and its relative lowering in the solution which is 50 g of
urea (NH2CONH2) dissolved in 850 g of water. (Vapour pressure of pure water at 298 K is 23.8
mm Hg)
Ans Mole of urea = 50/60 =0.833, Moles of water = 850/18 = 47.222
Mole fraction of urea(xB) = 0.0173
(P0A – PA) /P0A = xB, (23.8-PA)/23.8=0.0173PA = 23.39
V P of water in the given solution is 23.39 mm of Hg and its relative lowering is 0.0173.
LONG ANSWER TYPE QUESTIONS (5 MARKS)
1. i) Calculate the freezing point of a solution containing 0.5 g KCl (Molar mass = 74.5 g/mol)
dissolved in 100 g water, assuming KCl to be 92% ionized. Kf of water = 1.86 K kg / mol.
ii) Which of the following solutions has higher freezing point?
0.05 M Al2(SO4)3, 0.1 M K3[Fe(CN)6 ] Justify.
Ans i) KCl → K+ + Cl- n =2 i= 1- + n i=1+
∆Tf = i.kfm= (1+ 0.92) x1.86x0.5x1000/74.5x100 =0.24
∆Tf =Tfo -TfTf=-0.24oC
10 | Page
ii) 0.05 M Al2(SO4)3 has higher freezing point. 0.05 M Al2(SO4)3: i = 5,
Tf α No of particles; Tf = i x concentration = 5 x 0.05 = 0.25 moles of ions
0.1 M K3 [Fe (CN)6] i = 4,Tf = 4 x 0.1 = 0.4 moles of ions
2. i) While performing practicals it is directed that the bottle of liquid ammonia is to be cooled
before opening seal but Suman forgot to cool it before opening. What consequence she faced ?
ii) Sia’s father is suffering from high blood pressure but he is advised to consume less quantity
of common salt. Why?
iii) Out of 1M sucrose and 1M urea solution which has more osmotic pressure?
iv) A compound CuSO4.5H2O undergoes complete dissociation in water. What will be its value of
Van’t Hoff factor?
v) If molality of dilute solution is doubled,what will be the value of molal elevation
constant(Kb)?
Ans i) It is cooled to lower down the pressure of ammonia else the gas will bump out of the bottle.
ii) More salt use will increase ions in the body fluid which increases blood pressure
iii) Both have same osmotic pressure.
iv) i=2
v) Kb dosen’t depend upon ‘m’ so remain unchanged.

CASE STUDY BASED QUESTIONS (4 MARKS)


Anand is investigating values of van't Hoff factor 'i' for strong electrolytes KCl, NaCl, MgSO4 and K₂SO4, at
various concentrations. He recorded the following values of different concentrations.
Values of 'i' van't Hoff factor'i' for complete
Salt dissociation ofsolute
0.1 m 0.01 m 0.001 m
NaCl 1.87 1.94 1.97 2.0
KCI 1.85 1.94 1.98 2.0
MgSO4 1.21 1.53 1.82 2.0
K₂SO4 2.32 2.70 2.84 3.0

Observe the table carefully and answer the questions that follow:
(a) What is van't Hoff factor (i)?
(b) What happens to value of 'i' of electrolytes with dilution and why?'
(c) A solution contains 5.85 g of NaCl (Molar mass = 58.5 g/mol) per litre of solution. It has an osmotic
pressure of 4.75 atm at 27°C. Calculate the degree of dissociation of NaCl in this solution.
(Given R = 0.082 L atm K-1 mol-1).
OR
0.3 g of CH₂COOH (M = 60 g mol-¹) dissolved in 30 g of benzene shows a depression in freezing point
equal to 0.45°C. Calculate the percentage association of acid. [K, for benzene = 5.12 K kg mol-1]
(a) Van’t Hoff factor: i May be defined as the ratio of normal molecular mass to observed molecular
mass or the ratio of observed colligative property to calculated colligative property.
(b) On dilution as volume of solution increases, thus dissociation of electrolytes increases hence
‘i‘value also increases.
5.85 1 𝑖−1 1.93−1
(c) a) π=i CRT4.75=𝑖 × 58.5 × 1 × 0.082 × 300𝑖 = 1.93 ∝=
𝑛−1
=
2−1
= 0.93 𝑜𝑟 93%
[OR]
If ‘i ’ is the Van’t Hoff factor, then depression in freezing point –
𝑖 × 𝐾𝑓 × 𝑤1 × 1000
ΔT𝑓 = ; 𝑤𝑕𝑒𝑟𝑒 ΔT𝑓 = 0.45𝐾, 𝑤1 = 0.3𝑔 , 𝑤2 = 30𝑔 , 𝑀2 𝐶𝐻3 𝐶𝑂𝑂𝐻 = 60𝑔 , 𝐾𝑓 = 5.12
𝑀2 × 𝑤2
ΔT𝑓 × 𝑀2 × 𝑤2 0.45 × 30 × 60
𝑖 = = = 0.5213
𝐾𝑓 × 𝑤1 × 1000 5.12 × 0.3 × 1000
Van’t Hoff factor is 0.5273

11 | Page
ELECTROCHEMISTRY
Quick Revision Points
Electrochemistry: The study of production of electricity from energy released during spontaneous
chemical reactions and the use of electrical energy to bring about non-spontaneous chemical
transformations.
Electrochemical cell: A device in which chemical energy of the redox reaction is converted into
electrical energy. e.g., Daniel cell or Galvanic cell
The overall cell reaction is: Zn(s) + Cu2+ (aq) → Zn2+ (aq) + Cu(s)
The Daniel cell is represented as:
Zn(s)| Zn2+(aq)(C1) || Cu2+(aq) (C2)| Cu(s)
Oxidation half Reduction half
Half-cell reactions (redox couples) can be written as:
(a)At zinc electrode (oxidation), Zn metal dissolves, it acts as
anode (-), written on Left side
Zn(s) → Zn2+ (aq) + 2e –(oxidation)
(b) At copper electrode (reduction), Cu is deposited, it acts as
cathode (+), written on right side.
Cu2+ (aq) + 2e- → Cu(s) (reduction)
Salt Bridge: It consists of an inverted glass U-tube containing
semi-solid paste of an inert electrolyte such as KCl, KNO3 or
NH4Cl in gelatin or agar-agar jelly.
Functions of salt bridge: It helps in flow of ions by
completing the circuit and maintains electrical neutrality.

Electrode potential (E): The potential difference between the electrode and the electrolyte interface.

Standard electrode potential(E°):The electrode potential under standard conditions i.e; when
concentration of all the species involved in a half-cell is unity, T is 298K and P is 1 bar.

Cell potential (Ecell) Electromotive force (emf)


Difference between the electrode potentials Cell potential when no current is drawn
(reduction potentials) of the cathode and anode. through the cell.
(Measured in Volts).Ecell = Eright − Eleft

Note: By convention, the standard electrode potentials are reduction potential values.
Reduction potential = - Oxidation potential.
Reversibility of Daniel Cell:

12 | Page
Standard Hydrogen electrode (SHE):
It is a reference electrode which consists of Pt electrode
coated with Pt black, in acidic solutionof 1M H+ ion
concentration at 1 bar Pressure and 298 K temperature
having reduction potential zero. It can act both ways - as an
anode or as a cathode. SHE is represented as:
Pt(s) | H2(g, 1bar)│H+(aq, 1M)
Measurement of electrode potential: The electrode Potential of
unknown electrode can be measured by constructing an
electrochemical cell using SHE. Since the E° value of SHE is zero,
the standard cell potential directly gives the E° value of the
0 0 0
unknown electrode according to relation, Ecell = Ecathode − Eanode
Example: (i) When SHE is couples with Zn, Zn(s)|Zn2+(aq, 1M) || H+(aq, 1M)|H2(g, 1bar) |Pt(s)
EMF of the above cell = 0.76 V so E° of Zn+2/Zn electrode = -0.76V
(ii) When SHE is couples with Cu, Pt(s) | H2 (g, 1bar) |H+(aq, 1M)|| Cu2+(aq, 1M) | Cu(s)
EMF of the above cell = 0.34 V so E° of Cu /Cu+2 electrode = 0.34V
Nernst equation: It is an equation which gives the relationship between electrode potential and the
concentration of ions. When the concentration of any of the electrolytes in the two half cells is not
unity, then Nernst equation is used to determine the cell potential. For an electrode reaction
(reduction reaction), Mn+ (aq) + ne– → M(s), Nernst equation can be written as:
o RT M o 2.303RT 1
EM n +/M = EM n + /M − ln n+ EM n +/M = EM 𝑛 + /M − log n+
nF M nF M
where, EM n +/M Electrode potential, E o n + = Standard electrode potential, R = 8.314 JK −1 mol−1 ,
M /M
T = Temperature in kelvin, n = No. of electrons gained, F = Faraday constant 96500 C/mol
Substituting the value of R and F we get
o 0.0591 1
EM n +/M = EM n + /M − log⁡ n+ at 298K
n M
Thus, the reduction potential increases with the increase in the concentration of ions.
ne −
For a general electrochemical reaction of the type: aA + bB cC + dD
Nernst equation can be given as:
o RT [C]c [D]d o 2.303RT [C]c [D]d
Ecell = Ecell − ln⁡ a b Ecell = Ecell − log⁡ a b
nF [A] [B] nF [A] [B]
Substituting the values of R and F we get
∘ 0.0591 [C]c [D]d
Ecell = Ecell − log⁡ a b , at 298K
n [A] [B]
For example, (i) Nernst equation for Daniel cell (Zn – Cu cell) can be written as:
Here number of electrons n = 2 for both Zn and Cu ion.
RT 1
For Cathode: E Cu 2+/Cu = E 0Cu 2+/Cu − 2F ln⁡Cu 2+(aq )
RT 1
For Anode: E zn 2+ /Zn = E 0Zn 2+/Zn − 2F ln⁡Zn 2+(aq )
The cell potential, E(cell ) = E − E Zn 2+/Zn
Cu 2+ /Cu

o RT Zn2+ o 0.059 Zn2+


E(cell ) = E(cell ) − ln⁡ 2+ E(cell ) = E(cell ) − log⁡ 2+
2F Cu 2 Cu
(ii) Nernst equation for Ni – Ag cell can be written as:
Here n = 2 for Ni and n = 1 for Ag and should use the same number of electrons (𝑛) for both the
electrodes. Cell representation is Ni(s) Ni2+ aq ∥ Ag + aq Ag(s)
The cell reaction is Ni(s) + 2Ag +(aq) → Ni2+(aq) + 2Ag(s)
The Nernst equation can be written as
o RT Ni2+
E(cell ) = E(cell ) − ln⁡ + 2
2F Ag

13 | Page
Equilibrium Constant from Nernst Equation: For the Daniel cell, at equilibrium
∘ 2.303RT Zn2+ ∘ 2.303RT Zn2+
Ecell = 0 = Ecell − log E = log⁡
2F Cu2+ cell 2F Cu2+
Zn2+
But at equilibrium = Kc
Cu2+

o 2.303RT ∘ 0.0591
Ecell = log K c Ecell = log⁡K c
2F 2
∘ 0.0591 n ∘
In general, Ecell = n log⁡K c or,log⁡K c = 0.0591 Ecell
Electrochemical cell and Gibbs free energy:
The work done by a reversible galvanic cell is equal to decrease in its free energy.
Mathematically, Δr G = −nFEcell

If concentration of all the reacting species is unity, then, Ecell = Ecell and we get,
o ∘
Δr G = −nFEcell
FromΔr G , we can calculate the equilibrium constant of a reaction,
o

Δr Go = − RT ln K c or Δr Go = −2.303 RT log⁡K c
Electrochemical Series: The arrangement of various electrodes in the order of their decreasing
standard reduction potential values is known as electrochemical series.
Metallic and Electrolytic Conductance:

S. No. Metallic or electronic Conductance Electrolytic Conductance


(i) It is due to movement of electrons. It is due to movement of ions.

(ii) Does not involve transfer of matter. Matter moves in the form of ions.

Decreases with increase in temperature Increases with increase in temperature


(iii) as kernels start vibrating which produce due to decrease in interionic attraction or
hindrance in the flow of electrons. increase in dissociation.

Conductors Insulators Semiconductors Superconductors


Very high Very low Conductivity between Infinite conductivity
conductivity. conductivity. conductors and insulators. or zero resistivity.
e.g., Metals and alloys, e.g., Glass, e.g. Si, doped Si, Gallium e.g. Al, Nb
Carbon black, graphite ceramics arsenic
Metallic conductance depends upon - Nature and structure of the metal, Number of valence
electrons per atom and Temperature.
Electrolytic or ionic conductance depends upon - Nature of electrolyte or interionic attractions,
Solvation of ions, Nature of solvent and its viscosity, and Temperature.
Measurement of Conductance: The resistance of an electrolytic solution filled in a conductivity cell
is determined by Wheatstone bridge method having variable resistance (R1), fix resistance (R3 and R4)
and unknown resistance (R2 = R) of electrolyte solution. A null point detected by P detector such that,
R1 R3 R1 R 4
= or R 2 =
R2 R4 R3
The reciprocal of 𝑅2 gives the conductance (G) of the solution as,
1 l
κ= ×
R A
l/A = G* (called as cell constant). The cell constant is usually determined by measuring the resistance
of the cell containing a solution whose conductivity is already known. It depends on the distance
between the electrodes and their area of cross-section.
Conductance of Electrolytic solutions:
l
Resistance (R): It is obstruction to the flow of current. R = ρ A . Its SI unit is ohm.

14 | Page
Resistivity (ρ): Electrical resistance of a conductor of unit cross-sectional area and unit length. Its SI
A
unit is ohm metre. ρ = R l
Property Formula Units Effect of dilution

1 A κA Increases as larger
Ohm−1 Ω−1 /
Conductance (G) G= = = number of ions are
R ρl l Siemens (S)
produced.
Specific conductance Ohm−1 cm−1 /
1 𝑙
𝜅 = 𝜌 =𝐺 𝐴 = G x G* Decreases as number of
(𝜅, kappa) or
Sm−1 / Scm−1 ions per cm3 decreases.
conductivity

Molar conductivity Λm = κ × V S cm2mol–1/S Increases with dilution


1000
Λm =κ×
M
m2 mol–1 due to large increase in 𝑉.

Conductivity: The conductance of an electrolytic solution when placed between the electodes
separated by 1cm distance and having cross-section area 1 sq. cm
Molar conductivity: The conductance of 1 molar electrolytic solution when placed between the
electodes separated by 1cm distance and having cross-section area 1 sq. cm
Limiting molar conductivity: When concentration approaches zero i.e., at infinite dilution, the molar
conductivity is known as limiting molar conductivity (Λ° m).
Variation of Conductivity and Molar Conductivity with Concentration:
For a strong electrolyte: It is shown by Debye–Huckel Onsager equation: Λm = Λ°m – A 𝑐,
Here, Λ°m = Molar conductivity at infinite dilution (Limiting molar conductivity) Λ m = Molar
conductivity at concentration C, A = Constant which depends upon nature of solvent and temperature,
C = Concentration. Plot of Λm against 𝑐is a straight line with intercept equal to Λ°m and slope equal
to ‘– A’. Thus, Λm decreases linearly with 𝑐 , when C = 0, Λm = Λ°m and Λ°m can be determined
graphically by extrapolating the graph.
There is only a small increase in conductance with dilution. This is because a strong electrolyte is
completely dissociated in solution, so the number of ions remains constant and on dilution, interionic
attractions decrease as ions move far apart.
For weak electrolytes: There is a very large increase in
conductance with dilution especially near infinite dilution as
number of ions increases. Λm increases as concentration
decreases but does not reach a constant value even at infinite
dilution. Hence, their Λ°m cannot be determined experimentally.
Kohlrausch’s Law: It states that the limiting molar conductivity
of an electrolyte can be represented as the sum of the individual
contributions of the anion and cation of the electrolyte.
In general, if an electrolyte on dissociation gives 𝑣+cations and
𝑣−anions then its limiting molar conductivity is given by
Λ0m = v+ λ0+ + v− λ0−
Here, λ∘+ and λ∘− are the limiting molar conductivities of cations and anions, respectively.
Applications of Kohlrausch’s law:
(a) Calculation of molar conductivities of weak electrolyte at infinite dilution: eg, molar conductivity of
acetic acid (weak acid) at infinite dilution can be obtained from the knowledge of molar conductivities
at infinite dilution of strong electrolytes like HCl, CH3 COONa and NaCl as illustrated below:
Λ∘m CH 3 COOH = λ∘CH 3 COO − + λ∘H ∗ = λoCH 3 COO − + λoNa + + λoH + + λoCl − − λoNa + + λoCl −
i.e., Λom CH 3 COOH = Λom CH 3 COONa + Λom(HCl ) − Λom(NaCl )
Λ cm
(b) Determination of degree of dissociation of weak electrolytes: Degree of dissociation (α) =
Λ om
(c) Determination of dissociation constant (K) of weak electrolytes:
15 | Page
cα2 c Λcm /Λom 2 c Λcm 2
K = ∴K = =
1−α 1 − Λcm /Λom Λom Λom − Λcm
(d) Determination of solubility of sparingly soluble salts:
κ × 1000 κ × 1000 κ × 1000
Λom = = or, Solubility =
Molarity Solubility Λom
Electrolytic cell and electrolysis: Electrolysis is the process of decomposition of an electrolyte by
passing electricity through its aqueous solution or molten state.
Products of Electrolysis:
Products Reactions involved
Electrolyte
At At
cathode anode At cathode At anode

Na Cl2 1
Molten NaCl Na+ −
(l) + e → Na (l) Cl−
(l) → Cl2(g) + e

metal gas 2
Aqueous Cl2 1 1
H2 gas −
H2 O(l) + e− → H2(g) + OH(aq Cl−
(aq ) → Cl2(g) + e

NaCl gas 2 )
2
O2 1 + −
Dil. H2 SO4 H2 gas +
H(ag −
) + e → H2(g)
2H2 O(l) → O2(g) + 4H(aq ) + 4e
gas 2
1
Conc. H2 SO4 H2 gas S2 O2−
8
+
H(aq −
) + e → H2(g)
2SO2− 2−
4(aq ) → S2 O8 (aq) + 2e

2
AgNO3(aq)-Pt O2 + −
Ag Ag+(aq) + e- → Ag(s) 2H2 O(l) → O2(g) + 4H(aq ) + 4e
electrode gas
AgNO3(aq)-
Ag Ag+ Ag+(aq) + e- → Ag(s) Ag(s) → Ag+(aq) + e-
Ag electrode
+
CuSO4(aq)-Pt Cu O2 Cu2+(aq) + 2e- → Cu(s) 2H2 O(l) → O2(g) + 4H(aq ) + 4e

electrode gas
CuSO4(aq)- Cu Cu2+ Cu2+(aq) + 2e- → Cu(s) Cu(s) → Cu2+(aq) + 2e-
Cu electrode

Overvoltage/Over potential: Oxidation of H2O is relatively slow process and thus needs extra
potential. This extra potential needed to oxidise H2O is called overvoltage/over potential. Due to
overvoltage, the oxidation of chloride ion occurs at anode in preference to H2O.
Discharge potential: The minimum potential that must be applied across the electrodes to bring
about the electrolysis and thus discharge of the ions on the electrode is known as discharge potential.
It is in fact the ability of ions to discharge first at electrodes.
Faraday’s first law of electrolysis: The amount of chemical reaction which occurs at any electrode
during electrolysis is proportional to the quantity of electricity passed through the electrolyte.
m = Z × I × t, where Z = Electrochemical equivalent
Faraday’s second law of electrolysis: amount of various substances liberated by the same quantity
of electricity passed through the electrolytic solution is proportional to their chemical equivalent
weights. W1/E1 = W2/E2
Battery: Combination of galvanic cells in series and used as a source of electrical energy.
(i) Primary batteries are non-chargeable batteries such as Leclanche cell and Dry cell.
(ii) Secondary batteries are chargeable cells involving reversible reaction. Example: Lead storage
battery and Nickel-cadmium cells.
Dry cell (Leclanche cell): The anode consists of a zinc container and the cathode is a graphite
electrode surrounded by powdered MnO2 and C. The space is filled with paste of NH4Cl and ZnCl2.
At anode: Zn(s) → Zn2+(aq) + 2e– At cathode: MnO2 (s) + NH4+(aq)+ 2e– → MnO(OH) + NH3
The net reaction: Zn + NH4 (aq) + MnO2 → Zn2+ + MnO(OH) + NH3
+

16 | Page
Mercury cell: consists of zinc–mercury amalgam as anode and a paste of HgO and carbon as the
cathode. The electrolyte is a paste of KOH and ZnO. The electrode reactions are:
At Anode: Zn(Hg) + 2OH-→ ZnO(s) + H2O + 2e –
At Cathode: HgO(s) + H2O + 2e- → Hg (l) + 2OH–
The cell potential is approximately 1.35 V and remains constant as the ionic concentration of the
solution is not changed during its life.
Lead storage battery: Anode - Spongy lead; Cathode - Lead packed with Lead dioxide
Electrolyte -Aqueous solution of H2SO4 (38%)
Discharge reaction of cell:
At anode: Pb(s) +SO42–(aq) → PbSO4 (s) +2e–
At cathode: PbO2 filled in lead grid gets reduced to Pb2+ ions which combines with SO42– ions to form
PbSO4 (s). The reaction is PbO2 (s) + 4H+(aq) + SO42–(aq) + 2e– → PbSO4 (s) + 2H2O(l)
Complete cell reaction: Pb(s) + PbO2 (s) + 2H2SO4 (aq) → 2PbSO4 (s) + 2H2O (l)
Recharge reaction of cell: It changes the direction of electrode reaction. PbSO4 accumulated at
cathode gets reduced to Pb.
At cathode: PbSO4 (s) +2e– → Pb(s) + SO42–(aq)
At anode: PbSO4 gets oxidised to PbO2
PbSO4 (s) + 2H2O → PbO2 (s) + 4H+(aq) + SO42–(aq) + 2e–
Overall Reaction: PbSO4 (s) + 2H2O (l) → Pb(s) + PbO2 (s) + 2H2SO4 (aq)
Nickel–cadmium cell has longer life than the lead storage cell but is costly. Here, the overall reaction
during discharging: Cd(s) + 2Ni(OH)3 (s)→ CdO(s) + 2Ni(OH)2 (s) + H2O(l)
Fuel cells: Electrical cells that are designated to convert the energy
from the combustion of fuels such as hydrogen, carbon monoxide or
methane directly into electrical energy are called fuel cells.

The cell reactions are:


Anode: 2H2 (g) + 4OH– (aq) → 4H2O (l) + 4e-,
Cathode: O2 (g) + 2H2O (l) + 4e– → 4OH– (aq)
Net reaction: 2H2 (g) + O2 (g) →2H2O (l)

Advantages of fuel cell: pollution free, 75% efficiency, continuous source of energy.
Corrosion: The process of slow conversion of metals into their undesirable compounds (usually
oxide) by reaction with moisture and other gases present in the atmosphere. Rusting of iron:
Oxidation: Fe(s) → Fe2+ (aq) + 2e-,
Reduction: O2(g) + 4H+(aq) + 4e-→ 2H2O(l)
Atmospheric oxidation: 2Fe2+(s) + ½ O2 (g) + 2H2O(l) → Fe2O3+ 4H+(aq)
Fe2O3 + xH2O → Fe2O3.xH2O
Prevention of Corrosion:
(i)Barrier protection: By covering the surface with paint or a thin film of grease or by electroplating.
(ii) Sacrificial protection: By galvanization. (iii)Alloying

MULTIPLE CHOICE QUESTIONS (1 MARK)


Q1. Which metal is used as electrode which do not participate in the reaction but provides surface for
conduction of electrons?
(a) Cu (b) Pt (c) Zn (d) Fe
Q2. An electrochemical cell can behave like an electrolytic cell when
(a) Ecell = 0 (b) Ecell> Eext (c) Eext> Ecell (d) Ecell = Eext
Q3. Which cell will measure standard electrode potential of copper electrode?
(a) Pt(s)| H2 (g, 0.1 bar)| H+ (aq., 1 M) | | Cu2+ (aq., 1M)| Cu(s)
(b) Pt(s)| H2 (g, 1 bar) | H+ (aq., 1 M) | | Cu2+ (aq., 2M)| Cu(s)
(c) Pt(s)| H2 (g, 1 bar) | H+ (aq., 1 M) | | Cu2+ (aq., 1M)| Cu(s)
(d) Pt(s)| H2 (g, 1 bar) | H+ (aq., 0.1 M)| | Cu2+ (aq., 1M)| Cu(s)
Q4. The positive value of the standard electrode potential of Cu2+/Cu indicates that
(a) this redox couple is a stronger reducing agent than the H+ /H2 couple.

17 | Page
(b) this redox couple is a stronger oxidising agent than H+ /H2.
(c) Cu2+ can displace H2 from acid. (d) Cu can displace H2 from acid
Q5. On increasing temperature,
(a) ionic conductance increases and electronic conductance decreases.
(b) ionic conductance decreases and electronic conductance increases.
(c) both ionic and electronic conductance increase.
(d) both ionic and electronic conductance decrease.
Q6. The electrolyte used in the mercury cell is
(a) paste of NH4Cl and ZnCl2 (b) paste of HgO and carbon
(c) paste of KOH and ZnO (d) paste of PbO and H2SO4
Q7. A device that converts energy of combustion of fuels like hydrogen and methane directly into
electrical energy is known as:
(a) dynamo (b) Ni-Cd cell (c) fuel cell (d) electrolytic cell
Q8. Which one of the following is always true about the spontaneous cell reaction in a galvanic cell?
(a) E° cell> 0, ΔG° < 0, Qc>KC (b) E° cell< 0, ΔG° < 0, Qc<KC
(c) E° cell> 0, ΔG° > 0, Qc>KC (d) E° cell> 0, ΔG° < 0, Qc<KC
Q9. Charge carried by 1 mole of electrons is
(a) 6.023 × 1023 coulomb (b) 9.65 × 104 coulomb
(c) 1.6 × 10–19 coulomb (d) 6.28 × 1019 coulomb
Q10. To calculate the standard emf of the cell, which of the following options is correct if E° is
reduction potential values?
(a) emf = E°cathode – E°anode (b) emf = E°anode – E°cathode
(c) emf = E°anode + E°cathode (d) None of these
Q11. The quantity of charge required to obtain one mole of aluminium from Al2O3 is __________.
(a) 1F (b) 6F (c) 3F (d) 12F
Q12. What will happen during the electrolysis of aqueous solution of CuSO4 by using platinum
electrodes?
(a) Copper will deposit at cathode. (b)Copper will deposit at anode.
(c) Oxygen will be released at cathode. (d) Copper will dissolve at anode.
Q13. While charging the lead storage battery ______________.
(a) PbSO4 anode is reduced to Pb. (b) PbSO4 cathode is reduced to Pb.
(c) PbSO4 cathode is oxidised to Pb. (d) PbSO4 anode is oxidised to Pb.
Q14.Three faradays of electricity are passed through molten Al2O3, aqueous solution of CuSO4and
molten NaCl taken in differentelectrolytic cells. The amount of Al, Cu and Na deposited at thecathodes
will be in the ratio of
(a) 1 mole : 2 mole : 3 mole (b) 3 mole : 2 mole : 1mole
(c) 1 mole: 1.5 mole: 3 mole (d) 1.5 mole : 2 mole : 3 mole
Q15. Copper cannot replace….. from its salt solution
(a) Fe (b) Au (c) Hg (d) Ag
Answers :
1 2 3 4 5 6 7 8 9 10 11 12 13 14 15
b c c b a c c d b a c a b c a
ASSERTION REASON TYPE QUESTIONS (1MARK)
Given below are two statements labelled as Assertion (A) and Reason (R)
Select the most appropriate answer from the options given below:
a. Both A and R are true and R is the correct explanation of A
b. Both A and R are true but R is not the correct explanation of A.
c. A is true but R is false.
d. A is false but R is true.
Q1. Assertion: Ecell should have a positive value for the cell to function.
Reason: Ecathode< Eanode
Q2. Assertion: Copper sulphate cannot be stored in zinc vessel.
Reason: Zinc is less reactive than copper.
Q3. Assertion: Current stops flowing when Ecell = 0.
18 | Page
Reason: Equilibrium of the cell reaction is attained.
Q4. Assertion: Conductivity of all electrolytes decreases on dilution.
Reason: On dilution, number of ions per unit volume decreases.
Q5. Assertion: Conductivity decreases for weak electrolyte and increases for strong electrolyte with
decrease in concentration.
Reason: On dilution, the number of ions per unit volume that carry the current decreases.
Answers: 1(c), 2( c), 3(a), 4(a), 5(d)

SHORT ANSWER TYPE QUESTIONS (2 MARKS)


Q1. In the plot of molar conductivity (Λm) vs square root of concentration (C1/2), following curves are
obtained for two electrolytes A and B.

Answer the following:


(i) Predict the nature of electrolytes A and B.
(ii) What happens on extrapolation of Λm to concentration approaching zero for electrolytes A and B?
Ans. (i) A is a strong electrolyte while B is a weak electrolyte.
(ii) For electrolyte A, the plot becomes linear near high dilution and thus can be extrapolated to zero
concentration to get the molar conductivity at infinite dilution.
For weak electrolyte B, Λm increases steeply on dilution and extrapolation to zero concentration is not
possible. Hence, molar conductivity at infinite dilution cannot be determined.
Q2. Calculate Δr G∘ and log⁡K c for the following reaction at 298𝐾
0
2Cr⁡ (s) + 3Fe2+(aq) ⟶ 2Cr 3+(aq) + 3Fe(s) Given: Ecell = 0.30V
n 0
Ans.n=6, log⁡K c = 0.059 Ecell = 3.224 x 1030, ΔG° = -nFE°cell = -1737.7KJ/mol, or log Kc = 30.5084
Q3. Three iron sheets have been coated separately with three metals 𝐴, 𝐵 and 𝐶 whose
standardelectrode potentials are given below. Identify in which rusting will takes place faster when
coating is damaged.
E 0 for Metals A, B, C and Iron are − 0.46V, −0.66V, −0.20V and − 0.44V respectively.
Ans. As iron (−0.44𝑉) has lower standard reduction potential than 𝐶 (−0.20𝑉) only and therefore
when coating is broken, rusting will take place faster.
Q4. Can you store copper sulphate solutions in a zinc pot?
Ans. For this we have to check whether the following reaction will take place or not.
Zn s + CuSO4 aq ⟶ ZnSO4 aq + Cu s
o o o
Ecell = ECu 2+ /Cu − EZn 2+ /Zn = 0.34 − (−0.76) = 1.10V

No, asEcell is positive, the reaction will take place.
Q5. Suggest a way to determine the Λ0m value of water.
Ans.
Λom H 2 O = λoH + + λoOH − = λoH + + λoOH − + λoCl − − λ0Cl − + λ0Na + − λ0Na +
Rearranging we get
Λom H2O = λoH + + λoCl − + λoNa + + λoOH − − λoNa + + λoCl − Λom H2O
Λom(HCl )
= + Λom(NaOH )
− Λom(NaCl )
Thus, the molar conductivity of water at infinite dilution can be determined from the knowledge of
Λom(HCl ) , Λom(NaOH ) and Λom (NaCl ) .
Q6. Why on dilution the ⋀m of CH3COOH increases drastically while that of CH3COONa increases
gradually?
19 | Page
Ans.CH3COOH is a weak electrolyte, the number of ions increase on dilution due to an increase in
degree of dissociation.
Q7. (a) What is the role of ZnCl2 in a dry cell?
(b) Unlike dry cell, the mercury cell has a constant cell potential throughout its useful life. Why?
Ans (a) ZnCl2 combines with the NH3 produced to form the complex salt [Zn(NH3)2 Cl2] otherwise the
pressure developed due to NH3 would crack the seal of the cell
(b) Ions are not involved in the overall cell reaction of mercury cells
Q8. (a) Value of standard electrode potential for the oxidation of Cl– ions is more positive than that of
water, even then in the electrolysis of aqueous sodium chloride, why is Cl– oxidised at anode instead of
water?
(b) How will the pH of brine (aq. NaCl solution) be affected when it is electrolysed?
Ans. (a) On electrolysis of aqueous sodium chloride, oxidation of water at anode requires over
potential hence Cl– is oxidised instead of water.
(b) The pH of the solution will increase as NaOH is formed in the electrolytic cell.
Q9. (a) Which type of a metal can be used in cathodic protection of iron against rusting?
(b) Write the name of the electrolyte used in fuel cell
Ans. (a) More electropositive metal than iron such as Al, Zn, Mg etc. (b) Conc. Aq. KOH solution
Q10. Depict the galvanic cell in which the reaction Zn(s) + 2Ag+ (aq) → Zn2+ (aq) + 2Ag(s) takes place.
Further show: (i) Which of the electrode is negatively charged?
(ii) The carriers of the current in the cell.
Ans. Cell can be represented as Zn | Zn2+ (aq) ||Ag+ (aq) |Ag.
(i) Zinc electrode is negatively charged (anode)
(ii) Ions are the current carriers within the cell
SHORT ANSWER TYPE QUESTIONS (3 MARKS)
Q1. (a) Calculate the cell emf and ΔG∘ for the cell reaction at 25∘ 𝐶 for the cell:
Zn s Zn2+ 0.0004M ∥ Cd2+ 0.2M Cd s
∘ ∘
At 25∘ C: EZn 2+ /Zn = −0.763V; ECd 2+ /Cd = −0.403V; F = 96500Cmol
−1
; R = 8.314JK −1 mol−1 .
(b) If 𝐸 ∘ for copper electrode copper ions is 0.34𝑉 ? How does emf for copper electrode change
when concentration of Cu2+ ions is decreased? how will you calculate its emf value when the
solution in contact with it is 0.1𝑀 in the solution
∘ ∘ ∘
Ans. (a)Ecell = Ecathode − Eanode = −0.403 − −0.763 = 3.36V
The net cell reaction is,Zn(s) + Cd2+
(aq )
→ Zn2+
(aq )
+ Cd(s) , n=2
(b) Cu2+
aq

+ 2e → Cu s

∘ 0.0591 Zn2+
Ece ll = Ecell− log = 0.44 V, ΔG = −nFEcell = −84920J/mol
2 Cd2+
∘ 0.059 [Cu] 0.059 1 0.059
ECu 2+/Cu = ECu 2+ /Cu − log⁡ 2+ = 0.34 − log⁡ = 0.34 − log⁡10
2 Cu 2 0.1 2
0.059
= 0.34 − × 1 = 0.34 − 0.0295 = 0.3105𝑉
2
When the concentration of Cu2+ ion is decreased, the electrode potential for copper decreases.
Q2. The molar conductivity of sodium acetate, sodium chloride and hydrochloric acid are 83,127 and
426 mho cm2 mol−1 at 250∘ C respectively. Calculate the molar conductivity of acetic acid solution.
Ans. Given: Λ∘m CH3 COONa = 83 mho cm2 mol−1
Λ∘m NaCl = 127 mho cm2 mol−1 Λ∘m HCl = 426 mho cm2 mol−1 Λ∘m CH3 COOH = ?
Using Kohlrausch law of independent migration of ions
Λ∘m CH3 COOH = Λ∘m CH3 COONa + Λ∘m (HCl) − Λ∘m (NaCl)
Λ∘m CH3 COOH = 83 + 426 − 127 = 382 mho cm2 mol−1
Q3. A strip of nickel metal is placed in a 1 molar solution of Ni NO3 2 and a strip of silver metal is
placed in a 1 molar solution of AgNO3 . An electrochemical cell is created when the two solutions are
connected by a salt bridge and the two strips are connected by wires to a voltmeter.
(i) Write the balanced equation for the overall reaction occurring in the cell and calculate the cell

20 | Page
potential.
(ii) Calculate the cell potential, E, at 25∘ C for the cell if the initial concentration of Ni NO3 2 is 0.100
molar and the initial concentration of AgNO3 is 1.00 molar.
0 0
ENi 2+ /Ni = −0.25V; EAg + /Ag = 0.80V, log⁡10−1 = −1
Ans. Cell reaction: Ni + 2Ag+ → Ni2+ + 2Ag,
E°Cell = E°cathode - E°anode, E°Cell = 1.05V
o 0.059 Ni 2+
(ii)Ecell = Ecell − log log = 1.0795V
n Ag + 2
Q4. The electrochemical cell given alongside converts the chemical energy released during the redox
reaction to electrical energy: Zn s + Cu2+ aq ⟶ Zn2+ aq + Cu s It gives an electrical potential of
1.1V when concentration Zn2+ and Cu2+ ions is unity. State the direction of flow of current and also
specify whether zinc and copper are deposited or dissolved at their respective electrodes when:
(i) an external opposite potential of less than 1.1 V is applied.
(ii) an external potential of 1.1𝑉 is applied.
(iii) an external potential of greater than 1.1𝑉 is applied.
Ans. (i) Reaction continues to take place. Electrons flow from Zn electrode to copper electrode,
current flows from Cu to Zn. Zn dissolves and copper deposits at their respective electrodes.
(ii) The reaction stops and no current flows.
(iii) Reaction takes place in opposite directions. Electrons flow from copper electrode to zinc
electrode, current flows from Zn to𝐶𝑢. Cell functions as an electrolytic cell.

Q5. Explain redox potential. Reduction potentials of some ions are given below. Arrange them in
decreasing order of oxidizing power.

Ion 𝐶𝑙𝑂4− 𝐼𝑂4− 𝐵𝑟𝑂4−

Reduction potential 𝐸 ⊖ /𝑉 𝐸 0 = 1.19𝑉 𝐸 0 = 1.65𝑉 𝐸 0 = 1.74𝑉

Ans. It is a measure of the tendency of a chemical species to acquire electrons from or lose electrons
to an electrode and thereby be reduced or oxidized respectively. The more positive the reduction
potential of a species, the greater the species' affinity for electrons and tendency to be reduced the
higher the reduction potential, the higher is its tendency to get reduced. Hence, the order of oxidizing
power is: 𝐵𝑟𝑂4− > 𝐼𝑂4− > 𝐶𝑙𝑂4−

LONG ANSWER TYPE QUESTIONS (5 MARKS)


Q1. (i) State two advantages of H2—O2 fuel cell over ordinary cell.
(ii) Silver is electrodeposited on a metallic vessel of total surface area 500 cm2 by passing a current of
0.5 A for two hours. Calculate the thickness of silver deposited. [Given: Density of silver = 10.5 g cm–3,
Atomic mass of silver = 108 amu, F = 96,500 C mol–1]
Ans. (i) Pollution free, 75% efficiency, continuous source of energy.
(ii) w =ZIt = 4.029g, V=m/d = 0.3837 cm3,
Thickness of Ag deposited is x, V= A. x = 7.67x10-4cm
Q2. (i) Define limiting molar conductivity and fuel cell.
(ii) Resistance of a conductivity cell filled with 0.1 mol L–1 KCl solution is 100 ohm. If the resistance of
the same cell when filled with 0.02 mol L–1 KCl solution is 520 ohm, calculate the conductivity and
molar conductivity of 0.02 mol L–1 KCl solution. The conductivity of 0.1 molL–1 KCl solution is 1.29 ×
10–2 ohm–1 cm–1.
Ans (i) Limiting Molar conductivity -limiting value of molar conductivity when concentration
approaches to zero. Fuel cell - device which converts energy produced during the combustion of fuels
directly into electrical energy.
(ii) Cell constant = conductivity x resistance = 1.29 cm–1, Conductivity, κ = Cell constant/Resistance =
0.00248 ohm–1 cm–1, Λm = κx 1000/M = 124 ohm–1 cm2 mol–1

21 | Page
CASE BASED QUESTIONS
Read the passage carefully and answer the questions.
A process in which on passing electricity electrolyte split into ions and ions are deposited at opposite
electrode is called electrolysis.
Faraday’s First law of electrolysis: The amount of substance deposited or liberated during electrolysis
is directly proportional to the quantity of electricity passed through the solution.
Faraday's Second law of electrolysis When same amount of current is passed through different
electrolytes which are connected in series, the amount of substance deposited/ liberated from
different electrolytes is directly proportional to their equivalent weight.
1. Define electrochemical equivalent (Z).
2. How much electricity in terms of Faraday is required to produce20⋅0 g of Ca from molten CaCl2?
OR
2. How much electricity in terms of Faraday is required to produce40⋅0 g of Al from molten Al2O3?
3. A solution of Ni (NO3)2 is electrolysed between platinum electrodes using a current of 5 amperes
for 20 minutes. What mass of Ni is deposited at the cathode?
OR
3. A solution of CuSO4 is electrolysed for 10 minutes with a current of 1⋅5 amperes. What is the mass
of substance deposited at the cathode?
Ans- 1. Electrochemical equivalent is defined as the amount of substance deposited or liberated by the
passage of 1 ampere of current for one second i.e. one coulomb charge.
2. Ca2+ + 2e− → Ca To produce 40 g Ca, electricity needed = 2F
∴ To produce 20 g Ca, electricity needed = 40/ 2 × 20 = 1F
OR
2. Al3+ + 3e− → Al To produce 27 g Ca, electricity needed = 3F
∴ To produce 40 g Al, electricity needed = 27/ 3 × 40 = 4⋅44F
3. Quantity of electricity passed (Q) = Current in amperes × time in second = 5 × 20 × 60 = 6000 C
Ni+2 + 2e− → Ni
i.e. 2 × 96487 C of electricity deposit Ni = 58⋅5 g
∴ 6000 C of electricity deposit Ni =(58.5/2 × 96487) × 6000 = 1⋅82 g
OR
3. Quantity of electricity (Q) = Current × time = 1⋅5 × 10 × 60 = 900 C
According to the reaction: Cu+2 + 2e− → Cu
We required 2 F or 2 × 96487 C of electricity to deposit 1 mol or 63 g of Cu
∴ 900 C electricity will deposit = (63/2 × 96500) × 900 = 0⋅2938 g of Cu at the cathode.

CHEMICAL KINETICS
Quick Revision Points
For any chemical reaction - the feasibility can be predicted by the thermodynamics, its extent can be
determined from chemical equilibrium and speed of a reaction can determined in chemical kinetics.
Chemical kinetics: -Branch of chemistry, which deals with the study of reaction rates, factors
affecting reaction rate and their mechanisms, called chemical kinetics.
Based on reaction rate, chemical reactions can be classified in three categories: -
Type of reaction Example
Very fast reactions (Reactions which occur Reaction of AgNO3 and NaCl
instantaneously) Reactions of strong acids and strong bases.
Very slow reactions (Reactions which occur very Rusting of iron
slow)
Reactions which occur with moderate rates Inversion of cane sugar and hydrolysis of
starch.
Rate of reaction: The rate of a reaction can be defined as the change in concentration of a reactant or
product in unit time.
*Rate of reaction can be expressed in terms of: (i) the rate of decrease in concentration of any one of
the reactants, or (ii) the rate of increase in concentration of any one of the products.

22 | Page
Average rate of reaction: It is the disappearance of reactants or appearance of products over a long-
time interval.
Consider a hypothetical reaction, assuming that the volume of the system remains constant.
R P, Rate of disappearance of R= Decrease in concetration of R/Time taken ,
[ R ]
Rate of disappearance of R= − , {Since, ∆[R] is a negative quantity (as concentration of
t
reactants is decreasing), it is multiplied with –1 to make the rate of the reaction appositive quantity}
Rate of appearance of P = increase in concetration of P/Time taken ,
[ P ]
Rate of appearance of P=
t
*For expressing the rate of such a reaction where stoichiometric coefficient of reactants and products
are not equal to one, rate of disappearance of any of the reactant or the rate of appearance of product
is divided by their respective stoichiometric coefficients.
1 [ A] 1 [ B ] 1 [C ] 1 [ D ]
Consider a reaction: aA + bB  cC + dD, rav= - =- = =
a t b t c t d t
For example, in following reaction rate of consumption of HI is double than rate of formation of H2and
I2. 2HI (g)→ H2(g) +I2(g)

Instantaneous rate of reaction: It is defined as the change in concentration of any one of the
reactants or products at that particular instant of time. The change in concentration and change in
time is denoted by d (small change)

*Instantaneous rate of reaction may be calculated by taking tangent of the graph mentioned below.
Unit of rate of reaction - In liquid state or in aqueous solution unit of rate of reaction will be
mol L-1s–1. If a reaction is completed in gaseous phase then unit will be atm s–1 / bar s–1
Factors affecting Rate of reaction:
(a)Concentration of reactants (pressure in case of gases): -In general, If the Concentration of
reactants is increased than number of collisions among them will increase and rate of reaction will
increase.
(b)Temperature: -In most of the cases, the rate of chemical reaction increases by increasing the
temperature. Because by increasing the temperature the average energy of the molecules will increase
and a greater number of molecules will be able to cross the energy barrier.
(C) Catalyst –Catalyst can change the rate of reaction by changing its activation energy.
Rate law: - Rate law is the expression in which reaction rate is given in terms of molar concentration
of reactants with each term raised to some power, which may or may not be same as the
stoichiometric coefficient of the reacting species in a balanced chemical equation.
Consider a reaction: aA + bB  cC + dD , Rate  [A]x[B]y
 dR
Rate = K[A]x[B]y or = K[A]x[B]y…….(1) , (Note: The value of x & y may or may
dt
not be equal to a &b. x & y are determined by experimental data).
Equation (1) is known as differential rate equation, where k is a proportionality constant called rate
constant or specific rate of reaction. It is defined as - If the concentration of all reactants is one unit
 dR
than rate of reaction is equal to rate constant. [A]= B]=1mol-1, Equation (1) may be written as =K
dt
*In some cases, these exponents may be equal to stoichiometric coefficients.
For example- 2NO(g) + O2(g) →2NO2(g), Rate = k[NO]2[O2]
*In some cases, exponents of concentrations may differ from stoichiometric coefficients.
For example-(i) CHCl3 + Cl2→CCl4 + HCl, Rate = k [CHCl3 ] [Cl2]1/2
(2) CH3COOC2H5 + H2O →CH3COOH + C2H5OH, Rate = k [CH3COOC2H5]1 [H2O]0
Rate law for any reaction cannot be predicted by merely looking at the balanced chemical equation,
I.e., theoretically but must be determined experimentally.
23 | Page
Order of reaction -The sum of powers of the concentration of the reactants in the rate law expression
is called the order of that chemical reaction. Order of a reaction is an experimental quantity.
In the above-mentioned equation aA + bB  cC + dD,
 dR
According to equation no. (1) Rate = = K[A]x[B]y Order of Reaction(n)= x + y
dt
Order of a reaction can be 0, 1, 2, 3 and even a fraction
Units of Rate constant- Rate = K[A]x[B]y
K is rate constant. It may be written as- K=Rate /[A]x[B]y
K=Concentration /time ×1(Concentration)n K= (Concentration )1-n/time
Taking SI units of concentration in mol L–1 and time in s, the units of k = (mol L-1 )1-n s-1 where n is
order of reaction. For example
Reaction Order Unit
Zero order of reaction 0 K =molL-1s-1
First order of reaction 1 K=s-1
Second order of reaction 2 K= mol- 1L s-1
*The reactions taking place in one step are called elementary reactions.
*When a sequence of elementary reactions (called mechanism) gives us the products, the reactions
are called complex reactions. In complex reactions the slowest step is rate determining step.
Consider the decomposition of hydrogen peroxide which is catalysed by iodide ion in an alkaline
medium 2H2O2 I  2H2O + O2,
-
, Alkaline

On the basis of experimental evidences, Rate law for this reaction is mentioned as-
 d [ H 2 O2 ]
Rate = = K [ H 2O2 ] [I-]
dt
Mechanism: From the above mentioned rate law the following mechanism may be suggested because
two species (H2O2 and I-)mentioned in rate law should participate in slow step (rate determining
step)- (i) H2O2 + I– H2O + IO–(slow) (ii) H2O2 + IO– H2O + I– + O2 (Fast)
Molecularity of reaction: The number of reacting species (atoms, ions or molecules) taking part in an
elementary reaction, which must collide simultaneously in order to bring about a chemical reaction
is called molecularity of a reaction.
*The molecularity greater than three is not observed because the probability that more than three
molecules can collide and react simultaneously is very small.
*Molecularity cannot be zero or a non-integer.
S. No. Order of reaction Molecularity of reaction
1 Experimental property Theoretical property
2 It may be any number, It always an integer excepting zero
fractional, integral or even zero
3 It may change with change in It cannot change
experimental condition
4 It is applicable to elementary as It is applicable only for elementary reactions.
well as complex reactions For complex reaction molecularity has no meaning

Integrated Rate law of equations: -ZERO ORDER REACTIONS & FIRST ORDER REACTIONS-
ZERO ORDER REACTIONS FIRST ORDER REACTIONS-

For this type of For this type of reactions, the rate of the reaction is
reactions, the rate of the proportional to the first power of the concentration
reaction is proportional of the reactant R.
to zero power of the Consider the reaction,
concentration of RP
reactants.  d [ R]
Rate = = K[R]1 …(1)
Consider the reaction, RP dt

24 | Page
 d [ R]  d [ R]  d [ R]
Rate = = K[R]0  = K ….. (1)  = K dt ….(2)
dt dt [ R]
 d [R] = - K dt ……..(2)  d [ R]
Integrating both sides ∫ = K∫ dt
Integrating both sides ∫ d [R] = - K ∫ dt [ R]
 [R ] = - K t + C ……. (3) -ln[R]=kt + C ……..(3)
where, C is the constant of integration. where, C is the constant of integration
At t = 0, the concentration of the reactant At t = 0, the concentration of the reactant R = [R]0,
R = [R]0, where [R]0 is initial concentration where [R]0 is initial concentration of the reactant.
of the reactant. [R]0= - K × 0 + C  [R]0= C -ln[R]0=k × 0 + C  C=-ln[R]0
Substituting the value of C in equation (3) Substituting the value of C in equation (3)
 [R ] = - K t + [R]0  -ln[R]=kt -ln[R]0 …..…..(4)
[R] 0  [ R] 1
K = K = { ln[R]0 - ln[R]}
t t
Variation in the 1 [ R ]0 2.303 [ R ] 0
K= ln  k= log
concentration vs t [ R] t [ R]
time plot for a zero Variation in the concentration vs time plot for a
order reaction First order reaction
If we plot [R] This graph express the
against t, we get a straight line with slope = equation (4)
–k and intercept equal to [R]0. -ln[R]=kt -ln[R]0
Half life period for a zero order reaction: Comparing with y=mx + c
The half-life of a reaction is the time in If we plot In [R] against t we
which the concentration of a reactant is get a straight line with slop
reduced to one half of its =-K and intercept =In [R]0
initial concentration. It is Equation (4) may also be
represented as t1/2. written as In [R]/[R]0 = - kt
[R] 0  [ R] Or In [R]0/[R] = kt
t=
k
[R] Half life period for a First
At t1 / 2 , [R ] = 0
2 order reaction :
[R] 0  [ R]0 / 2 [ R]
t1 / 2 = t1 / 2  2.303 [R ] 0
k 2K t1 / 2 = log
k [ R]0 / 2
(Half-life for zero order reaction  Initial
concentration of reactant) t1 / 2 =0.693/k
Examples of zero order reactions;- (For a first order reaction,
(a) The decomposition of gaseous ammonia half-life period is constant, it does not depend on
on a hot platinum surface is a zero order initial concentration of the reactant)
reaction at high pressure. Examples of first order reaction:- All natural and
(b)The thermal decomposition of HI on gold artificial radioactive decay of unstable nuclei take
surface. place by first order kinetics
For first order gas phase reactions –
A(g) → B(g) + C(g)
At t=0 p1atm 0 atm 0 atm
At time t (p1–x ) atm xatm xatm

25 | Page
Rate equation for radioactive decay-
𝑘=2.303/t log N0/Nt (N0 is initial number of atoms and Nt is number of atoms after t time)
Pseudo first order reaction: The reaction which is bimolecular but order is one is known as Pseudo
first order reaction. In this type of reaction one of the reactants is present in large excess i.e. the
concentration does not get altered much during the course of the reaction
e.g. (i) Hydrolysis of ester
H
CH3COOC2H5 + H2O  CH3COOH + C2H5OH, Rate = K[CH3COOC2H5]
(ii) Inversion of cane sugar is another example of pseudo first order reaction
H
C12H22O11+ H2O  C6H12O6 + C6H12O6, Rate = K[C12H22O11]
Temperature Dependence of the Rate of a Reaction: Most of the chemical reactions are accelerated
by increase in temperature. For a chemical reaction with rise in temperature by 10°, the rate constant
k t+10 k 308 K
is nearly doubled. Temperature coefficient = =k ≃2
kt 298 K
The temperature dependence of the rate of a chemical reaction can be accurately explained by
Arrhenius equation 𝑘 = 𝐴𝑒−𝐸𝑎/RT where A = the Arrhenius factor or the frequency factor or pre-
exponential factor. R = gas constant and 𝐸𝑎= activation energy measured in joules/mole.
*It can be understood using following reaction- H2(g)+I2(g)
→2HI(g)

According to Arrhenius, this reaction can take place only when a


molecule of hydrogen and a molecule of iodine collide to form an
unstable intermediate. It exists for a very short time and then
breaks up to form two molecules of hydrogen iodide.
*The energy required to form this intermediate, called
activated complex (C), is known as activation energy(Ea).
Taking natural logarithm of both sides of equation (1)
In K = In A ‒Ea/RT …….(2) , where: slope=-Ea/R and intercept =In A
log K = log A ‒ Ea/2.303RT …….(3) ,
At temperature T1 log K1 = log A ‒ Ea/2.303RT1 …….(4)
At temperature T2 log K2 = log A ‒ Ea/2.303RT2 …….(5)
Subtracting eq. (4) from eq. (5) , we obtain-
K1 is rate constant on temperature t1
K2 is rate constant on temperature t2
*It has been found from Arrhenius
equation (1) that increasing the temperature or decreasing
the activation energy will result in an increase in the rate of
the reaction and an exponential increase in the rate constant.
Effect of Catalyst:-The catalyst provides an alternate
pathway or reaction mechanism by reducing the activation
energy between reactants and products and hence lowering
the potential energy barrier.
*A catalyst does not alter Gibbs energy, ΔG of a reaction.
It catalyzes the spontaneous reactions but does not catalyze
nonspontaneous reactions. It is also found that a catalyst
does not change the equilibrium constant of a reaction
rather, it helps in attaining the equilibrium faster.
Collision Theory of Chemical Reactions-According to this theory, the reactant molecules are
assumed to be hard spheres and reaction is postulated to occur when molecules collide with each
other. For a bimolecular elementary reaction, A + B → Products, rate = 𝑍𝐴𝐵 𝑒−𝐸𝑎/𝑅𝑇
where 𝑍𝐴𝐵= collision frequency of reactants, A and B and 𝑒−𝐸𝑎/𝑅𝑇 = fraction of molecules with energies
equal to or greater than 𝐸𝑎.
*Comparing with Arrhenius equation, we can say that A is related to collision frequency.

26 | Page
*The collisions in which molecules collide with sufficient kinetic energy (called threshold energy) and
proper orientation, leads to the formation of products are called as effective collisions.
*To account for effective collisions, another factor P, called the probability or steric factor is
introduced. It considers the fact that in a collision, molecules must be properly oriented
rate = 𝑃𝑍𝐴𝐵 𝑒−𝐸𝑎/𝑅𝑇
*Thus, in collision theory activation energy and proper orientation of the molecules together
determine the criteria for an effective collision and hence the rate of a chemical reaction.
*Collision theory also has certain drawbacks as it considers atoms/ molecules to be hard spheres and
ignores their structural aspect.

MULTIPLE CHOICE QUESTIONS (1 MARK)


1. If the rate of the reaction is equal to the rate constant, the order of the reaction is
(a) 3 (b) 0 (c) 1 (d) 2
2. In the reaction, A + 2B  6C + 2D, if the initial rate – d [A]/dt at t = 0 is 2.6 × 10–2 M sec–1, what will
be the value of – d [B]/dt at t = 0?
(a) 8.5 × 10–2 M sec–1 (b) 2.5 × 10–2 M sec–1 (c) 5.2 × 10–2 M sec–1 (d) 7.5 × 10–2 M sec–1
3. The rate constant (K1) of one reaction is double of the rate constant (K2) of another reaction. Then
the relationship between the corresponding activation energies of the two Rxns (Ea1 and Ea2) will be:
(a) Ea1>Ea2 (b) Ea1 = Ea2 (c) Ea1< Ea2 (d) Ea1 = 4 Ea2
4. For a reaction, AB, it has been found that the order of the reaction is zero with respect to A.Which of
the following expression correctly describes the reaction?
(a) k=2.303/t log A0/A (b) [A0]-[A]=kt (c) t1/2 =0.693/k (d) t1/2 α 1/[A0]
5. What is the order of a reaction which has a rate expression, rate  K[ A] [B]3/2 1

(a) 3/2 (b) ½ (c) 0 (d) None of these


6. An exothermic reaction, X Y, has an activation energy 30 kJmol–1. If energychange (ΔE) during the
reaction is–20 kJ, then the activation energy for the reverse reaction is
(a) –30kJ (b) 20KJ (c) 50kJ (d) 10kJ
7. In a certain reaction, 10% of the reactant decomposes in 1 h, 20% in 2 h, 30% in 3 h, and so on. The
dimensions of the rate constant are:
(a) s−1 (b) mol L-1 s -1 (c) mol-1 L s -1 (d) mol s -1
8. A catalyst alters, which of the following in a chemical reaction?
(a) Enthalpy change (b) Gibbs energy (c) Internal energy (d) Activation energy
9. For the reaction A  B  products, doubling the concentration of A the rate of the reaction is doubled,
but on doubling the concentration of B rate remains unaltered. The over all order of the reaction is
(a) 1 (b) 0 (c) 2 (d) 3
10. The half- life (in seconds) for a first order reaction which takes 4 seconds for 30% completion is
(a) 6.65 (b) 5.45 (c) 8.95 (d) 7.77
11. The unit of rate constant of second order reaction is usually expressed as
(a) mol- 1L-1 s-1 (b) mol L-1 s-1 (c) s-1 (d) mol- 1L s-1
𝑑[𝐴]
12. For a reaction 2A→3B, rate of disappearanceof A, − is equal to
𝑑𝑡
3 𝑑[𝐵] 2 𝑑[𝐵] 1 𝑑[𝐵] 2 𝑑[𝐵]
(a) + 2 𝑑𝑡 (b) + 3 𝑑𝑡 (c) + 3 𝑑𝑡 (d) + 1 𝑑𝑡
13. The slope in the plot of log[R]0/[R] vs. time for a first-order reaction is
+𝑘 −𝑘
(a) 2.303 (b) -k (c) 2.303 (d)+k
14. The reaction 2NO (g) + O2 (g) → 2NO2(g) follows third order kinetics. If volume of reaction vessel is
reduced to 1/3, the rate of reaction would be
(a) 1/3 times (b) 2/3 times (c)27 times (d) 9 times
15. If the rate equation between A and B is expressed as r= k[A][B] , the reaction is:
2

(a) First order in A (b) Second order in B (c) Overall third order (d) All are correct
Answer:

Q1 Q2 Q3 Q4 Q5 Q6 Q7 Q8 Q9 Q10 Q11 Q12 Q13 Q14 Q15


b c c b b c b d a d d b a c d

27 | Page
ASSERTION REASON TYPE QUESTIONS (1MARK)
Select the most appropriate answer from the options given below:
(a) Both A and R are true and R is the correct explanation of A
(b) Both A and R are true but R is not the correct explanation of A.
(c) A is true but R is false. (d) A is false but R is true.
1. Assertion: The enthalpy of reaction remains constant in the presence of a catalyst.
Reason: A catalyst lowers down the activation energy but the difference in energy of reactant and
product remain same.
2. Assertion: All collisions of reactant molecules lead to product formation.
Reason: Only those collisions in which molecules have correct orientation and sufficient kinetic
energy lead to compound formation.
3. Assertion: The molecularity of the reaction H2+Br2 → 2HBr appears to be 2.
Reason: The two molecules of the reactant are involved in the given elementary reaction.
4. Assertion: If the activation energy of a reaction is zero, temperature will have no effect on
the rate constant.
Reason: Lower the activation energy, faster is the reaction.
5. Assertion: The slowest elementary step in a complex reaction decides the rate of the reaction.
Reason: The slowest elementary step always has the smallest molecularity in all the steps.
Answers:
Q1 Q2 Q3 Q4 Q5
a d a b c

SHORT ANSWER TYPE QUESTIONS (2 MARKS)


1. The rate constant for a first order reaction is 60 s–1. How much time will it take to reduce the
initial concentration of the reactant to its 1/10th value?
Answer- t = 2.303/k log [A0]/[A]
= 2.303/ 60 log 1/1/10 = 0.038 s
2. A first order reaction found to have a rate constant k = 5.5 x10 -14 s -1 find the half life of the
reaction,
Answer-For first order reactiont ½ = 0.693/k
t ½ = 0.693/5.5 x10 -14 = 1.26 X 10 13 s
3. If hydrolysis of cane sugar is completed in acidic medium then its rate of reaction depends only
on one molecule of cane sugar. (i) What will be the name of the reaction (ii) What will be unit
of rate constant for this reaction?
Answer-( i) Pseudo first order reaction (ii) unit of k will be s-1
OR
4. From the following graph of rate Vs concentration determine the order of reaction.

Answer – (i) Order of reaction is zero. (ii) Order of reaction is First


5. For an elementary reaction A(g) → B(g) + C(g) the half-life period is 10 minutes. In what period
of time would the concentration of A be reduced ti 10 % of original concentration?
Answer- The reaction is following first order sok = 2.303 log [ R ] t=33 minutes
0

t [ R]

6. Detrive the general for of expression for the half life of the first order reaction.
Answer k=2.303/t1/2 logR0/R
k=2.303/ t 1/2 logR0/R0/2 k=2.303/ t 1/2 log2 k=2.303/ t 1/2 0.3010 t 1/2= 0.693/k
28 | Page
7. Write two differences between order and molecularity.
Answer
order molecularity
Can be fractional Whole number
Determine rate of reaction Doesnot determine rate of reaction
8. (i) What is quantitative relationship between temperature and rate constant.
(ii) Define collision frequency.
Answer –(i)k = A e -Ea /RT
(ii) The number of collisions per second per unit volume of the reaction mixture is known
collision frequency (Z).
9. (a) In a complex reaction which step controls the overall rate of reaction and what is it called?
(b) From the rate expression for the following reactions determine their order of reaction and
the dimensions of the rate constants:
H2O2(aq) +3I- +2H+ ----2H2O(Liq) +I3- Rate =k[H2O2] [I-]
Answer- (a) slowest step, which is called rate determining step.
(b)

10. Analyse the given graph, drawn between concentrations of reactant vs. time.

(a) Predict the order of reaction.


(b) Theoretically, can the concentration of the reactant reduce to zero
after infinite time? Explain.
Answer- (a) 1st order (b) No, due to exponential relation / the curve
never touches the x-axis.

SHORT ANSWER TYPE QUESTIONS (3 MARKS)


1. A reaction is first order in A and second order in B.
(i) Write the differential rate equation.
(ii) How is the rate affected on increasing the concentration of B three times?
(iii) How is the rate affected when the concentrations of both A and B is doubled?
Answer --- (i) d(A)/dt =k (A) (B)2
(ii) (3B)2= rate become nine (9)times
(iii) (2A) (2B)2= 8 times
2. The rate of the chemical reaction doubles for and increase of 10 K in absolute temperature
from 298 K. Calculate Ea.
Answer – k1= k and k2= 2k, T1= 298K T2 =308K
logk2/k1= Ea/2.303R (T2-T1/T1T2)
log2k/k= Ea/2.303 x 8.314(308-298/308 x 298)
log2= Ea/2.303 x 8.314(10/308 x 298)
0.3010 = Ea/2.303 x 8.314(10/308 x 298)
Ea = 52.8kJ/mol
3. Write the factors which affect the rate of a chemical reaction.
Concentration of reactants: The rate of a chemical reaction is proportional to the
concentration of the reacting species taking part in the reaction. It is maximum to start with
and slowly decreases since the concentration of the reacting species decreases accordingly.
Temperature: Actually, the energy of the reactant species increases with the increase in
temperature and so will be number of collisions. Hence rate increase’
Presence of catalyst: Generally, increase the rate of reaction. Or any other factors

29 | Page
4. For the reaction A + B → products, the following initial rates were obtained at various given
initial concentrations. Determine the half-life period.
S.N. A mol/L B mol/L Initial rate M/s

1 0.2 0.2 0.1


2 0.4 0.2 0.2
3 0.2 0.4 0.1
Answer-For A-Rate =k[A] x For B Rate =k[B] y

From case 1 and 2, 2× Rate =k[2A]x From case 1 and 3, Rate =k[2B] y
X=1, y=0 Order n=x+y=1+0=1
5. From the rate expression for the following reactions, determine their order of reaction and the
dimensions of the rate constants.
(i) 3NO (g) → N2O (g) Rate = k[NO]2
(ii) H2O2 (aq) + 3I– (aq) +2H+→ 2H2O (l) + 3 I− Rate = k[H2O2] [I-]
(iii) CH3CHO (g) → CH4 (g) + CO(g) Rate = k [CH3CHO]3/2
Answer—(i) 2 (ii)2 (iii)3/2
CASE STUDY BASED QUESTIONS (4 MARKS)
1. Read the passage given below and answer the following questions:
There are nuclear reactions constantly occurring in our bodies, but there are very few of them
compared to the chemical reactions, and they do not affect our bodies much. All of the physical
processes that take place to keep a human body running are chemical processes. Nuclear reactions
can lead to chemical damage, which the body may notice and try to fix. The nuclear reaction
occurring in our bodies is radioactive decay. This is the change of a less stable nucleus to a more
stable nucleus. Every atom has either a stable nucleus or an unstable nucleus, depending on how big
it is and on the ratio of protons to neutrons. The ratio of neutrons to protons in a stable nucleus is
thus around 1:1 for small nuclei (Z < 20). Nuclei with too many neutrons, too few neutrons, or that
are simply too big are unstable. They eventually transform to a stable form through radioactive
decay. Wherever there are atoms with unstable nuclei (radioactive atoms), there are nuclear
reactions occurring naturally.
The interesting thing is that there are small amounts of radioactive atoms everywhere: in your chair,
in the ground, in the food you eat, and yes, in your body. The most common natural radioactive
isotopes in humans are carbon-14 and potassium-40. Chemically, these isotopes behave exactly like
stable carbon and potassium. For this reason, the body uses carbon-14 and potassium-40 just like it
does normal carbon and potassium; building them into the different parts of the cells, without
knowing that they are radioactive. In time, carbon-14 atoms decay to stable nitrogen atoms and
potassium-40 atoms decay to stable calcium atoms. Half-life of c-14 is 6000 years
(The Practical Science by Paul B. Kelter, Michael D. Mosher and Andrew Scott states)

(a) Why is Carbon -14 radioactive while Carbon -12 not? (Atomic number of Carbon: 6)
(b) Which are the two most common radioactive decays happening in human body?
(c) Suppose an organism has 20 g of Carbon -14 at its time of death. Approximately how much
Carbon -14 remains after 10,320 years? (Given antilog 0.517 = 3.289)
OR
(c) Approximately how old is a fossil with 12 g of Carbon -14 if it initially possessed 32 g of Carbon -
14? (Given log 2.667 = 0.4260)
Answer-(a) Ratio of neutrons to protons is 2.3: 1 which not the stable ratio of 1:1 is
(b) carbon-14 atoms decay to stable nitrogen atoms and potassium-40 atoms decay to stable calcium
(c) t = 2.303/ k log (Co/Ct), Co = 20 g Ct =? t = 10320 years
k = 0.693/6000 (half-life given in passage) substituting in equation:
10320 = 2.303 / (0.693/6000) log 20/ Ct 0.517 = log 20 / Ct
antilog (0.517) = 20/Ct 3.289 = 20/Ct Ct = 6.17 g
OR
t = 2.303/ k log (Co/Ct)
Co = 32 g, Ct = 12, t =? k = 0.693/6000 (half-life given in passage) substituting in equation:
30 | Page
t = 2.303 / (0.693/6000) log 32/ 12
t = 2.303 x 60000 /0.693 log 2.667 t = 2.303x6000x0.4260 /0.693 = 8494 years
LONG ANSWER QUESTIONS (5 MARKS)
1. For a certain chemical reaction variation in concentration, In[R] Vs time (s) plot is given alongside.
For this reaction write/draw:
1. What is the order of the reaction?
2. What is the unit of rate constant (k)?
3. Give the relationship between k and t1/2(half-life period).
4. What does the slope of above line indicate?
5. Draw the plot of log [R0]/[R] vs time (s)
Answer: 1. First order 2. time-1 (s-1)
0.693
3. k= 𝑡 4. Slope=-k (Rate constant)
1/2

2. (a) Define the following terms:


(i) Collision frequency(ii) Rate constant (k)
(b) The rate constant of a first order reaction increases from 4 x 10-2 to 24 x 10-2 when the
temperature changes from 300 K to 350 K. Calculate the energy of activation (Ea). (log 2 = 0.3010, log
3 = 0.4771, log 4 = 0.6021, log 6 = 0.7782)
Answer: (a) (i) Collision frequency: It is defined as the number of collisions taking place per second
per unit volume.
(ii) Rate Constant: It is equal to the rate of reaction when molarity is 1M.
24 𝑥 10 −2
𝑘
b) 𝑙𝑜𝑔 𝑘 2 = 2.303𝑅
1
𝑎
(𝑇 − 𝑇 )
𝐸 1
1
1
2
, = log 4 𝑥 10 −2
𝐸
= 2.303𝑥8.314
𝑎
(300 − 350 )
1 1

19.147𝑥300𝑥350
E a= log 6, Ea = 19.147x2100x0.7782, Ea=31290Jmol-1 = 31.29 k Jmol-1
50

THE d- AND f-BLOCK ELEMENTS


QUICK REVISION POINTS:
1. d-block elements
(a) DEFINITION: A transition element is defined as the one which has incompletely filled d orbitals
in its ground state or in any one of its oxidation states. Zinc, cadmium and mercury of group 12
have full d10 configuration in their ground state as well as in their oxidation states and hence, are
not regarded as transition metals. Their Outer electronic configuration is (n -1) d1-10 ns 1-2.
(b) MELTING AND BOILING POINTS: High MP & BP which is due to their strong metallic bond
(strong interatomic bonding) due to unpaired electrons in (n-1) d orbitals. In any row the m. p. of
these metals rise to a maximum at d5 except for anomalous values of Mn & Tc.
(c) ENTHALPIES OF ATOMIZATION: High enthalpies of atomization Because of large number of
unpaired electrons in their atoms they have stronger metallic bonding.
(d) VARIATION IN ATOMIC AND IONIC SIZES: In general, ions of the same charge in a given series
show progressive decrease in radius with increasing atomic number. The atomic radii decreases
from group 3 to 6 because of increase in effective nuclear charge (Zeff) gradually, The atomic radii
of group 7, 8, 9 and 10 elements is almost same because Screening effect counter balances
increased effective nuclear charge, Group 11 and 12 elements have bigger size due to increase
inter-electronic repulsion as result electron cloud expands and size increases. Members of
second (4d) and the third (5d) series in each group of transition elements have similar radii and
shows similar properties because lanthanoid contraction essentially compensates for the
expected increase in atomic size with increasing atomic number.
31 | Page
(e) DENSITY: The decrease in metallic radius coupled with increase in atomic mass results in a
general increase in the density of these elements. Thus, from titanium (22) to copper (29) the
significant increase in the density may be noted.
(f) VARIATION IN IONISATION ENTHALPIES OF TRANSITION METALS: There is an increase in
ionisation enthalpy along each series of the transition elements from left to right due to an
increase in nuclear charge which accompanies the filling of the inner d orbitals. Irregular variation
of ionisation enthalpies is mainly attributed to varying degree of stability of different 3d-
configurations (e.g., d0, d5, d10 are exceptionally stable).
(g) OXIDATION STATES: The transition elements show variable oxidation state due to small energy
difference between (n-1)d and ns orbital as a result both (n-1) d& ns electrons take part in bond
formation. The highest oxidation state of an element is equal to number of unpaired e - present in
(n-1) d and ns orbital.
(h) MAGNETIC PROPERTIES: Most of the transition metal ions are paramagnetic. Due to the
presence of unpaired electrons in d-orbitals. Magnetic moment is given by

Where, n = number of unpaired electrons and BM = Bohr magneton (unit of magnetic moment).
(i) FORMATION OF COLOURED IONS: Due to the presence of unpaired electrons in d-orbitals
which undergo d-d transition.
(j) FORMATION OF COMPLEX COMPOUNDS: -Due to smaller sizes of the metal ions, their high
ionic charges and the availability of d orbitals for bond formation.
(k) CATALYTIC PROPERTIES: Due to Variable oxidation states (vacant dorbital) and large surface
area.
(l) FORMATION OF INTERSTITIAL COMPOUNDS: Transition elements form interstitial
compounds because size of C, N, O, and B is similar to size of interstitial voids of transition metal
(m) ALLOY FORMATION: As the atomic sizes are very similar one metal can replace the other
metal from its lattice and form a solid solution, which is an alloy.
(n) STANDARD ELECTRODE POTENTIALS: Transition elements have lower negative value of
Standard Electrode Potentials, because of their high ionization potential, high heat of
sublimation & low enthalpy of hydration. The E0 (M2+/M) values are not regular which can be
explained from the irregular variation of ionization enthalpy and sublimation enthalpy of Mn due
to half-filled orbitals.
(o) Oxides of transition metals in lower oxidation states are generally basic while those in the
higher oxidation states are acidic. Acidic character increases with increase in oxidation state is
due to decrease in size of metal ion and increase in effective nuclear charge.e.g. MnO (basic),
Mn3O4 (amphoteric), Mn2O7 (acidic).
2. LANTHANOIDS
(a) The 14 elements after Lanthanum (57) having atomic number 58 to 71 are collectively known as
Lanthanoids. The general electronic configuration of these elements is [Xe] 4f1-14, 5d0-1, 6s2.
(b) LANTHANOID CONTRACTION (ATOMIC AND IONIC SIZES): The overall decrease in atomic and
ionic radii from lanthanum to lutetium is due to poor shielding of 4f electrons is known as
lanthanoid contraction. Due to lanthanoid contraction :- (a)Basic character of oxides and
hydroxides decreases from La(OH)3 to Lu (OH)3, (b) Sizes of 4d and 5d metal pairs like Zr and Hf;
Nb and Ta are almost similar. (c) Separation of lanthanoids becomes difficult.
(c) OXIDATION STATES: Most common oxidation state of these elements is +3. Occasionally +2 (Eu2+
and Yb2+) and +4 ions (Ce4+ and Tb4+) in solution or in solid compounds are also obtained.
(d) COLOUR FORMATION: Many trivalent lanthanoid ions are coloured both in the solid state and in
aqueous solutions. Colour of these ions may be attributed due tothe presence of unpaired
electrons in f subshell.
(e) MAGNETIC BEHAVIOUR: The lanthanoid ions other than the f0 type (La3+ and Ce4+) and the f 14
type (Yb2+ and Lu3+) are all paramagnetic.
(f) USES: A well-known alloy is mischmetall which consists of a lanthanoid metal (~ 95%) and iron (~
5%) and traces of S, C, Ca and Al. A good deal of mischmetall is used in Mg-based alloy to produce
bullets, shell and lighter flint.

32 | Page
3. ACTINOIDS
(a) The 14 elements after Actinium(89) having atomic number 90 to 113 are collectively known
as Actinoids
(b) The actinoids are radioactive elements. These facts render their study more difficult.
(c) Electronic Configurations: The general electronic configuration is [Rn] 5f1-14, 6d0-1, 7s2
(d) Ionic Sizes (Actinoid contraction): The overall decrease in atomic and ionic radii across the
series due to poor shielding of 5f electrons is known as Actinoid contraction.
(e) The actinoid contraction is, however, greater from element to element in this series, is due to
poor shielding by 5f electrons.
(f) Oxidation States: The actinoids show in general +3 oxidation states. The actinoids exhibit a
larger number of oxidation states, which is in part attributed to the fact that the 5f, 6d and 7s
levels are of comparable energies. The elements, in thefirst half of the series, frequently exhibit
higher oxidation states. For example, the maximum oxidation state increases from +4 in Th to
+5, +6 and +7respectively in Pa, U and Np but decreases in succeeding elements.
(g) The actinoids are highly reactive metals, especially when finely divided.
(h) The magnetic properties of the actinoids are more complex than thoseof the lanthanoids.
Although the variation in the magnetic susceptibility of the actinoids with the number of
unpaired 5 f electrons is roughly parallel to the corresponding results for the lanthanoids
4. POTASSIUM DICHROMATE (K2Cr2O7)
(a) Preparation: - It takes place in three steps-
i. 4FeCr2O4+ 8Na2CO3 +7O2   8Na2CrO4(yellow solution)+ 2Fe2O3 +8CO2
ii. 2Na2CrO4 + 2H  +  Na2Cr2O7 (orange sodium) + 2 Na++ H2O
iii. Na2Cr2O7 + 2 KCl   K2Cr2O7 (Orange crystals)+ 2 NaCl
(b) The chromates and dichromates are interconvertible in aqueous solution depending upon pH
of the solution. 2 CrO42– + 2H+→Cr2O72– + H2O Cr2O72– + 2 OH-→2CrO42– + H2O
(c) Potassium dichromate in acidic solution act as oxidizing agent.
 Cr2O72–+ 6I–+ 14H+→ 2Cr3++ 3I2 + 7H2O
 Cr2O72–+ 3H2S+ 8H+→ 2Cr3++ 3S+ 7H2O
 Cr2O72–+ 3Sn2++ 14H+→ 2Cr3++ 3Sn4++ 7H2O
 Cr2O72–+ 6Fe2++ 14H+→ 2Cr3++ 6Fe3+ + 7H2O
(d) Structure of chromates and dichromates

(e) Uses: Potassium dichromate is a very important chemical used in leather industry and as an
oxidant for preparation of many azo compounds. Potassium dichromate is used as a primary
standard in volumetric analysis.
5. POTASSIUM PERMANGNATE [KMnO4]
a) Preparation: -- It takes place in two steps: -
 2MnO2 + 4 KOH + O2   2K2MnO4 (Dark green)+ 2H2O
 3 MnO4 +4H 
2- +  2MnO4-(Purple)+MnO2 +2H2O [Disproportionation]
b) Properties of KMnO4
 Potassium permanganate are isostructural with those of KClO4.
 When heated KMnO4 decomposes at 513 K. 2KMnO4   K2MnO4 + MnO2 + O2
Heat , 

 KMnO4 act as Oxidising agent in acidic, alkaline& neutral medium.


 MnO4–+ 8H++ 5e–   Mn2+ + 4H2O (Acidic)
 2MnO4–+ 16H+ + 5C2O42-   2Mn2+ + 8H2O +10CO2
 MnO4–+ 8H++ 5Fe2+   Mn2+ + 4H2O + 5Fe3+
 2MnO4–+ 16H+ + 10I–   2Mn2+ + 8H2O +5I2

33 | Page
 5NO2-+2MnO4–+ 6H+   2Mn2+ + 3H2O + 5NO3-
 5SO32-+2MnO4– + 6H+   2Mn2+ + 3H2O + 5SO42-
 2MnO4–+ H2O + I–   2MnO2+ 2OH- + IO3–
 8 MnO4– +3 S2O32- + H2O   8MnO2 + 2OH- + 6 SO42-
 2 MnO4–+ 3Mn2++ 2H2O   8MnO2 + 4H+
6. Permanganate titrations in presence of hydrochloric acid are unsatisfactory since hydrochloric
acid is oxidised to chlorine.
7. Uses: Besides its use in analytical chemistry, potassium permanganate is used as a favourite
oxidant in preparative organic chemistry. Its uses for the bleaching of wool, cotton, silk and
other textile fibres and for the decolourisation of oils are also dependent on its strong
oxidising power.
8. Structure of Permanganate &Manganate:

MULTIPLE CHOICE QUESTIONS (1 MARK)


1. Which of the following is a strong oxidising agent? (At. No. Mn =25 Zn=30, Cr=24 Sc=21)
(a) Mn3+ (b) Zn2+ (c) Cr3+ (d) Sc 3+
2. In which of the following pairs both the ions are coloured in aqueoussolution?
(a) Sc3+ Ti3+ (b) Sc3+Co2+ (c) Ni2+ Cu+ (d) Ni2+Ti3+
3. Which of the following has the same ionic size?
(a)Zr4+, Hf4+ (b)Zn2+, Hf4+ (c) Fe2+, Ni2+ (d)Zr4+ , Ti4+
4. Magnetic moment of 2.83 B.M. is given by which of the following ion?
(a)Ti3+ (b) Ni2+ (c) Cr3+ (d)Mn2+
5. The correct order of decreasing second ionising enthalpy of Ti(22) V(23) Cr(24) Mn(25)
(a) V > Mn < Cr>Ti (b) Mn <Cr <Ti<V (c) Ti> V> Cr > Mn (d) Cr> Mn > V >Ti
6. Which of the following ion has maximum number of unpaired electrons?
(a)Fe3+ (b) V3+ (c)Ti3+ (d) Sc3+
7. Which of the following is most stable in aqueous solution?
(a) Mn2+ (b) Cr3+ (c) V3+ (d) Ti3+
8. KMnO4 is not acidified by HCl instead of H2SO4 because
(a) H2SO4 is stronger acid than HCl (b) HCl is oxidised to Cl2 by KMnO4
(c) H2SO4 is dibasic (d) rate is faster in presence of H2SO4
9. The oxidation state of Cr in final product formed by reaction of KI and acidified dichromate
solution is-
(a) +4 (b) +6 (c) +2 (d) +3
10. Which of the following lanthanoids show +2 oxidation state besides the characteristic oxidation
state +3 of lanthanoids?
(a) Ce (b) Eu (c) Tb (d) Ho
11. The difference between the oxidation number of Cr in chromate and dichromate ion is
(a) 1 (b) 2 (c) 3 (d) 0
12. Which of the following are d-block elements but not regarded as transition elements?
(a) Cu, Ag, Au (b) Zn, Cd, Hg (c) Fe, Co, Ni (d) Ru, Rh, Pd
13. Transition elements form alloys easily because they have
(a) Same atomic number (b) Same electronic configuration
(c) Nearly same atomic size (d) None of the above
14. Which of the following has magnetic moment value of 5.9?
(a) Fe2+ (b) Fe3+ (c) Ni2+ (d) Cu2+
15. The properties which is not characteristic of transition elements-
(a) Variable oxidation state (b) Tendency to form complexes
(c) Formation of colour compounds (d) natural radioactivity.
34 | Page
ANSWERS :
1 (a) 2 (d) 3 (a) 4 (b) 5 (d) 6 (a) 7 (b) 8 (b) 9 (d) 10 (b) 11(d) 12 (b) 13 (c) 14 (b) 15(d)
ASSERTION REASON TYPE QUESTIONS (1MARK)
In the Following questions a statement of Assertion(A) is followed by a statement of Reason(R). Select
the most appropriate answer from the options given below:
a. Both A and R are true and R is the correct explanation of A
b. Both A and R are true but R is not the correct explanation of A.
c. A is true but R is false.
d. A is false but R is true.
1. Assertion: Cr (VI) in the form of dichromate in acidic medium is a strong oxidizing agent.
Reason: Cr (III) is more stable in aqueous medium.
2. Assertion: Ce (IV) is stable.
Reason: All the Lanthanoides cannot exhibit +4 oxidation states.
3. Assertion: Zn, Cd, Hg and Cn are not regarded as transition metals.
Reason: These elements do not belong to d-block of periodic table.
4. Assertion: In the series Sc to Zn the enthalpy of atomisation of Zinc is the lowest.
Reason: Zinc has allpaired electrons, so metal-metal bond is weaker.
5 Assertion: Magnetic moment of Mn2+ is more than that of Cr2+.
Reason: Higher the atomic number smaller is the magnetic moment.
ANSWERS 1.(a) 2(c) 3(c) 4(a) 5.(c)

SHORT ANSWER TYPE QUESTIONS (2 MARKS)


Q1. Why transition elements exhibit variable oxidation states?
Ans. Transition elements exhibit variable oxidation state due to less energy gap between ns and (n-1)
d orbitals. With same energy (n-1) d electrons also get ejected with ns electrons.
2. Explain the followings:
(a) Why transition elements act as a catalyst?
(b) Why transition elements form alloys?
Ans (a) Transition element act as a catalyst due to variable oxidation state.
(b)Transition elements forms alloys due to similar radii.
3 Calculate the magnetic moment of a divalent ion in aqueous solution if its at.no is 25.
Zn2+ salts are white while Cu2+ salts are coloured. Why?
[ Ans: μ = 5(5  2) = 5.92BM]
Cu2+ (3d9 4s0) has one unpaired electron in d-subshell absorbs radiation in visible region
resulting in d-d transition and hence Cu2+ salts are coloured. Zn2+ (3d10 4s0) has completely filled
d-orbitals. No radiation is absorbed for d-d transition and hence Zn2+ salts areColourless.
4 Which is a stronger reducing agent Cr2+or Fe2+and why?
[ Ans: because change in 3d4→ 3d3 occurs in case of Cr2+ to Cr3+ But 3d6→ 3d5 occurs in case of Fe2+
to Fe3+. In a medium (like water) d3is more stable as compared to d5]
5. In the following ions: Mn3+, V3+, Cr3+, Ti4+ (Atomic no.: Mn=25, V=23, Cr=24, Ti=22)
A. Which ion is most stable in an aqueous solution?
B. Which ion is strongest oxidizing agent?
C. Which ion is colorless?
D. Which ion has the highest number of unpaired electrons?
Ans. A. Cr3+ because of half filled t2g level.
B. Mn3+, as the change from Mn3+ to Mn2+ results in stable half filled (d5) configuration.
C. Ti4+, as Ti4+ has empty d-orbitals therefore d-d transition cannot occur in Ti4+.
D. Mn3+ (3d4 4s0). It has 4 unpaired electrons.
6. The elements of 3d transition series are given as: Sc, Ti, V, Cr, Mn, Fe, Co, Ni, Cu, Zn
Answer the following:
A. Copper has exceptionally positive EoM²/M value. Why?
B. Which element is a strong reducing agent in +2 oxidation state and why?
C. Zn2+ salts are colourless. Why?
35 | Page
D. Which element has the highest melting point
Ans A. Because the sum of sublimation enthalpy and hydration enthalpy to convert Cu(s) to Cu2+(aq)
is so high that it is not balanced by its hydration enthalpy.
B. Cr is strongest reducing agent in +2 oxidation state. Cr2+ has configuration 3d4. After losing one
electron it forms Cr3+ which has stable half filled t2g, level.
C. Zn2+ (3d10) has completely filled d-orbitals. As a result of this, d-d transition cannot occur and hence
Zn2+ salts are colourless.
D. Mn
7 What is Lanthanoid contraction? Give its cause. What are its Consequences?
Ans: Lanthoanoidcontraction: The regular decrease in the atomic and ionic radii of lanthanoids with
increasing atomic number is known as Lanthanoid contraction.
Consequences of Lanthanoid contraction:
(i) Diffcult to separate the lanthanoids because the change in ionic radii is very small, their
chemical properties are similar. Hence, separations of lanthanoids are difficult. (ii) Similarity in size
of elements belonging to same group of second and third transition series due to lanthanoid
contraction the size of Zr (160 pm) is same as that of Hf(159 pm).]
8. When pyrolusite ore MnO2 is fused of with KOH in presence of air,a green coloured compound (A) is
obtained which undergoes disproportionation reaction in acidic medium to give purple coloured
compound (B).
a) Write the formulae of (A) & (B).
b) What happens when compound (B) is heated.
[Ans: (a) A= K2MnO4 B=KMnO4 (b) 2KMnO4   K2MnO4 + MnO2 + O2]
9. When chromite ore FeCr2O4 is fused with NaOH in presence of air, a yellow coloured compound
(A) obtained which on acidification with dilute sulphuric acid gives a compound (B)Compound
(B) on reaction with KCl forms an orange coloured crystalline compound (C).
a) Write the formulae of (A), (B) & (C).
b) Write one use of compound (C).
Ans: (a) A = Na2CrO4 B= Na2Cr2O7 C= K2Cr2O7 (b) Potassium dichromate is used as a primary
standard in volumetric analysis.]
10 How would you account for the following:-
The oxidising power of the following three oxo ions in the series follows the order VO2+<Cr2O72-<MnO4-
Ans: This is due to the increasing stability of the lower species to which they are reduced
SHORT ANSWER TYPE QUESTIONS (3 MARKS)
1. Account for the following:
(a) Mn2O7 is acidic whereas MnO is basic.
(b) Though copper has completely filled d-orbital (d10) yet it is considered as a transition metal.
(c) Actinoids show wide range of oxidation states.
Ans. (a) Mn has + 7 oxidation state in Mn2O7 and +2 in MnO. In higher oxidation state of the metal,
valence electrons involved in bonding and are not available. Instead effective nuclear charge is high
and hence it accept electrons and behave as aLewisacid.
(b) Copper exhibits +2 oxidation state wherein it will have incompletely filled d-orbitals (3d9), hence
transition metal.
(c) This is due to comparable energies of 5f, 6d and 7s-orbitals.
2. Explain giving a suitable reason for each of the following -
a. Metallic bonding is more frequent for the 4d & 5d series of transition metals than 3d series.
b. Cu+ salts are colourless while Cu2+ salts are Coloured.
c. Mn2+ exhibits maximum paramagnetism.
Ans: (a) [4d and 5d transition elements (2nd and 3rd series) are Larger in size than the
corresponding 3d elements. Hence the valence electrons are less tightly held and form M-M bond
more frequently (b) Due to the absence of unpaired electrons in d-orbitals Cu+ salts are colourless
while due to presence of unpaired electrons in d-orbitals Cu2+ salts are Coloured (c) Because Mn2+
contain 5 unpaired electrons in their 3d sub shells]
3. Account for the following

36 | Page
a. Although Zr belongs to 4d and Hf belongs to 5d transition series but it is quite difficult to
separate them.
b. There is in general increase in density of element from titanium to copper
c. Most of the transition metals and their compounds possess catalytic properties.
Ans: (a) [ because of lanthanoid contraction (b) The decrease in metallic radius coupled with increase
in atomic mass results in a general increase in the density (c) The catalytic activity of transition metal
ions is due to following two reasons: Variable oxidation states & Large surface area]
4. Explain the following observations.
(a) Mn Shows the highest oxidation state of +7 among 3d series elements.
(b) Mn Shows the highest oxidation state of +7 with oxygen but with fluorine it shows the highest
oxidation state of +4.
(c) MnO is basic while Mn2O7 is acidic in nature.
Ans: (a) Total 7 electrons are present in 3d and 4s in Mn and hence it can exhibit maximum
oxidation state of +7
(b) Because of ability of oxygen to form multiple bonds using 2p orbitals of oxygen and 3d orbitals
of Mn.
(c) Mn in Mn2O7 is in +7 oxidation state and Mn maximum oxidation state is +7. Hence it is Acidic. If
an element in a compound is in lower oxidation state then it will show basic character. Mn is in+2
oxidation state in MnO hence basic.]
5. Account for the following
(a) Cu+ ion is not stable in aqueous solutions
(b) d1 configuration is very unstable in ions.
(c) E0 value for the Mn2+/Mn much more than expected.
Ans: (a) It disproportionate to give Cu2+ and Cu. (b) The ions in d1 configuration tend to lose one
more electrons to get into stable d0 configuration (c) due to greater stability of half-filled d-subshell
(d5) in Mn.]
6. Assign reasons for the following:
(a) Scandium is a transition element but Zinc is not.
(b) Silver atom has completely filled d orbital (4d10) in its ground state, yet it is transition element.
(c) In the series Sc(Z = 21) to Zn (Z = 30), the enthalpy of atomisation of zinc is the lowest
Ans: (a) Scandium is a transition element because it has incompletely filled d orbitals but Zinc has
completely filled d orbitals i.e 3d10 configuration (b) Silver (Z = 47) can exhibit +2 oxidation state
wherein it will have incompletely filled d-orbitals (4d), hence a transition element (c) because in
the formation of metallic bonds, no electrons from 3d-orbitals are involved in case of zinc, while in
all other metals of the 3d series, electrons from the d-orbitals are always involved in the formation
of metallic bonds]
LONG ANSWER TYPE QUESTIONS (5 MARKS)
1. Give Reasons for the following
i. Ce4+ in aqueous solution is a good oxidising agent.
ii. Actinoids contraction is greater from element to element than lanthanoid contraction.
iii. The actinoids exhibit a larger number of oxidation states than the corresponding lanthanoids
iv. La3+ and Lu3+do not show any colour in solutions.
v. Chemistry of all the lanthanoids are quite similar.
[Ans: (i) because it can readily change to the most stable +3 oxidation state by gaining one electron
(ii) due to poor shielding by 5f-electrons in actinoids than that by 4f-electrons in the lanthanoids
(iii) because of the very small energy gap between 5f, 6d and 7s subshells. (iv) due to absence of
unpaired electron in f orbital (v) The change in the size of the lanthanoids due to lanthanoid
contraction is very small]
2. A Complete and balance the following reactions
a) Cr2O72–+ Fe2++ H+→
b) MnO4– + H++ C2O42- →
c) Cr2O72–+OH- →
d) MnO4– + H2O + I-→

37 | Page
B. Name the oxo metal anion of one of the transition metals in which the metal exhibits the
oxidation state equal to the group number.
Ans (a) Cr2O72–+ 6Fe2++14 H+→2Cr+3+ 6Fe3+ + 7H2O
(b) 2MnO4–+ 16 H++5C2O42- → 2Mn2+ + 8H2O +10CO2
(c) Cr2O72–+2 OH-→ 2CrO42- + H2O
(d) 2 MnO4–+ H2O + I– → 2MnO2 + 2OH- + IO3–
B. In MnO4- ion, the oxidation state of Mn is + 7. It is equal to its group number 7.
In CrO42-ion, the oxidation state of Cr is + 6. It is equal to its group number 6.
CASE BASED QUESTIONS
The Following questions are case based questions carries 4 (1+1+2) marks. Read the given
passage carefully and answer the questions that follow:
Transition elements are elements that have partially filled d-orbitals. The configuration of these
elements corresponds to (n - 1) d1-10 ns1-2. It is important to note that the elements mercury, cadmium
and zinc (are not considered transition elements because of their electronic configurations, which
corresponds to (n - 1) d1-10 ns2.
Some general properties of transition elements are :
These elements can form colored compounds and ions due to d-d transition;
These elements-.exhibit many oxidation states;
A large variety of legends can bind themselves to these elements, due to this, a wide variety of stable
complexes formed by these ions. The boiling and melting point of these elements are high. These
elements have a large ratio of charge to the radius.
a. Give reason Zn is soft whereas Cr is hard.
b. Explain why transition metals show higher oxidation state with oxygen as compare to fluorine.
c. Why Zn, Cd and Hg have low melting and boiling point?
OR
c. The enthalpies of atomization of transition elements are high. Give reason
Ans a. Cr (3d54s1) has five unpaired electrons in its d-orbitals whereas Zn (3d104s2) has no unpaired
electrons in its d-orbitals. As a result of this weak metallic bonds exist in Zn whereas strong metallic
bonds exist in Cr. Hence, Zn is soft whereas Cr is hard.
b. Ability to stabilize higher oxidation states by oxygen is due to form multiple bonds with metals
c. All the electrons in d-subshell are paired. Hence, the metallic bonds present in them are weak
OR
c. Because of large number of unpaired electrons in their atoms, transition elements have stronger
interatomic interaction and hence stronger bonding between atoms resulting in higher enthalpies of
atomization
COORDINATION COMPOUNDS
QUICK REVISION POINTS
 Coordination Compounds – A coordination compound contains a central metal atom or ion
surrounded by number of oppositely charged ions or neutral molecules called ligands. There is
a coordinate bond between metal atom and these ions or molecules, e.g., [Cu(NH3)4]2+.
 Double Salt: When two or more salts are added to form a stable solid together and break into
constituentions when dissolved in water or any solvent e.g. FeSO4(NH4)2SO4.6H2O(Mohr’s salt).
 Werner’s Theory of Co-ordination compounds:
(i) Metal ions possess two types of valency (a) primary or ionisable valency and (b)
Secondary ornon- ionisable valency.
(ii) Every metal ion has a fixed number of secondary valencies and this is known as
coordination number.
(iii) Primary valencies are satisfied by anions while secondary valencies are satisfied by
negative group or neutral molecules with lone pair of electrons.
(ii) Secondary valencies are directed in space towards internal positions and provide a
definite geometry to the complex.
Type Of Ligands
(a) Mono dentate Ligand: only one donor atom is present. eg. Cl-,Br-

38 | Page
(b) Didentate: When ligand can bind through two donor atoms as in H 2NCH2CH2NH2 (ethane-
1,2-diamine) (en) or C2O42– (oxalate anion)
(c) Polydentate: When several donor atoms are present in a single ligand is said to be
polydentate.Ethylenediaminetetraacetateanion EDTA4– is an important hexadentate ligand.
(d) Chelate ligand: When a di- or polydentate ligand uses its two or more donor atoms to
bind single metal ion, it is said to be a chelateligand.
(e) Ambidentate ligand: Ligand which can ligate through two different atoms is called
ambidentate ligand. Examples of such ligands are the NO2– and SCN– ions. NO2– ion can
coordinate either through nitrogen or through oxygen to a central metal atom or ion. Similarly,
SCN– ion can coordinate through the sulphur or nitrogen atom.
 Coordination number: The coordination number of a metal ion in a complex can be
defined as the number of ligand donor atoms to which the metal is directly bonded. For
example, in the complex ions, [PtCl6]2–and [Ni(NH3)4]2+,the coordination number of Pt
and Ni are 6 and 4.
 Homoleptic complexes: Those complexes in which metal or ion is coordinate bonded
to only one kind of donor atoms. For example: [Co(NH3)6]3+
 Heteroleptic complexes: Those complexes in which metal or ion is coordinate bonded to
more than one kind of donor atoms. For example: [CoCl2(NH3)4]+, [Co(NH3)5Br]2+
 Naming of Mononuclear Coordination Compounds
The following rules are followed while naming coordination compounds.
 The cation is named first in both positively and negatively charged coordinationentities.The
ligands are named in alphabetical order before the name of the central atom/ion.
 Names of the anionic ligands end in −o and those of neutral and cation ligands are thesame.
[Exceptions: aqua (H2O), ammine (NH3), carbonyl (CO), nitrosyl (NO)]
 To indicate the number of the individual ligands, the prefixes mono−, di−, tri−, etc., are used. If
these prefixes are present in the names of ligands, then the terms −bis, −tris, −tetrakis, etc., are
used. For example, [NiCl2(PPh3)2] is named as dichlorobis(triphenylphosphine)nickel(II).
 Oxidation state of the metal is indicated by a Roman numeral in parentheses.
 If the complex ion is cation, then the metal is named as the element. For example, Fe in a
complexcation is called iron and Pt is called platinum.
 If the complex ion is anion, then the metal is named with ‘−ate’ ending. For example, Co in a
complex anion, [Co(SCN)4]2− is called cobaltate.
 The neutral complex molecule is named same as the complex cation.
 IUPAC names of some coordination compounds compounds
1 K2[Zn(OH)4] potassium tetrahydroxozincate(II)
2 K3[Al(C2O4)3] potassium trioxalatoaluminate(III)
3 K3[Cr(C2O4)3] potassium trioxalatochromate(III)
4 Hg[Co(SCN)4] mercury tetrathiocyanatocobaltate(III)
5 K2[Ni(CN)4] potassium tetracyanonickelate(II)
6 [Pt(NH3)BrCl(NO2)]– amminebromidochloridonitrito-N-platinate(II)
7 Fe4[Fe(CN)6]3 iron(III) hexacyanoferrate(II)
8 [Co(NH3)4(H2O)Cl]Cl2 tetraamineaquachloridocobalt(III) chloride
9 [CoCl2(en)2] + dichloridobis(ethane-1,2-diamine)cobalt(III) ion
10 [Ni(CO)4] tetracarbonylnickel(0)
11 [Cr(en)3]Cl3 tris(ethane–1,2–diamine) chromium(III) chloride
12 [PtCl2(en)2](NO3)2 dichloridobis(ethane–1,2–diamine)platinum(IV) nitrate
 Isomerism in complexes: -Compounds that have the same molecular formula but different
structural formula or spatial arrangement of atoms arecalled isomer sand the phenomenon is
called isomerism.
 Structural Isomerism: -These are isomers which differ in the structural arrangement of ligands
around the central atom.They are of four types:

39 | Page
 Ionisation Isomerism: It arises due to the interchange of ions between the inside and outside of
co-ordination sphere. They give different types of ions in aqueous solution. An example is
[Co(NH3)5SO4]Br and [Co(NH3)5Br]SO4.
1) Linkage isomerism: It arises in a coordination compound containing ambidentate ligand. Eg.
Complexes containing thiocyanate ligand, SCN–, may bind either through nitrogen to give M–NCS or
through sulphur to give M–SCN.
2) Co-ordination Isomerism: If both anionic and cationic parts are complexes, the isomerism
arises due to the interchange of ligandsbetween cationic and anionic entities. Eg.
[Co(NH3)6][Cr(CN)6] and [Cr(NH3)6][Co(CN)6]
3) Solvate isomerism: This form of isomerism is also known as ‘hydrate isomerism’ if water is
involved as the solvent. Solvate isomers differ in the no.of solvent molecule which is directly
bonded to the metal ion as ligand. An example is [Cr (H2O)6]Cl3 (violet) and [Cr(H2O)5Cl]Cl2.H2O
(grey-green).
Stereoisomerism: - These are isomers which differ only in the spatial arrangement of ligands
around the central atom.They have same atom to atom bond. These are of two types:
(i) Geometrical isomerism (ii) Optical isomerism
Geometrical Isomerism: This type of isomerism is shown by heteroleptic complexes. It arises due to
the different possible geometric arrangements of the ligands around the central atom. It is mainly
found in co-ordination complexes with co-ordination numbers 4 (square planar complexes) and 6
(octahedral complexes). Geometrical isomerism in which the same ligands are on the same side of
the central metal atom is called cis-isomer and the isomer in which the same ligands are on the
opposite side is called trans-isomer. Square planar complexes with formula [MX2L2] (X and L are
unidentate ligands) can show this isomerism. eg.:[Pt (NH3)2Cl2]

Square planar complexes of the type MABXL (where A,B,X,L are unidentate ligands) show three
geometrical isomers-two cis and one trans. Octahedral complexes with formula [MX2L4] can also show
this type of isomerism. Here the two ligands Xmay be oriented cis or trans to each other. eg. [ Co
(NH3)4Cl2]

This type of isomerism also arises when bidentate ligands (L–L) are present in complexes with
formula [MX2(L– L)2] e.g.: [Co(en)2Cl2]+
Optical isomerism: -Optical isomers are mirror images that cannot be superimposed on one
another. These are also called enantiomers. The molecules or ions that cannot be superimposed are
called chiral. There are two forms of optical isomers- dextro (d) and laevo(l) depending upon the
direction they rotate the plane of polarized light in a polarimeter (d-rotates to the right, l-rotates to
the left). Optical isomerism is common in octahedral complexes involving bidentate ligands. In aco-
ordination entityof the type [PtCl2(en)2] +,only the cis-isomer shows optical activity. The trans-isomer has
a plane of symmetry and is optically inactive. Another example is [Co(en)3]3+

Fac-mer isomerism: It is a type of geometrical isomerism occurs in octahedral co-ordination


entities of the type [Ma3b3]. If similar ligands occupy three adjacent positions of an octahedral face, it is

40 | Page
called facial (fac) isomer. When the positions are around the meridian of the octahedron, it is called
meridional (mer) isomer. eg.[Co(NH3)3(NO2)3].
BONDING IN COMPLEX COMPOUNDS
 Valence Bond Theory (VBT): According to this theory, the metal atom or ion under the
influence of ligands can use its (n- 1)d, ns, np or ns, np, nd orbitals for hybridization to yield a
set of equivalent orbitals of definite geometry.
Coordination Type of Distribution of hybrid orbitals in Examples
number hybridization space(geometry)

4 sp3 Tetrahedral [NiCl4]2-,[CoF4]2-


4 dsp2 Square planar [Ni(CN)4]2-
5 dsp3 Trigonal bipyramidal [Fe(CO)5]
6 sp3d2 Octahedral [outer orbital complex or [CoF6]3-
high spincomplex] ,[Fe(H2O)6]3+
6 d2sp3 Octahedral [inner orbital or low spin or [Cr(NH3)6]3+
spin paired complex] [Co(NH3)6]3+
 The inner orbital (low spin) or the outer orbital (high spin) complexes are formed depending
upon whether the d–orbitals of inner shell or d–orbitals of outer shell are used
inhybridization. The complex will be diamagnetic if all electrons are paired. If unpaired
electrons are present then the complex willbe paramagnetic.
 In Presence of Strong ligands like CO, CN- C2O42- NH3, en electrons of central atom/ions gets
paired. In Presence of Weak ligands like F-, Cl-, I-, Br- H2O electrons of central atom/ions
remain unpaired.
CRYSTAL FIELD THEORY (CFT)
Crystal field splitting in octahedral coordination: In an octahedral co-ordination entity
there are six ligands surrounding the metal atom/ion and they are along the axis of the
octahedron. So,ther epulsion between the electrons in metal d orbitals and the electrons
(or negative charges) of the ligands is greater for the dx2-y2 and dz2 orbitals, which are
pointing towards the axes, than the dxy, dyz and dxz orbitals, which are directed between
the axes. So, the energies of the dx2-y2 and dz2 orbitals (called eg orbitals) will be raised and
that of the dxy, dyz and dxz orbitals (called t2g orbitals) will be lowered. Thus, the
degeneracy of the d orbitals has been removed. This splitting of the degenerate levels due
to the presence of ligands in a definite geometry is termed as crystal field splitting.
 This splitting of the degenerate levels due to the presence of ligands in a definite
geometry is termed as crystal field splitting. The energy of the two eg orbitals will
increase by (3/5) Δo and that of the three t2g will decrease by (2/5) Δo.
 The crystal field splitting, Δo, depends upon the field produced by the ligands and charge
on the metal ion. Some ligands are able to produce strong fields in which case, the
splitting will be large whereas others produce weak fields and consequently result in
small splitting of dorbitals.

41 | Page
 Spectrochemical series: In general, ligands can be arranged in a series in the order of
increasing field strength as given below:I–<Br–<SCN–<Cl–<S2–<F–<OH–<C2O42–<H2O<NCS–
<edta4–<NH3<en<CN–<CO Such aseries is termed as Spectrochemical series.
 Assigning electrons in the d -orbitals of metal ion in octahedral coordination entities: For d4
ions, two possible patterns of electron distributionarise:
(i) If Δo< P, the fourth electron enters in one of the eg orbitals giving the configuration t 2g3 eg1.
Ligands for which Δo< P are known as weak field ligands and form high spincomplexes.
(ii) If Δo> P, it becomes more energetically favourable for the 4th electron to occupy a t2g orbital
with configuration t2g4 eg0. Ligands which produce this effect are known as strong field ligands
and form low spin complexes
(b) CRYSTAL FIELD SPLITTING IN TETRAHEDRAL COMPLEXES COMPOUNDS:

BONDING IN METAL CARBONYLS:-The metalcarbon bond in carbonyl compounds possesses


both sigma and pi characters. TheM–Cς bond is formed by the donation of lone pair of
electrons on the carbonyl carbon into a vacant orbital of the metal. The M–C π bond is formed by
the donation of a pair of electrons from a filled d orbital of metal into the vacant antibonding π*
orbital of carbon monoxide. The metal to ligand bonding creates a synergic effect which
strengthens the bond between CO and themetal eg. [Ni(CO)4]
 STABILITY OF COMPLEX:
For a reaction of the type:
M + 4L → ML4, β = [ML4]/[M][L]4
As the value of the stability constant increases, the stabilityofthecomplexalso increases.
Theabovereactioncanbeconsideredtotakeplacein4steps:
M + L → ML, K1 = [ML]/[M][L]. ML+L→ML2,K2=[ML2]/[ML][L]
ML2+L→ML3 ,K3=[ML3]/[ML2] [L] ML3+L→ML4,K4=[ML4]/[ML3] [L]
Where K1, K2, K3 and K4 are referred to as stepwise stability constants. The overall stability
constant, β = K1 × K2 × K3 × K4 and1/β is called instability constant or dissociation constant.
 Factors affecting the stability of Complexes:
Nature of the central ion: Greater the charge density (charge/radius ratio) on the central ion,
greater is the stability of the complex. For example, complexes of Fe3+ are more stable than Fe2+.
Chelate effect: The presence of chelate rings in the complex increases its stability increases.
When chelation occurs entropy increases entropy increases therefore the formation of the
complex becomes more favourable.
chelate [Cd(en)2]2+is 10000 times stable than [Cd(CH3-NH2)4]2+.
Application of complex compounds:
(i) They are used in photography, i.e., AgBr forms soluble complex with sodium thiosulphate in
photography.
(ii) K[Ag(CN)2] is used for electroplating of silver, K[Au(CN)2] is used for gold plating.
(iii) Some of ligands oxidise Co2+ to Co3+ ion.
(iv) EDTA is used for estimation of Ca2+ and Mg2+ in hard water.
(v) Silver and gold are extracted by treating Zn with their cyanide complexes.
(vi)Ni2+ is tested and estimated by DMG (dimethylglyoxime).
(vii) Cis-platin [Pt(NH3)2Cl2] is used as antitumor agent in the treatment of cancer.
(viii) EDTA is used to remove Pb by forming Pb-EDTA complex which is eliminated in urine.
(ix) Haemoglobin contains Fe, chlorophyll contains (Mg) and Vitamin B12 contain Co2+.

42 | Page
(x) Bauxite is purified by forming complex with NaOH.
(xi) Coordination compounds are used as catalysts for many industrial processes.
MULTIPLE CHOICE QUESTIONS (1 MARK)
Q1. When 0.1 mol CoCl3(NH3)5 is treated with excess of AgNO3, 0.2 mol of AgCl
are obtained. The conductivity of solution will correspond to
(a) 1:3 electrolyte (b) 1:2 electrolyte
(c) 1:1 electrolyte (d) 3:1 electrolyte
Q2. What is the oxidation no. and coordination no. of Fe in [Fe (CN)6]4-?
(a) + 2 and 3 (b) +3 and 3 (c) + 2 and 6 (d) + 6 and 6
Q3. Which of the following is a pair of ambidentate ligands?
(a) CN-, Ox2- (b)CN-, SCN- (c)CO, SCN- (d) CO, Ox2-
Q4. Which of the following ligand has lowest ∆0 value?
(a) CN- (b)CO (c)F- (d) NH3
Q5. Which of the following statement is not correct?
(a) [Ni(CN)4]2- involves dsp2 hybridisation (c) [Fe(H2O)6]3+ is inner orbital complex
(b) [NiCl4]2- is tetrahedral (d) [Fe(CN)6]4- is diamagnetic
Q6. Which of the following complexes show linkage isomerism?
(a) [Co(NH3)5 (NO3)]2+ (b) [Co(H2O)5CO]3+ (c) [Cr(NH3)5 SCN]2+ (d) [Fe(en)2Cl2]+
Q7. Type of ligand and coordination no. of metal in complex [Fe(Ox)3]3-.
(a) monodentate and 3 (b) bidentate and 3
(c) monodentate and 6 (d) bidentate and 6
Q8. The formula of the complex tris(ethane-1,2-diamine)cobalt(III) sulphate is
(a) [Co(en)3]SO4 (b) [Co(en)3 SO4]
(c) [Co(en)3]2(SO4)3 (d) [Co(en)3](SO4)3
Q9. IUPAC name of complex K2[PdCl4] is
(a) potassium tetracyanopalladate (II) (b) potassium tetrachloridopalladium (II)
(c) potassium tetrachloridopalladate (II) (d) potassium tetrachloridopalladate (III)
Q10. On the basis of CFT what will be the electronic configuration of d5 in terms of t2g and eg
orbitals when ∆0 > P.
(a) t2g4 eg1 (b) t2g5 eg0 (c) t2g0 eg5 (d) t2g3eg2
Q.11 In the coordination entity [Co(NH3)3(NO2)3], if all three N atoms of the amine ligands
occupy adjacent positions at the corners of an octahedral face, the geometrical isomer
formed is known as _______ isomer.
(a) cis (b) trans (c) fac (d) mer
Q.12 Which of the following compounds has a meridional isomer?
(a) [Fe(NO)5Br]+ (b) [Al(CO)3(NO2)3]
(c) [K(NH)3)4(NO)2] + (d) [Fe(H2O)2(CO)2(NO)2]3+
Q.13 Which of the following do not show geometrical isomerism? (Assume all ligands are
unidentate)
(a) Square planar [MXL3] (b) Square planar [MX2L2]
(c) Octahedral [MX2L4] (d) Octahedral [MX3L3]
Q.14 The square planar complex [MABCD] is known to form three isomers, two cis and one
trans. Shown below are the two cis isomers of the complex. Identify the third trans isomer.

43 | Page
Q.15. Which reagent is used for detecting Ni2+ ions in solution?
(a) EDTA (b) Dimethylglyoxime
(c) α-nitroso-β-naphthol (d) Cupron
Q.16. Hardness of water is estimated by simple titration with which compound?
(a) Na2(EDTA) (b) Fe(EDTA)
(c) Mg(EDTA) (d) Co(EDTA)
Q.17 Which of the following biologically important coordination compounds has a
magnesium central atom?
(a) Chlorophyll (b) Haemoglobin
(c) Vitamin B12 (d) Carboxypeptidase-A
Q.18 The complex ion [Ag(S2O3)2] is associated with which field?
3-

(a) Electroplating (b) Medicine


(c) Water treatment (d) Photography
Answers
1. (b) 2. (c) 3. (b) 4. ( c) 5. ( c) 6. ( c) 7. ( d) 8. ( c) 9. (c) 10. (b) 11(c) 12.(b) 13(a)
14(a), 15(b), 16(a), 17(a) 18(d)
ASSERTION REASON TYPE QUESTIONS (1MARK)
( a) Assertion and reason both are correct statements and reason is correct explanation for
assertion.
(b) Assertion and reason both are correct statements but reason is not correct explanation for
assertion.
(c) Assertion is correct statement but reason is wrong statement.
(d) Assertion is wrong statement but reason is correct statement.
Q1. Assertion: Co (III) forms paramagnetic octahedral complexes with weak field ligands.
Reason: With weak field ligands ∆°< P and the electronic configuration of Co (III) will
be t2g4 eg2 having four unpaired electrons.
Q2. Assertion:1 mol of [CrCl2(H2O)4] Cl.2H2O will give 1 mole of AgCl with AgNO3 solution.
Reason: Cl- ions satisfying secondary valences will not be precipitated.
Q3. Assertion: All the ligands must contain one donor atom.
Reason: All inner orbital complexes are diamagnetic.
Q4. Assertion: NO2- is an ambidentate ligand.
Reason: An ambidentate ligand contains two attachment sites and at a time only one
site can be used for bonding.
Q5. Assertion: The complex [Co (NH3)3Cl3] does not give precipitate with AgNO3 solution.
Reason: The given complex is non-ionizable.
Answers – 1. (a) 2. (a) 3. (c) 4. (a) 5. (a)

SHORT ANSWER TYPE QUESTIONS (2 MARKS)


Q 1. Distinguish between Double salt and Coordination compound.
Double salt Coordination compound
They completely dissociate into simple They do not dissociate into simple
ions when dissolved in water. ions when dissolved in water
They give the tests for the constituent They does not give tests for the
ions. constituent ions.
Ex. FeSO4.(NH4)2 SO4.6H2O(Mohr’s salt) Ex. K4[Fe(CN)6]
Q.2 When a co-ordination compound CoCl3.5NH3 is mixed with AgNO3, 2 mole of AgCl is
precipated per mole of the compound. Write (a) structural formula of the complex.
(b) IUPAC name of the complex.
Ans. (a) [Co(NH3)5Cl]Cl2 (b) pentaamminechloridocobalt(III) chloride.
Q3. Explain Homoleptic and Heteroleptic complexes with example.
Ans.The complex in which metal atom is bound to only one type of ligand is called homoleptic
complex. Ex. [Ni(CN)4]2-
The complex in which metal atom is bound to more than one type of ligand is called heteroleptic
complex. eg. [Co(NH3)4Cl2]+
44 | Page
Q4. What do you meant byambidentate ligands& chelate ligands?
Ans (i) Ambidentate ligands: Ligands which can ligate through two different donorsites. eg SCN-
(ii) When di or polydentate ligands uses its two or more donor atoms to bind a single metal ion is
called chelate ligand.eg ethane-1,2-diamine.
Q5. Giving a suitable example for each, explain:
(i) Ionisation isomerism (ii) Coordination Isomerism
Ans: (i) Ionisation isomerism: This form of isomerism arises when the counter ion in a complex salt
is itself a potential ligand and can displace a ligand which can then become the counter ion. An
example is provided by the ionisation isomers [Co(NH3)5SO4]Br and [Co(NH3)5Br]SO4
(ii) Coordination Isomerism: This type of isomerism arises from the interchange of ligands
between cationic and anionic entities of different metal ions present in a complex. An
example is provided by [Co(NH3)6][Cr(CN)6], in which the NH3 ligands are bound to Co3+
and the CN– ligands to Cr3+. In its coordination isomer [Cr(NH3)6][Co(CN)6], the NH3
ligands arebound to Cr3+ and the CN– ligands to Co3+
Q 6 Explain the following:
(i) Low spin octahedral complexes of nickel are not known.
(ii) CO is a stronger ligand than NH3 for many metals.
Ans: (i) Ni in its atomic or in it’s ionic state cannot afford two vacant 3d orbitals hence d2sp3
hybridisation is not possible.
(ii) Because in case of CO back bonding takes place in which the central metal uses its filled d
orbital with empty anti bonding 𝜋* molecular orbital of CO.
Q7. Draw the possible geometrical isomers of [Pt(NH3)BrCl(Py)]
Ans:

Q.8 [NiCl4]2– is paramagnetic while [Ni(CO)4] is diamagnetic though both are tetrahedral. Why?
Ans. In [Ni(CO)4], Ni is in zero oxidation state whereas in [NiCl4]2– it is in +2 oxidation state. In
the presence of strong ligand, CO, the unpaired d-electrons of Ni pair up but Cl– being weak
ligand is unable to pair up the unpaired electrons
Q.9 For the complex [Co(NH3)6]3+, write the hybridization, magnetic character and spin type of
the complex.
Ans. Hybridization: d2sp3, magnetic character: Diamagnetic, spin: Low spin complex
Q10. Write name and formula of rhodium complex used for hydrogenation of alkene.
Ans. Wilkinson catalyst , [(Ph3P)3RhCl]
SHORT ANSWER TYPE QUESTIONS (3 MARKS)
Q.1 Write IUPAC names of the following coordination compounds:
(a) [Fe(CN)6]3– (b) K2 [PtCl4] (c) [Co(NH3)4Cl(-ONO)]Cl
Ans. (a) Hexacyanoferrate(III)ion (b) Potassium tetrachloridoplatinate(II)
(c) Tetraamminechloridonitrito-O-cobalt (III) chloride
Q.2Write the formulae for the following co-ordination compounds:
(a) Tetraammineaquabromidocobalt(III) chloride
(b) Potassium tetrahydroxonickate(II)
(c) Potassium trioxalatoaluminate(III)
Ans. (a) [Co(NH3)4(H2O)Br]Cl2 b) K2 [Ni(OH)4] c) K3 [Al(C2O4)3]
Q.3 (a) Give an example of ionization isomerism.
(b) Indicate the types of isomerisms exhibited by the complex[Co(NH3)5(NO2)] (NO3)2
(c) Which of the following is more stable complex and why?
[Co(NH3)6]3+ and [Co(en)3]3+
Ans. (a) [Co (NH3)5 (Br)] SO4 and[Co (NH3)5 (SO4)] Br
45 | Page
(b) It shows ionisation isomerism and linkage isomerism.
(c) [Co(en)3]3+ is more stable complex than [CO(NH3)6]3+ because of chelate effect.
Q.4 Write the name, structure and the magnetic behaviour of the following complex:
[Ni(CN)4]2-
Ans. Tetracynidonickelate(II)ion, square planar, diamagnetic
Q.5 (a) Why are low spin tetrahedral complexes not formed
(b) [Fe(H2O)6]3+ isstronglyparamagneticwhereas[Fe(CN)6]3-isweakly paramagnetic.
Explain. (At. no. Fe = 26)
Ans. (a) Low spin tetrahedral complexes are rarely observed because orbital splitting
energies for tetrahedral complexes are not sufficiently large for forcing pairing.
(b) In both the cases, Fe is in oxidation state +3. Outer electronic configuration of Fe3+is:

In the presence of CN–, the 3d electrons pair up leaving only one unpaired electron. The
hybridisation involved is d2sp3 forming inner orbital complex which is weakly
paramagnetic. In the presence of H2O (a weak ligand), 3d electrons do not pair up. The
hybridisation involved is sp3d2 forming an outer orbital complex. As it contains five unpaired
electrons so itis strongly paramagnetic. .
LONG ANSWER TYPE QUESTIONS (5 MARKS)
Q.1 Answer the followings: (2 + 2 + 1)
(a) The Spin magnetic moment of [MnBr4]2- is 5.9 BM. Predict the geometry of the complex ion?
(b) [Ni(CO)4] is tetrahedral but [Ni(CN)4]2- is square planar. Explain.
(c) Give one example of hexadentate ligand.
Ans: (a) Ans: Since coordination number of Mn2+ ion is 4 it will either sp3 with tetrahedral
shape or dsp2 with square planar shape but it is tetrahedral as d orbital will occupy by the
presence of 5 unpaired electrons.
(b) In [Ni(CO)4], Ni is in zero oxidation state whereas in [NiCN4]2– it is in +2 oxidation state. In
the presence of strong ligand, CO, the unpaired d-electrons of Ni pair up and it uses one S and
three p orbitals for sp3 hybridisation so it is tetrahedral and CN–is also strong ligand but the
oxidation state is +2 so it uses one d one s and two p orbitals for dsp2 hybridisation. So it is
square planar.
(c) EDTA4-
Q.2. (a)On the basis of CFT, write the electronic configuration for d5 if ∆o < P
For the complex [Fe(en)2Cl2]Cl, identify the following:
(i) Oxidation number of iron (ii) Hybridization in Fe
(iii) Shape of the complex (iv) Denticity of ligand ethylene-di-amine
Ans: (a) t2g eg
3 2

(b) (i) +3, (ii) d2sp3 (iii) Octahedral (iv) 2


CASE BASED QUESTIONS
Read the passage given below and answer the following questions: (1+1+2=4)
Werner, a Swiss chemist prepared large no. of coordination compounds and studied their
physical and chemical properties. He proposed that, in coordination compounds, metal possess
two types of valences- primary and secondary valences. Primary valency is ionizable and
secondary valency is non-ionizable. In a series of complexes of Co(III) chloride with ammonia,
it was found that some of the chloride ions could be precipitated as AgCl on adding excess
amount of AgNO3 solution. The no. of ions furnished by a complex in a solution can be
determined by precipitation reactions. The measurement of molar conductance of solution of
complexes helps to estimate the no. of ions furnished by the complexes in solution.
(a)Write difference between primary and secondary valences.
(b) How many moles of AgCl are formed by the complex [Co(NH3)4Cl2]Cl with excess amount
of AgNO3 solution.
(c) Calculate number of primary and secondary valences shown in [Co(NH3)5Cl]Cl2.

46 | Page
OR
(c) Calculate number of primary and secondary valences shown in [Ni(H2O)6]Cl2.
Ans: (a). Primary valency is ionizable but secondary valency is not ionisable.
Primary valencies are satisfied only by anions no but secondary valenciesare satisfied by both
anions and neutral molecules
(b) One mole (c) 3 and 6 or 2 and 6

Haloalkanes and Haloarenes


QUICK REVISION POINTS:
The replacement of hydrogen atom(s) in a hydrocarbon, aliphatic or aromatic, by a halogen atom(s)
results in the formation of alkyl halide (haloalkane) and aryl halide (haloarene), respectively.
Classification: Haloalkanes and haloarenes may be classified as follows-
On the These may be classified as mono,
Basis of di, or polyhalogen (tri-, tetra-,
Number etc.) compounds depending on
of whether they contain one, two, or
Halogen more halogen atoms in their
Atoms structures.

Compounds (a) Alkyl halides or


Containing haloalkanes (R—X)
sp 3 C—X
Bond
(b) Allylic halides

(c) Benzylic halides

Compounds (a) Vinylic halides


Containing
sp2 C—X
Bond
(b) Aryl halides

Nature of C-X Bond: Since halogen atoms are more electronegative than
carbon, the carbon halogen bond of alkyl halide is polarised; the carbon atom
bears a partial positive charge whereas the halogen atom bears a partial
negative charge.

Nomenclature:
Common Name Alkyl halide

IUPAC Name Haloalkane

47 | Page
Methods of Preparation:1. From Alcohols
Thionyl chloride is preferred because the other two
products are escapable gases. Hence the reaction
gives pure alkyl halides.
The reactions of 1° and 2° alcohols with HX require
the presence of a catalyst, ZnCl2. With tertiary
alcohols, the reaction is conducted by simply shaking
with concentrated HCl at room temperature.
The order of reactivity of alcohols with a given
haloacid is 3°>2°>1°.

2. From Hydrocarbons
(a) By free radical halogenation

(b) By electrophilic substitution

(c) Sandmeyer’s reaction

(d) From alkenes


(i) Addition of hydrogen halides
(ii) Addition of halogens

3. Halogen Exchange:
Finkelstein reaction

Swarts reaction

48 | Page
Physical Properties:
a) Alkyl halides are colorless when pure. However, bromides and iodides develop color when
exposed to light. Many volatile halogen compounds have a sweet smell
b) Physical State: Lower members of alkyl halides (CH3F, CH3Cl, CH3Br and C2H5Cl) are colorless
gases at room temperature. The higher members up C18 are colorless liquids whereas other
members are colorless solids.
c) Solubility: Despite of polar nature of alkyl halides, they are insoluble in water due to the
inability to form hydrogen bonds. Still, they are soluble in non-polar solvents.
d) Melting and Boiling Point:
● Due to greater polarity as well as higher molecular mass as compared to the parent
hydrocarbon, the intermolecular forces of attraction (dipole-dipole and van der Waals) are
greater in the halogen derivatives. That is why the boiling points of chlorides, bromides, and
iodides are considerably higher than those of the hydrocarbons of comparable molecular mass.
● The boiling points of alkyl halides decrease in the order: RI> RBr> RCl> RF. This is because the
magnitude of van der Waal forces increases with the increase in size and mass of halogen atom.
The boiling points of isomeric haloalkanes
decrease with an increase in branching.

Boiling points of isomeric dihalobenzenes are


nearly the same. However, the para-isomers
are high melting as compared to their ortho
and meta-isomers. It is due to the symmetry
of para-isomers that fits in crystal lattice
better as compared to ortho- and meta-
isomers.
e) Density: The density increases with an increase in the number of carbon atoms,halogen atoms,
and atomic mass of the halogen atoms.
Chemical Reactions: The reactions of haloalkanes may be divided into the following
categories-
1. Nucleophilic substitution 2. Elimination reactions 3. Reaction with metals
1. Nucleophilic substitution reactions
a. CH3CH2Cl + KOH(aq) →
CH3CH2OH (Ethanol) + KCl
b. CH3CH2Br + NaOH(aq) → CH3CH2OH (Ethanol) +NaBr
c. CH3CH2I + NaOR → CH3CH2OR (Alkoxy ethane)+NaI
d. CH3CH2I + AgCN → CH3CH2NC (Alkyl Isocyanide) +Ag I
e. CH3CH2I + KCN → CH3CH2CN(Alkyl cyanide) +KI
f. CH3CH2Cl + AgNO2 → CH3CH2 NO2 (Nitroalkane) +AgCl
g. CH3CH2Cl + KNO2 → CH3CH2ONO(Alkylnitrite) +KCl
h. CH3CH2Br +NH3 → CH3CH2NH2 (Primary Amines) +HBr
i. CH3CH2Br +RNH2 → CH3CH2NHR(N-alkylethaneamine) +HBr
j. CH3CH2Br CH3CH2_H

★ Ambident nucleophiles: Groups like cyanides and nitrites possess two nucleophilic centers
and are called ambident nucleophiles. Cyanide group, linking through carbon atom resulting in
alkyl cyanides and through nitrogen atom leading to isocyanides. Similarly, nitrite ion linkage
through oxygen results in alkyl nitrites while through nitrogen atom, it leads to nitroalkanes.
★ Haloalkanes react with KCN to form alkyl cyanides as main product while AgCN forms
isocyanides as the main product. Since KCN is predominantly ionic and provides cyanide ions
in solution. However, AgCN is mainly covalent in nature and Nitrogen is free to donate electron
pairs forming isocyanide as the main product.

49 | Page
Mechanism: This reaction has been found to proceed by two different mechanisms-
(a) Substitution nucleophilic bimolecular (SN2): The reaction between CH3Cl and hydroxide ion to
yield methanol and chloride ion follows second order kinetics, i.e., the rate depends upon the
concentration of both the reactants. This process is called an inversion of configuration (Walden
inversion).

(b) Substitution nucleophilic unimolecular (SN1):


These reactions are generally carried out in polar
protic solvents (like water, alcohol, acetic acid,
etc.). The reaction between tert-butyl bromide
and hydroxide ion yields tert-butyl alcohol and
follows the first order kinetics, i.e., the rate of
reaction depends upon the concentration of only
one reactant, which is tert- butyl bromide.

● We can sum up the order of reactivity of alkyl halides towards SN1 and SN2 reactions as follows:

● For the same reasons, allylic and benzylic halides show high reactivity towards the S N1
reaction. The carbocation thus formed gets stabilized through resonance.

● For a given alkyl group, the reactivity of the halide, R-X, follows the same order in both the
mechanisms R–I> R–Br>R–Cl>>R–F.
● A SN2 reaction proceeds with complete stereochemical inversion while a SN1 reaction proceeds
with racemization.
● The stereoisomers related to each other as non-superimposable mirror images are called
enantiomers. Enantiomers possess identical physical properties namely, melting point, boiling
point, solubility, refractive index, etc. They only differ with respect to the rotation of plane
polarised light. If one of the enantiomers is dextro rotatory, the other will be laevo rotatory.
● A mixture containing two enantiomers in equal proportions will have zero optical rotation, as
the rotation due to one isomer will be canceled by the rotation due to the other isomer. Such a
mixture is known as a racemic mixture or racemic modification and the process is known as
racemization. A racemic mixture is represented by prefixing dl or (±) before the name.
● Retention of configuration is the preservation of the integrity of the spatial arrangement of
bonds to an asymmetric center during a chemical reaction or transformation.
● Inversion, retention, and racemization: There are three outcomes for a reaction at an
asymmetric carbon atom. Consider the replacement of a group X by Y in the following reaction;
50 | Page
-If (A) is the only compound obtained, the
process is called retention of
configuration.
-If (B) is the only compound obtained, the
process is called inversion of
configuration.
-If a 50:50 mixture of the above two is
obtained then the process is called
racemization.
2. Elimination reactions When a haloalkane with a β-hydrogen atom is heated with an alcoholic
solution of potassium hydroxide, there is the elimination of hydrogen atom from β-carbon and a
halogen atom from the α-carbon atom. As a result, an alkene is formed as a product. Since the β-
hydrogen atom is involved in elimination, it is often called β-elimination.

If there is the possibility of the formation of more than one alkene due to the availability of more than
one α-hydrogen atom, as per Saytzeff rule, “the preferred product is that alkene which has the greater
number of alkyl groups attached to the doubly bonded carbon atoms.” Thus, 2-bromopentane gives
pent-2-ene as the major product.

Elimination versus substitution: A chemical reaction is the result of competition; it is a race that is
won by the fastest runner. A collection of molecules tend to do, by and large, what is easiest for them.
An alkyl halide with α-hydrogen atoms when reacted with a base or a nucleophile has two competing
routes: substitution (SN1 and SN2) and elimination. Which route will be taken up depends upon the
nature of alkyl halide, strength and size of base/nucleophile, and reaction conditions. Thus, a bulkier
nucleophile will prefer to act as a base and abstract a proton rather than approach a tetravalent
carbon atom (steric reasons) and vice versa. Similarly, a primary alkyl halide will prefer a SN2
reaction, a secondary halide- SN2 or elimination depending upon the strength of base/nucleophile
and a tertiary halide- SN1 or elimination depending upon the stability of carbocation or the more
substituted alkene.

3. Reaction with metals:


Wurtz reaction

Reactions of Haloarenes:
1. Nucleophilic substitution Aryl halides are extremely less reactive towards nucleophilic
substitution reactions due to the following reasons:
(i) C—Cl bond acquires a partial double bond character due to resonance. As a result, the bond
cleavage in haloarene is more difficult than in haloalkane, and therefore, they are less reactive toward
nucleophilic substitution reaction.

(ii) In haloalkane, the carbon atom attached to halogen is sp3 hybridised while in the case of
haloarene, the carbon atom attached to halogen is sp2-hybridised.
(iii) In the case of haloarenes, the phenyl cation formed as a result of self-ionization will not be
stabilized by resonance and therefore, SN1 mechanism is ruled out.
(iv) Because of the possible repulsion, it is less likely for the electron-rich nucleophile to approach
electron-rich arenes.
51 | Page
Replacement by hydroxyl group
The presence of an electron-withdrawing group
(-NO2) at ortho- and para-positions increases the
reactivity of haloarenes.

2. Electrophilic
substitution
reactions

I. Halogenation

II. Nitration

III. Sulphonation

IV. Friedel-Crafts
reaction

3. Reaction with Wurtz-Fitting reaction


metals

Fitting reaction

Polyhalogen Compounds
Dichloromethane Used as a solvent as a paint remover, as a propellant in aerosols, and as a
(Methylene chloride) process solvent in the manufacture of drugs.

Trichloromethane The major use of chloroform today is in the production of the freon
(Chloroform) refrigerant R-22. Chloroform is slowly oxidized by air in the presence of
light to an extremely poisonous gas, carbonyl chloride, also known as
phosgene. It is therefore stored in closed dark-colored bottles completely
filled so that air is kept
out.

52 | Page
Triiodomethane It was used as an antiseptic but the antiseptic properties are due to the
(Iodoform) liberation of free iodine and not due to iodoform itself.

Tetrachloromethane Is used in the manufacture of refrigerants and propellants for aerosol cans.
(Carbon When carbon tetrachloride is released into the air, it rises into the
tetrachloride) atmosphere and depletes the ozone layer.

Freons The chlorofluorocarbon compounds of methane and ethane are collectively


known as freons. They are extremely stable, unreactive, non-toxic, non-
corrosive, and easily liquefiable gases. Freon 12 (CCl2F2) is one of the most
common freons in industrial use. It is manufactured from
tetrachloromethane by Swarts reaction. In the stratosphere, freon is able to
initiate radical chain reactions that can upset the natural ozone balance.

p,p’-Dichlorodiphenyl The DDT is effective against the mosquito that spreads malaria and lice that
trichloroethane(DDT) carry typhus.

4,4’- Dichlorodiphenyl
trichloroethane.

MULTIPLE CHOICE QUESTIONS (1 MARK)


1. Which of the following reaction will yield 2,2-Dibromopropane:
(a) CH3-C≡CH +2HBr → (b) CH3-CH=CH-Br +HBr →
(c) CH≡CH +2HBr → (d) CH3-CH=CH2 +HBr →
2. Fluorobenzene can be synthesized in laboratory
(a) by heating phenol with HF and KF (b) by heating the diazonium salt with HBF4
(c) by direct fluorination of benzene with F2 gas(d)by heating 1-Bromobenzene with NaF solution
3. Toluene reacts with chlorine in the presence of iron (III) chloride gives X and in presence of light
gives Y then X and Y are
(a) X= m-Chlorotoluene, Y= p-Chlorotoluene
(b) X= o and p-Chlorotoluene, Y= Trichloromethyl benzene
(c) X= Benzyl chloride, Y= m-Chlorotoluene
(d) X= Benzyl chloride, Y= o-Chlorotoluene
4. Chlorobenzene is formed by reaction of chlorine with benzene in the presence of AlCl3. Which of the
following species attacks the benzene ring in this reaction?
(a) Cl– (b) Cl+ (c) AlCl3 (d) [AlCl4]–
5. Toluene reacts with a halogen in the presence of iron (III) chloride giving ortho and para halo
compounds. The reaction is
(a) Electrophilic elimination reaction (b) Electrophilic substitution reaction
(c) Free radical addition reaction (d) Nucleophilic substitution reaction
6. Which of the following is not chiral?
(a) 2-Hydroxypropanoic acid (b) 2-Butanol
(c) 2, 3-Dibromobutane (d) 3-Bromopentane
7. Which of the following statement is not correct about the mechanism of this reaction in the
presence polar protic solvent?

(a) A carbocation will be formed as an intermediate in the reaction.


(b) OH– will attach the substrate (b) from one side and Cl– will leave it simultaneously from other
side.
(c) An unstable intermediate will be formed in which OH– and Cl– will be attached by weakbonds

53 | Page
(d) Reaction proceeds through SN2 mechanism
8. Reaction of C6H5CH2Br with aqueous sodium hydroxide follows:
(a) SN1 mechanism (b) SN2 mechanism
(c) Any of the above two depending upon the temperature of reaction (d) Saytzeff rule
9. Which one of the following compounds is more reactive towards SN1 reaction?
(a) CH2=CHCH2Br (b) C6H5CH2Br
(c) C6H5CH (C6H5) Br (d) C6H5CH(CH3) Br
10. Which of the following isomer has the highest melting point:
(a) 1,2-dicholorbenzene (b) 1,3 -dichlorobenzene
(c) 1,4-dicholorbenzene (d) all isomers have same melting points
11. The position of –Br in the compound in CH CH=CHC(Br)(CH ) can be classified as ____________.
3 3 2

(a) Allyl (b) Aryl (c) Vinyl (d) Secondary


12. Which of the following compounds will have highest melting point?
(a) Chlorobenzene (b) o-Dichlorobenzene (c) m-Dichlorobenzene (d) p-Dichlorobenzene
13. Ethylidene chloride is a/an ______________.
(a) vic-dihalide (b) gem-dihalide (c) allylic halide (d) vinylic halide
14. A primary alkyl halide would prefer to undergo _____________.
(a) SN1 reaction (b) SN2 reaction (c) α–Elimination (d) Racemisation
15 . Which of the following alkyl halides will undergo SN1 reaction most readily?
(a) (CH3)3C—F (b) (CH3)3C—Cl (c) (CH3)3C—Br (d) (CH3)3C—I
Answer:
1 2 3 4 5 6 7 8 9 10 11 12 13 14 15

a b b b b c a a c c a d b b d
ASSERTION REASON TYPE QUESTIONS (1MARK)
For following questions select the most appropriate answer from the options given below:
a. Both A and R are true and R is the correct explanation of A
b. Both A and R are true but R is not the correct explanation of A.
c. A is true but R is false.
d. A is false but R is true.
1. Assertion: Thionyl chlorides are preferred over phosphorus chlorides (tri and penta) for the
preparation of alkyl chlorides from alcohols.
Reason: Phosphorus chlorides give pure alkyl halides.
2. Assertion: KCN reacts with methyl chloride to give methyl isocyanide
Reason: CN – is an ambident nucleophile.
3. Assertion: tert-Butyl bromide undergoes Wurtz reaction to give 2, 2, 3, 3-tetramethylbutane.
Reason: In Wurtz reaction, alkyl halides react with sodium in dry ether to give hydrocarbon
containing double the number of carbon atoms present in the halide.
4. Assertion: Presence of a nitro group at ortho or para position increases the reactivity of haloarenes
towards nucleophilic substitution.
Reason: Nitro group, being an electron withdrawing group decreases the electron density over the
benzene ring.
5. Assertion: In monohaloarenes, further electrophilic substitution occurs at ortho and para
positions.
Reason: Halogen atom is a ring deactivator.
Answer
1 2 3 4 5

c d a a b
SHORT ANSWER TYPE QUESTIONS (2 MARKS)
1. Arrange each set of compounds in order of increasing boiling points.
(a) Bromoethane, Bromoform, Chloroethane, Dibromoethane.
(b) 1-Chloropropane, Isopropyl chloride, 1-Chlorobutane.

54 | Page
Ans: (a) Chloroethane <Bromoethane <Dibromoethane < Bromoform
(b) Isopropyl chloride < 1-chloropropane < 1- Chlorobutane
2. (a) Why does p-dichlorobenzene have a higher m.p. than its o- and m-isomers?
(b) Why is (±)-Butan-2-ol optically inactive?
Ans: (a) p-isomers are comparatively more symmetrical and fit closely in the crystal lattice, thus
require more heat to break these strong forces of attraction. Therefore, higher melting point than o-
and m-isomers.
(b) CH3CH(OH)CH2CH3 (±)-Butan-2-ol is optically inactive because in racemic mixture one type of
rotation is cancelled by other.
3. Which compound in each of the following pairs will react faster in SN2 reaction with —OH?
(a) CH3Br or CH3I (b) (CH3)3 CCl or CH3Cl
Ans: (a) CH3I: Because Iodide is better leaving group than bromide.
(b) CH3Cl: Carbon atom leaving group is less hindered.
4. Explain why: (a) The dipole moment of chlorobenzene is lower than that of cyclohexyl chloride.
(b) Alkyl halides, though polar, are immiscible with water.
Ans: (a) Chlorobenzene has lower dipole moment than cyclohexyl chloride due to lower magnitude of
-ve charge on the Cl atom and shorter C – Cl distance. Due to greater S character, a sp2 -hybrid carbon
is more electronegative than a sp3-hybrid carbon. Therefore, the sp2 -hybrid carbon of C – Cl bond in
chlorobenzene has less tendency to release electrons to Cl than a sp3 hybrid carbon of cyclohexyl
chloride.
(b) Alkyl halides and polar molecules are held together by dipole-dipole interaction. The molecules of
H2O are held together by H- bonds. Since the new forces of attraction between water and alkyl halide
molecules are weaker than the forces of attraction already existing between alkyl halide-alkyl halide
molecules and water molecules, therefore alkyl halides are immiscible (not soluble) with water.
5. Account for the following:
(a) The C–Cl bond length in chlorobenzene is shorter than that in CH3–Cl
(b) Grignard reagent should be prepared under anhydrous conditions.
Ans: (a) In haloalkanes, the halogen atom is attached to sp3 -hybridized carbon while in haloarenes it
is attached to sp2 -hybridized carbon whose size is smaller than sp3 orbital carbon. Therefore C–Cl
bond in chloro-benzene is shorter than alkyl chloride.
(b) Grignard reagents are very reactive. In the presence of moisture, they react to give alkanes.
Therefore, Grignard reagents should be prepared under anhydrous conditions.
RMgX +H2O→ RH +Mg (OH)X
6. Draw the structure of major mono halo product in each of the following reactions:

Ans:

7. Give reason for the following:


(a) During the electrophilic substitution reaction of haloarenes, para substituted derivative is the
major product.
(b) The product formed during SN1reaction is a racemic mixture.
55 | Page
Ans: (a) At the ortho position, higher steric hindrance is there, hence para isomer is usually
predominate and is obtained in the major amount.
(b) During the SN1 mechanism, intermediate carbocation formed is sp2 hybridized and planar in
nature. This allows the attack of nucleophile from either side of the plane resulting in a racemic
mixture.
8. Identify the major product formed when 2-cyclohexylchloroethane undergoes a
dehydrohalogenation reaction. Name the reagent which is used to carry out the reaction.
Ans: The major product formed when 2-cyclohexylchloroethane undergoes dehydrohalogenation
reaction is 1- cyclohexylethene. The reagent which is used to carry out the reaction is ethanolic KOH
9. (a) Give the IUPAC name of the following compound:

(b) Predict the order of reactivity of the following compounds in SN1 reaction:
C6H5CH2Br, C6H5C(CH3) (C6H5) Br, C6H5CH(C6H5) Br, C6H5CH(CH3) Br
Ans: (a) 4-Bromo-3-methylpent-2-ene
(b) C6H5C(CH3) (C6H5) Br> C6H5CH(C6H5) Br> C6H5CH(CH3) Br> C6H5CH2Br
10. (a) Predict the increase in order of reactivity of four isomeric bromobutanes in SN1 reaction.
(b) What happen when bromomethane react with sodium iodide in the presence of dry acetone.
Ans: (a)

(b) Iodomethane and sodium bromide formed (Finkelstein Reaction)


SHORT ANSWER TYPE QUESTIONS (3 MARKS)
1. Rearrange the compounds of each of the following sets in order of reactivity towards S N2
displacement:
(a) 2-Bromo-2-methylbutane, 1-Bromopentane, 2-Bromopentane
(b) l-Bromo-3-methylbutane, 2-Bromo-2-methyl-butane, 3-Bromo-2-methylbutane
(c) 1-Bromobutane, l-Bromo-2,2-dimethyl-propane, l-Bromo-2-methylbutane Answer:
Ans: (a) 1-Bromopentane > 2-Bromopentane > 2-Bromo-2-methylbutane
(b) 1-Bromo-3-methylbutane > 3-Bromo-2 methylbutane> 2-Bromo-2-methylbutane
(c) 1-Bromobutane > 1-Bromo-2-methylbutane > 1-Bromo-2, 2-dimethylpropane
2. Give reasons for the following:
(a) Ethyl iodide undergoes SN2 reaction faster than ethyl bromide.
(b) It is necessary to avoid even traces of moisture during the use of a Grignard reagent.
(c) C—X bond length in halobenzene is smaller than C—X bond length in CH3—X.
Ans: (a) I, is better leaving group/C—I bond is weaker than C—Br bond.
(b) Grignard reagents are highly reactive and react with water to give corresponding
hydrocarbons.
RMgX+H2ORH + Mg(OH)X
(c) In halobenzene, halogen atom is attached to the sp2 hybrid carbon atom while in CH3-X
halogen atom is attached to sp3 hybrid carbon atom. Hence C-X bond length in halo benzene is smaller
than CH3-X
3. Give reasons:
(a) n-Butyl bromide has higher boiling point than t-butyl bromide.
(b) Racemic mixture is optically inactive.
(c) The presence of nitro group (-NO2) at o/p positions increases the reactivity of haloarenes
towards nucleophilic substitution reactions.
Ans: (a) n-Butyl bromide has higher boiling point than t-butyl bromide because it has larger surface
area hence have more Van der Waals’ forces.

56 | Page
(b)Rotation due to one enantiomer is cancelled by another enantiomer. The presence of nitro
group (-NO2) at ortho and para positions withdraws the electron density’ from benzene ring and thus
facilitating the attack of nucleophile.
4. Why can aryl halides not be prepared by reaction of phenol with HCl in the presence of ZnCl2?
Ans: [C—O bond in phenols is more stable due to resonance effect and it has double bond character,
hence breaking of this bond is difficult.]
5. Cyanide ion acts as an ambient nucleophile. From which end it acts as a stronger nucleophile in the
aqueous medium? Give a reason for your answer.
Ans: [It acts as a stronger nucleophile from the carbon end because it will lead to the formation of C–C
bond which is more stable than the C–N bond.]

LONG ANSWER TYPE QUESTIONS (5 MARKS)


1. A compound ‘A’ with molecular formula C4H10O is a unreactive towards sodium metal. It does
not discourse Bromine water and does not react with NaHSO3 solution. On refluxing ‘A’ with
excess of HI gives ‘B’ which react with aq. NaOH to form ‘C’. ‘C’ can be converted into ‘B’ by
reacting with P and I2 . ‘C’ on heating with conc. H2SO4 forms ‘D’ decolorizes bromine water.
Identify A to D and write the reactions involved.
Ans: A is not alcohol therefore it does not react with Sodium metal. ‘A’ is not aldehydes and
ketones as it does not react with NaHSO3 ‘A’ is not unsaturated hydrocarbon as it does not add
Br2 (aq). It is likely to be ether.
CH3CH2OC2H5 + 2HI  2C2H5I + H2O
‘A’ excess ‘B’
C2H5I + NaOH (aq)  C2H5OH + NaI
‘B’ ‘C’
C2H5OH + (P/I2) C2H5I
C B
C2H5OH + Conc. H2SO4 C2H4
C D
2. (a) What are ambident nucleophiles? Explain with an example.
(b) Haloalkanes easily dissolve in organic solvents, why?
(c) Why chloroform stored in completely filled closed dark coloured bottles.
(d) Of the two bromo derivatives, C6H5CH (CH3)Br and C6H5CH(C6H5)Br, which one is more
reactive in SN1 substitution reaction and why?
Ans: (a) An ambident nucleophile is a nucleophile that can form new bonds at two or more spots
in its structure, usually due to resonance contributors. Example: SCN– can act as a nucleophile with
either the S or N is attacking.
(b) Because the new forces of attraction set up between haloalkanes and solvent molecules are of
the same strength as the forces of attraction being broken.
(c) Chloroform is slowly oxidized by air in the presence of light to an extremely poisonous gas,
carbonyl chloride, also known as phosgene.
(d) Since the reactivity of SN1 reactions increases as the stability of intermediate carbocation
increases. Of the two 2° bromides, the carbocation intermediate derived from C6H5-CH(C6H5)Br i.e.
C6H5C+HC6H5 is more stable as compared to the carbocation C6H5C+HCH3 obtained from
C6H5CH(CH3)Br because it is stabilized by two phenyl groups due to resonance.
CASE-BASED QUESTION (4 MARKS)
1. Read the passage given below and answer the following questions:
Nucleophilic substitution reactions are of two types; substitution nucleophilic bimolecular (SN2) and
substitution nucleophilic unimolecular (SN1) depending on molecules taking part in determining the
rate of reaction. Reactivity of alkyl halide towards SN1 and SN2 reactions depends on various factors
such as steric hindrance, stability of intermediate or transition state and polarity of solvent. S N2
reaction mechanism is favoured mostly by primary alkyl halide or transition state and polarity of
solvent, SN2 reaction mechanism is favoured mostly by primary alkyl halide then secondary and then
tertiary. This order is reversed in case of SN1 reactions.

57 | Page
(a) Why are haloalkanes more reactive towards nucleophilic substitution reactions than haloarenes
and vinylic halides?
(b) Allyl chloride is hydrolysed more readily than n-propyl chloride. Why?
(c) Name the two possible alkenes which will yield 1-chloro-1-methylcyclohexane on their reaction
with HCl. Write the reactions involved.
OR
(c) (i) Which alkyl halide from the following pair is chiral and undergoes faster SN2 reaction.
(a) (b)
Br
Br
(ii) Out of SN1 and SN2 which reaction occurs with
(a) Inversion of configuration (b) Racemisation
Ans: (a) Haloalkanes are more reactive than haloarenes and vinylic halides because of the presence of
partial double bond character C-X bond in haloarenes and vinylic halides. Hence they do not undergo
nucleophilic reactions easily.
(b) Allyl chloride shows high reactivity as the carbocation formed in the first step is stabilised by
resonance while no such stabilisation of carbocation exists in the case of n-propyl chloride.
(c)

OR
(c) (i) structure b is chiral, but faster SN2 is given by a, as it is primary and less bulky.
(ii) (a) SN2 (b) SN1
Alcohols, Phenols and Ethers
Quick Revision Points
Alcohols Phenols Ethers

Classification of Alcohols and Phenols


● Alcohols and phenols may be classified as mono–, di–, tri- or polyhydric compounds depending on
whether they contain one, two, three or many hydroxyl groups respectively

Classification of Monohydric alcohols


according to the hybridization of the carbon atom to which the hydroxyl group is attached.
(i) Compounds containing 𝐶𝑠𝑝 3 − 𝑂𝐻 bond:
(a) Alkyl alcohols (b) Allylic alcohols (c) Benzylic alcohols
–OH group is attached to an —OH group is attached to a sp3 —OH group is attached to a
sp3 hybridised carbon atom of hybridised carbon adjacent to the sp3—hybridised carbon atom
an alkyl group. carbon-carbon double bond, that next to an aromatic ring.
is to an allylic carbon.

58 | Page
(ii) Compounds containing 𝐶𝑠𝑝 2 − 𝑂𝐻 bonds:
(a)Vinylic alcohol (b) Aryl alcohols or Phenols
—OH group bonded to a carbon-carbon —OH group bonded to the sp2-hybridized carbon
double bond, i.e., to a vinylic carbon atom of an aromatic ring

Classification of Ethers
(a) simple or symmetrical ether- if the alkyl or aryl Structures of Functional Groups
groups attached to the oxygen atom are the same.
Diethyl ether, C2H5OC2H5.

(b) mixed or unsymmetrical ether- if the two


groups are different. C2H5OCH3 and C2H5OC6H5.
Isomerism in Alcohols: Alcohols exhibit four types of isomerism: (i)Chain isomerism: Butan-1-ol, 2-
methylpropan-1-ol (ii)Position isomerism: Propan-1-ol, propan-2-ol (iii)Functional isomerism:
Ethanol, methoxy methane (iv)Optical isomerism: Monohydric alcohols containing chiral carbon
atoms exhibit optical isomerism. E.g. Butan-2-ol, Pentan-2-ol
Nomenclature of Alcohols:According to IUPAC system, ‘e’ of the parent alkane is replaced with the
suffix ‘ol’. For naming polyhydric alcohols, the ‘e’ of an alkane is retained and the ending ‘ol’ is added.
E.g. C2H5OH (Ethane + ol) = Ethanol
Nomenclature of Phenols: - The simplest hydroxy derivative of benzene is phenol. It is its common
name and IUPAC name. For disubstituted compounds, the terms ortho (1,2- disubstituted), meta (1,3
disubstituted), and para (1,4-disubstituted) are often used in the common names.E.g.C6H5OH (Phenol)
Nomenclature of Ethers: - The larger (R) group is chosen as the parent hydrocarbon and smaller
group is written as alkoxy or phenoxy. CH3OC2H5(Methoxy Ethane)
Preparation of Alcohols
(1)From alkenes
(i)By acid catalysed hydration: Mechanism
Alkenes react with water in the Step 1: Protonation of alkene to form
presence of acid as a catalyst to form carbocation by electrophilic attack of .
alcohols.
In case of unsymmetrical alkenes, the
addition reaction takes place in
accordance with Markovnikov’s rule

(ii) By hydroboration–oxidation:

propene Diborane Tripropylborane


(2). From carbonyl compounds
By reduction of aldehydes and ketones:
Reducing agents : with catalyst, sodium borohydride (NaBH4) or LiAlH4.
Aldehydes yield primary alcohols whereas ketones give secondary alcohols.

59 | Page
(3) By reduction of carboxylic acids and esters:

(4) From Grignard reagents:

Methanal→ primary alcohol , Other aldehyde → secondary alcohol , Ketones → tertiary alcohol
Preparation of Phenols
(1)From haloarenes (3). From benzene sulphonic acid

(2)From diazonium salts 4. From cumene

Physical Properties of Alcohols and Phenols


Boiling Point- b.p is directly proportional to the molecular mass (as the number of C atoms increases
the van der Waal’s forces increase but as the branching of C chain increases the surface area decreases
so b.p decreases) b.p of alcohols are higher than corresponding hydrocarbons due to strong
intermolecular H bonding. Alcohols > phenols> Ethers> alkanes- Decreasing order of Boiling point.
Solubility
• Soluble in water due to their ability to form hydrogen bonds with water molecules.
• The solubility decreases with an increase in the size of alkyl/aryl (hydrophobic) groups.
• Several of the lower molecular mass alcohols are miscible with water in all proportions.
Chemical Reactions of Alcohols and Phenols
(i) Reactions involving cleavage of O – H Bond
(a) Acidity of alcohols and phenols:
R – OH + 2Na →2R – ONa +H2 C6H5 – OH + 2Na →2C6H5 – ONa +H2
Alcohol Sodium Alkoxide Phenol Sodium Phenoxide
Alcohols are weaker acids than water due to the (+I effect) group present in alcohols, which decreases
the polarity of -O-H bond. Acid strength of alcohols: 1° > 2° > 3° Electron releasing group increases
electron density on oxygen to decrease the polarity of the –OH bond. Order of acidity is: .
𝑅𝐶𝑂𝑂𝐻 > 𝐻2 𝐶𝑂3 > 𝐶6 𝐻5 𝑂𝐻 > 𝐻2 𝑂 > 𝑅𝑂𝐻Phenol is more acidic than alcohol due to stabilization of
phenoxide ion through resonance.

Presence of electron withdrawing groups ( , -CN, -X) increases the acidity of phenol by stabilising
phenoxide ions while the presence of electron releasing groups (-R, - , -OR) decreases the acidity of
phenol by destabilising phenoxide ions. Thus, increasing acidic strength is o-cresol < p-cresol < m-
cresol < phenol < o-nitrophenol < 2, 4, 6 Trinitrophenol (picric acid) Higher Ka and lower pKa values
correspond to the stronger acid.
(b) Esterification
Alcohols and phenols react with carboxylic acids, acid chlorides, and acid anhydrides to form esters.

60 | Page
The introduction of acetyl (CH3CO-) group in phenols is known as acetylation.
(ii) Reactions involving cleavage of carbon–oxygen (C–O) bond in Alcohols
1. Reaction with hydrogen halides:
• Alcohols react with hydrogen halides to form alkyl halides.
ROH + HX →R–X +
• Reactivity of Alcohols: Tertiary > Secondary > Primary
• Lucas Test ( Conc. HCl + Anhyd. ZnCl2): - • tertiary alcohols- Immediate turbidity
(insoluble R-Cl formation)
• Secondary alcohols- Turbidity appears After 5 minutes
• Primary alcohols- No turbidity at room temperature.
2. Reaction with phosphorus trihalides: (Refer to chapter Haloalkanes for reaction)
3. Dehydration:

4. Oxidation:
• Oxidation of alcohols involves the formation of a carbon-oxygen double bond with cleavage
of O-H and C-H bonds.

• Also known as catalytic dehydrogenation reactions as it involves loss of dihydrogen from an


alcohol molecule
• Primary alcohol → Aldehyde → Carboxylic acid
• Primary alcohol → Carboxylic acid (strong oxidizing agent, acidified )
• Secondary alcohol → Ketone
• CrO3 (Chromic anhydride) in an anhydrous medium is used as the oxidizing agent for the
isolation of aldehydes.

• A better Oxidizing agent: pyridinium chlorochromate (PCC), a complex of chromium


trioxide with pyridine and HCl. (PCC retains C=C bond, if present)

• Secondary alcohols are oxidised to ketones by chromic anhydride (CrO3).

• Tertiary alcohols do not undergo oxidation reactions.


• When the vapours of a primary or secondary alcohol are passed over heated copper at 573
K, dehydrogenation takes place and an aldehyde or a ketone is formed while tertiary
alcohols undergo dehydration.
Chemical Reactions of Phenols
1. Electrophilic aromatic substitution on the aromatic ring of Phenol -OH group is activating and
ortho and para directing. Due to resonance electron density is more at o- and p- position
61 | Page
Nitration Halogenation (Bromination)

2. Kolbe’s reaction

3. Reimer-Tiemann reaction

4. Reaction of phenol with zinc dust 5. Oxidation

Preparation of Ethers
1. By dehydration of alcohols 2. Williamson synthesis
• An alkyl halide is reacted with sodium alkoxide.

• Better results are obtained if the alkyl halide is primary (as


tert. halide under goes elimination reaction)

Aryl halide + sodium


alkoxide→?
This method is not appropriate to
prepare mixed Ethers
Chemical Reactions of Ethers
1. Reactions involving Cleavage of C–O bond in ethers

Alkyl aryl ethers are cleaved at the alkyl-oxygen bond due to the
more stable aryl-oxygen bond. The reaction yields phenol and
alkyl halide

62 | Page
When primary or secondary alkyl groups are present, it is the lower alkyl group that forms
alkyl iodide (SN2 reaction).

• when one of the alkyl group is a tertiary group, the halide formed is a tertiary halide (SN1
reaction).

2. Electrophilic substitution
The alkoxy group (-OR) is ortho, para directing and activates the aromatic ring towards
electrophilic substitution

MULTIPLE CHOICE QUESTIONS (1 MARKS)


1. Which Compound is predominantly formed when phenol is allowed to react with bromine in
aqueous medium?
a) Picric Acid (b) o-bromophenol (c)2,4,6-tribromophenol (d) p-bromophenol
2.

(a) Toluene (b) Cresol (c) Benzene (d) Benzol


3. The correct order of boiling point on Primary (1 ), Secondary (2 ) and tertiary (30)isomeric alcohol
0 0

is
(a) 10>20>30 (b) 30>20>10 (c) 20>10>30 (d) 20>30>10
4. The reaction: -

(a) Reimer-Tiemann Reaction (b) Kolbe’s Reaction


(c) Williamson’s synthesis (d) Wurtz Reaction
5. Which of the following alcohols will not undergo oxidation?
(a) Butanol (b) Butan-2-ol (c) 2-methylbutan-2-ol (d) 3-methyl butan-2-ol
6. Dehydration of alcohol is an example of which type of reaction:
(a) Substitution (b) Elimination (c) Addition (d) rearrangement
7. Among the following sets of reactants which one produces anisole?
(a) CH3CHO, RMgX (b) C6H5OH, NaOH, CH3I
(c) C6H5OH, neutral FeCl3 (d) C6H5CH3, CH3COCl, AlCl3
8. The compound that reacts fastest with Lucas reagent (at room temperature) is: -
(a) Butan-1-ol (b)Butan-2-ol (c) 2-methyl propan-2-ol (d) 2-methyl propan-1-ol
9. The compound A on treatment with Na gives B, and with PCl5 gives C. B and C react together to give
diethyl ether. A, B and C are respectively
63 | Page
(a) C2H5OH, C2H5ONa, C2H5Cl (b) C2H5OH, C2H5Cl, C2H5ONa
(c) C2H5Cl, C2H6, C2H5OH (d) C2H5OH, C2H6, C2H5Cl
10. What additives are used in the process of denaturation of alcohols
(i) Nitric acid (ii) Copper sulphate (iii) pyridine (iv) Sulphuric acid
(a) I & II (b) ii & iii (c) ii & iv (d) iii & iv
11. Give IUPAC name of the compound

a) 2-Chloro-5-hydroxyhexane c) 5-Chlorohexan-2-ol
b) 2-Hydroxy-5-chlorohexane d) 2-Chlorohexan-5-ol
12. Phenol is less acidic than
(a) Ethanol b) o-nitrophenol c) o-methylphenol d) o-methoxyphenol
13. Arrange the following compounds in increasing order of boiling point.
Propan-1-ol, butan-1-ol, butan-2-ol, pentan-1-ol
(a) Propan-1-ol, butan-2-ol, butan-1-ol, pentan-1-ol
(b) Pentan-1-ol, butan-2-ol, butan-1-ol, propan-1-ol
(c) Propan-1-ol, butan-1-ol, butan-2-ol, pentan-1-ol
(d) Pentan-1-ol, butan-1-ol, butan-2-ol, propan-1-ol
14. Which of the following reagents can be used to oxidise primary alcohols to aldehydes?
(i) 𝐶𝑟𝑂3 in anhydrous medium. (iii) Pyridinium chlorochromate
(ii) 𝐾𝑀𝑛𝑂4 in acidic medium (iv) Heat in the presence of Cu at 573K.
(a)(i), (ii) (b) (i), (iii), (iv) (c) (ii), (iii), (iv) (d) (iii), (iv)
15. Which one is secondary alcohol

a) (i) b) (ii) c) (iii) d) (iv)

ANSWERS MULTIPLE CHOICE QUESTIONS:


Q.No 1 2 3 4 5 6 7 8 9 10 11 12 13 14 15
Answer c) c) a) a) c) b) b) c) a) b) c) b) a) b) d)

ASSERTION REASON TYPE QUESTIONS (1MARK)


The following questions are statement of assertion followed by a statement of reason is given.
Choose the correct answer out of the following choices.
a) Assertion and reason both are correct statements and reason is correct explanation for
assertion.
b) Assertion and reason both are correct statements but reason is not correct explanation for
assertion.
c) Assertion is correct statement but reason is wrong statement.
d) Assertion is wrong statement but reason is correct statement.
1. Assertion: p-hydroxybenzoic acid is more acidic than phenol.
Reason: carboxyl group helps in the stabilisation of the phenoxide ion by dispersal of
negative charge due to resonance.
2. Assertion: Phenol is a stronger acid than ethanol.
Reason: Ethanol reacts with Na and phenol reacts with Na and NaOH.
3. Assertion: o-Nitrophenol is more volatile than p-nitrophenol.
Reason: o-nitrophenol has intramolecular H- bonding but p-nitrophenol has inter
molecular H- bonding.
4. Assertion: Lucas reagent is used to distinguish between 10, 20 and 30 alcohols.
Reason: Surface area of 30 alcohol is more than 20 and 10.
5. Assertion: Bromination of phenol does not required the presence of Lewis acid.

64 | Page
Reason: -OH group attached to benzene ring has highly activating effect.
ANSWERS:
Q.No. 1 2 3 4 5
Answer a) b) a) c) a)
SHORT ANSWER TYPE QUESTIONS (2 MARKS)
1. For the pair phenol and cyclohexanol. Answer the following: -
(a) Why phenol more acidic than cyclohexanol?
(b) Give one chemical test to distinguish between the two
Answer: - a) Phenol is more acidic than cyclohexanol because the conjugate base of phenol i.e.
phenoxide is more stable than conjugate base of cyclo hexanol due to resonance.
b) Phenol gives violet colour with neutral FeCl3 solution due to formation of Fe[Co(C6H5)6]complex.
2. Two compound (A) and (B) have same molecular formula C4H10O. Both reacts with Na metal librate
H2 gas. Compound (A) gives yellow ppt. When reacts with I2 in NaOH but compound (B) does not
respond to I2 in NaOH. Identify (A) and (B)
Ans:- (A) and (B) are Alcohols as they reacts with Na metal to give H2 gas.
(A) Butan-2-Ol ( B )2- methyl propan-2-ol
3. L-Butyl chloride on heating with sodium methoxide give 2-methyl propane instead of t-butyl methyl
ether. Explain
Ans (Hint) L-Butyl chloride is 30 alkyl halide and CH3ONa is a strong base
4. Nitration is an example of aromatic electrophilic substitution and its rate depends upon the group
already present in the benzene ring. Out of benzene and phenol, which one is more easily nitrated and
why?
Answer: -Phenol will be easily nitrated since the —OH group attached to the benzene ring activates it
towards electrophilic substitution due to +R effect.
5. In Kolbe’s reaction, instead of phenol, phenoxide ion is treated with carbon dioxide. Why?
Answer: -Phenoxide ion is more activating than phenol towards electrophilic substitution reaction.
Hence, it undergoes electrophilic substitution with carbon dioxide, a weak electrophile.
6. Arrange the following sets of compounds in order of their increasing boiling points:
(a) Pentan-1-ol, butan-1-ol, butan-2-ol, ethanol, propan-1-ol, methanol.
(b) Pentan-1-ol, n-butane, pentanal, ethoxyethane.
Answer: - (a) Methanol, ethanol, propan-1-ol, butan-2-ol, butan-1-ol, pentan-1-ol.
(b) n-Butane, ethoxyethane, pentanal and pentan-1-ol.
7. The carbon– oxygen bond length (136 pm) in phenol is slightly less than that in methanol (142 pm).
Explain.
Answer: -This is due to
(i) Partial double bond character of C-O bond because of resonance in phenol and
(ii) sp2 hybridised state of carbon of phenol to which oxygen is attached.
8. Give reasons: -

a) The bond angle in alcohols is slightly less than the tetrahedral angle (109°-28’).

b) In methoxymethane bond angle is slightly greater than the tetrahedral angle


Answer: a)It is due to the repulsion between the unshared electron pairs of oxygen.
b) It is due to more repulsive interaction between the two bulky methyl groups.
9. Give the IUPAC names of the following compounds: -

Answer: -(i) 2,6-Dimethylphenol (ii) 1-Ethoxy-2-nitrocyclohexane

10. Ethers can be prepared by Williamson synthesis in which an alkyl halide is reacted with sodium
alkoxide. Di-tert-butyl ether can’t be prepared by this method. Explain.

65 | Page
Answer: -In tert-butyl halides, elimination is favoured over substitution, so alkene is the major
product formed and ether is not formed.

SHORT ANSWER TYPE QUESTIONS (3 MARKS)


1. Give reasons for the following: -
(i) 30 halides are not used to prepare ether by williamson’s ether synthesis.
(ii) o-nitrophenol is less soluble in water than the m & p -isomers
(iii) Phenols give o & p nitrophenol on nitration with conc HNO3 and sulphuric acid mixture.
Answers: - (i) Because 30 halides undergo elimination reaction and produce alkene.
(ii) Due to intramolecular H-bonding in o-nitrophenol and intermolecular H-bonding in m & p
isomers.
(iii) Due to o,p-directing effect of -OH group.
2. Give one chemical test each to distinguish between the following pairs of compounds:
(i) Phenol and Benzoic acid (ii) Propan-1-ol and Propan-2-ol (iii) Methanol and ethanol.
Answer: - (i)Add 𝑁𝑎𝐻𝐶𝑂3 to each. Phenol will not react, whereas benzoic acid will give brisk
effervescence due to 𝐶𝑂2 .
(ii) Add 𝐼2 and NaOH to each one. Propan-2-ol will give yellow ppt. of iodoform (iodoform test),
whereas propan-1-ol will not give yellow ppt.
(iii) Add 𝐼2 and NaOH to each one. Ethanol will give yellow ppt. of iodoform (iodoform test),
whereas methanol will not give yellow ppt.
3. Give the major products that are formed by heating each of the following ethers with HI.

4. Give the structures and IUPAC names of the products expected from the following reactions:
(a) Catalytic reduction of butanal.
(b) Hydration of propene in the presence of dilute sulphuric acid.
(c) Reaction of propanone with methylmagnesium bromide followed by hydrolysis.

5. An organic compound (A) having molecular formula C6H6O gives a characteristic colour with
aqueous FeCl3 solution. (A) on treatment with NaOH gives (B), which on treatment with CO2 and
further acidification gives a compound (C). Identify A,B and C and Write the Chemical reactions
involved.

A B C
LONG ANSWER TYPE QUESTIONS (5 MARKS)
Q 1. (a) Give names of the reagents to bring about the following transformations:
(i) Ethanoic acid to ethanol
(ii) Propan-1-ol to propanal
66 | Page
(iii)Pent-3-en-2-ol to pent-3-en-2-one
(b) How will you distinguish between the following pairs of compounds?
(i) pentan-2-ol and pentan-3-ol
(ii) Phenol and ethanol
Ans. (a) (i) LiAlH4/H3O+(ii) Pyridinium ChloroChromate (iii)Pyridinium ChloroChromate
(b) (i) iodoform test is +ve for pentan-2-ol and will give yellow ppt of iodoform but no reaction with
pentan-3-ol
(ii) Neutral FeCl3 test is +ve for phenol but no reaction in ethanol
(2 ) Account for the following: -
(a) Alcohols are more soluble in water than the hydrocarbon of comparable molecular masses.
(b) Phenol doesn’t react with NaHCO3 whereas carboxylic acid reacts.
(c) Sodium metal can be used for drying Di ethyl ether but not ethyl alcohol.
(d) 2, 4, 6-trinitrophenol gives sodium bicarbonate test.
(e) Ortho nitro phenol is more volatile than para nitro phenol
Answer: -(a)Alcohols have hydrogen bonding whereas hydrocarbons have weak van der Waal‘s force
of attraction.
(b) Phenol is less acidic than carbonic acid whereas carboxylic acid is more acidic than carbonic acid.
(c) Sodium metal does not react with ether whereas it reacts with alcohol.
(d) 2, 4, 6-trinitrophenol is a stronger acid due to the presence of three nitro groups showing
electron-withdrawing effect.
(e)Ortho nitro phenol has intramolecular hydrogen bonding and so molecules are less associated with
each other whereas para nitro phenol has intermolecular hydrogen bonding.

2. CASE BASED QUESTIONS


Read the passage given below and answer the following questions:
Alcohols play very important role in our daily life. Ordinary spirit used as an antiseptic contains
methanol. Ethanol is present in cough syrups, tonics, wine, beer and whiskies, Sugar, starch, cellulose
are carbohydrates which also contain large number -OH groups. Phenol is also an antiseptic in low
concentration (0.2%) where as 2% solution of phenol is used as disinfectant. The fragrance of rose is
due to citronellal (unsaturated alcohol). Phenol is used for preparation of many useful compounds like
aspirin, methyl salicylate (Iodex) and phenyl salicylate (salol) used as intestinal antiseptic.
(a) How is phenol prepared from cumene?
(b) Why does phenol turn pink after long standing?
(c) Explain the industrial methods of preparation of Methanol.
OR
(c) Consumption of Methanol is lethal. Explain why?
Answer -(a)

(b) On standing for a long-time phenol gets oxidized in the presence of air and turns pink in
colour. During oxidation, phenol gets oxidised to quinone and then quinone combines with the
phenol to give phenoquinone, which is pink in colour.
(c) Commercially, methanol is prepared by catalytic reduction of carbon monoxide.
In the process of synthesis of methanol, water gas is used. Water-gas is an equimolar mixture
of carbon monoxide, CO and hydrogen, H2.
The reaction takes place in the presence of zinc oxide, ZnO and chromium oxide, Cr2O3 as the
catalyst and at 573K temperature.
The reaction is represented as follows:

67 | Page
Or
(c) Methanol(CH3OH) is highly toxic and unfit for consumption. Methanol ingested in large
quantities is metabolised to formic acid or formate salts, which is poisonous to the
central nervous system and may cause blindness, coma and death.
Aldehydes, Ketones & Carboxylic Acids
QUICK REVISION POINTS:

 General formula: C H O having >C=O group.


n 2n

 Aldehydes: where R = H, e.g., CH CHO, HCHO, C H CHO, etc.


3 6 5

 Ketones: where R = alkyl or aryl group. e.g., CH COCH , CH COC H , C H COC H , etc.
3 3 3 6 5 6 5 6 5

 Structure:

 Nomenclature: The common Name: Common name word for the given number of Carbon
atoms + aldehyde (e.g., Formaldehyde, Acetaldehyde, Benzaldeyde, etc.)
The IUPAC names: Alkanal (e.g., Methanal, Ethanal, etc.)

 Methods of preparation of Aldehydes and Ketones:


 Oxidation of alcohols:

Note: Tertiary alcohols can’t be oxidized at all without breaking C-C bonds, whereas primary alcohols
oxidize first to aldehydes and then to carboxylic acids. To stop it up to aldehyde stage only, mild

68 | Page
oxidizing agents like PCC (Pyridinium Chlorochromate: Chromium trioxide with Pyridine & HCl) are
used.
 Catalytic dehydrogenation of alcohols: 1°alcohols give aldehydes, 2° alcohols give ketones
and 3° alcohols give alkenes.

 Reductive ozonolysis of alkenes:1° and 2° alkenes give aldehyde and 3° gives ketones:

 Rosenmund reduction: In this reaction

Note: Used for both aliphatic & aromatic Acid halides. Used to increase the No. of C atoms in the
parentmolecules
Reduction of nitriles:

Alternatively, nitriles are selectively reduced by diisobutylaluminium hydride, (DIBAL-H) to imines


followed by hydrolysis to aldehydes:

From esters:
Note: DIBAL-H (Di-isobutyl Aluminum Hydride) is bulky reducing agent used for the partial
reduction of Esters, Nitriles, etc. to Aldehydes only. Unlike LiAlH , it will not reduce aldehydes further
4

to alcohols and Nitriles to Primary amines.


 From alkynes:

69 | Page
Oxidation of 1,2-glycols:

 From acyl chlorides:

Preparation of Aromatic aldehyde/ketone:

 Friedel-Crafts acylation:

 From nitriles:

 Physical properties of Aldehydes and Ketones:


(i)Most of the aldehydes (except formaldehyde which is a gas) are liquids at room
temperature. The lower ketones are colourless liquids and have a pleasant smell.
(ii) Both of these have relatively high b.p. as compared to hydrocarbons of comparable
molecular masses due to presence of polar carbonyl group. But they have lower b.p. than
alcohols of comparable molecular masses (due to absence of H-bonding)
compound b.p.(K) Molecular mass
n-Butane 273 58
Methoxyethane 281 60
Propanal 322 58
Acetone 329 58
Propan-1-ol 370 60

(iii) The lower members of aldehydes and ketones (up to four carbon atoms) are soluble in
water due to hydrogen bonding.

(iv)Aromatic aldehydes and ketones are much less soluble than corresponding aliphatic
aldehydes and ketones due to larger benzene ring.
(v) All carbonyl compounds are fairly soluble in organic solvents.
 Chemical properties:
 Nucleophilic addition, Addition-elimination, Oxidation & Reduction reactions:

70 | Page
Note: 1) Reaction with HCN is used to increase the number of C-atom in the parent molecule.
2) Reaction with NaHSO is called Bisulphite test in which we get the product in a crystalline form.
3

3) In the formation of Acetal/ketal, reagent HCl (g) is taken in vapour phase.


4) In Fehling solution copper sulphate (A) is taken with Rochelle Salt (B- Sodium potassium tartrate).
Alone Copper sulphate won’t solve the purpose. Rochelle salt have a chelating effect on the carbonyl
gr. That’s why, this method is not suitable with benzaldehyde.

71 | Page
Reactions due to α - Hydrogen:

The product formed by aldol condensation between benzaldehyde and acetone is benzalacetone.

72 | Page
(v)
Note: Aromatic aldehydes and ketones undergo electrophilic substitution at the ring in which the
carbonyl group acts as a deactivating and meta directing group.
 Uses of aldehydes and ketones:
 In chemical industry aldehydes and ketones are used as solvents, starting materials and
reagents for the synthesis of other products.
 Formaldehyde is well known as formalin (40%) solution used to preserve biological specimens
and to prepare Bakelite (a phenol-formaldehyde resin), urea-formaldehyde glues and other
polymeric products.
 Acetaldehyde is used primarily as a starting material in the manufacture of acetic acid, ethyl
acetate, vinyl acetate, polymers and drugs. Benzaldehyde is used in perfumery and in dye
industries.
 Acetone and ethyl methyl ketone are common industrial solvents.
 Many aldehydes and ketones, e.g., butyraldehyde, vanillin, acetophenone, camphor, etc. are
well known for their odours and flavours.
CARBOXYLIC ACIDS:
General Formula: C H O having—COOH group. RCOOH where, R=H or alkyl or aryl.
n 2n 2

Nomenclature: The common names end with the suffix –‘ic acid’ and have been derived from Latin or
Greek names of their natural sources.
–In the IUPAC system, aliphatic carboxylic acids are named by replacing the ending
–‘e’ in the name of the corresponding alkane with –‘oic acid’. In numbering the
carbon chain, the carboxylic carbon is numbered one.
Structure : In carboxylic acids, the bonds to the carboxyl carbon lie in one plane and are separated by
about 120°. The carboxylic carbon is less electrophilic than carbonyl carbon because of the possible
resonance structure.

Classification: They are classified as mono, di, tri and polycarboxylic acids depending upon the
number of carboxyl groups present in a molecule.

73 | Page
 Methods of preparation of Carboxylic acids:

(vi) From Alkylbenzene: (Irrespective of length of Alkyl group on bezene ring, Benzoic acid will be
formed always)

74 | Page
 Physical properties of Carboxylic acids:

 Chemical Properties: Chemical properties of carboxylic acids are classified as follows:

 The effect of
the following groups in increasing acidity order is:
Ph < I < Br < Cl < F < CN < NO < CF
2 3

 Thus, the following acids are arranged in order of increasing acidity (based on pK values
a

(-log of k values):
a

CF COOH > CCl COOH > CHCl COOH > NO CH COOH > NC-CH COOH >FCH COOH > ClCH COOH >
3 3 2 2 2 2 2 2

BrCH COOH > HCOOH > ClCH CH COOH >C H COOH > C H CH COOH > CH COOH > CH CH COOH
2 2 2 6 5 6 5 2 3 3 2

75 | Page
 Uses of Carboxylic Acids:
 Methanoic acid is used in rubber, textile, dyeing, leather and electroplating industries.
 Ethanoic acid is used as solvent and as vinegar in food industry.
 Hexane dioic acid is used in the manufacture of nylon-6, 6.
 Esters of benzoic acid are used in perfumery.
 Sodium benzoate is used as a food preservative.
 Higher fatty acids are used for the manufacture of soaps and detergents.
To distinguish Aldehydes from Ketones. Detection tests:
 Trick: TASty FAAli Red brown IMeLY
 TASty -:Tollen’s test, Aldehyde group, Silver mirror
FAAli -: Fehling’s test, Aliphatic Aldehyde
Red brown -:Red brown ppt in Fehling’s test
IMeLY -:Iodoform test, Methyl group Linked to -C=O- group, Yellow ppt
Cannizaro’s Reaction:
 Trick: CRAKN Reviews
Interpretation: Cannizzaro Reaction is given by Aldehydes and Ketones having no alpha-Hatom
To memorise regents used for converting -C=O- to alkanes in Clemmensen and Wolff-Kishner
reduction:
 Trick: Can Zebra Woo Nightingale
Interpretation: To memorise regents used for converting –C=O– to alkanes
C and Z- Clemmensen reduction -: Zn-Hg/HCl
W and N- Wolff- Kishner reduction -: NH NH /OH-
2 2

******************
MULTIPLE CHOICE QUESTIONS (1 MARK)
1. The addition of HCN to carbonyl compounds is an example of_
(a) Electrophilic addition (b) Nucleophilic addition
(c) Nucleophilic substitution (d) Electrophilic substitution
2. The major product of nitration of benzoic acid is-
(a) 2-nitrobenzoic acid (b) 4-nitrobenzoic acid
(c) 3-nitrobenzoic acid (d) 2,4- dinitro benzoic acid
3. What is the hybridization of the carbonyl oxygen in carboxylic acids?
(a) sp (b) sp2 (c) sp3 (d) dsp2
4. The greater acidity of carboxylic acids compared to alcohols arises primarily from:
(a) the electron-donating effect of the hydroxyl group
(b) the electron-withdrawing effect of the carboxyl oxygen
(c) the acidity of α-hydrogens of carboxylic acids
(d) the resonance stability associated with the carboxylate ion
5. Which of the following reagents will convert acetic acid into acetyl chloride?
(a) NaCl (b) HCl/ZnCl
2 (c) SOCl
2 (d) HCl
6. Acid chlorides undergo reduction to the corresponding aldehydes on treatment with hydrogen in
the presence of Pd/BaSO4. This reaction is called:
(a) Clemmensen reduction (b) Rosenmund reduction (c) Wolf-Kishner reduction (d) None of these
76 | Page
7. Which of the following orders of relative strengths of acids is correct?
(a) ClCH COOH > FCH2 COOH > BrCH2 COOH
2

(b) ClCH COOH > BrCH2 COOH > FCH2 COOH


2

(c) BrCH COOH > ClCH2 COOH > FCH2 COOH


2

(d) FCH COOH > ClCH2 COOH > BrCH2 COOH


2

8. Cannizzaro's reaction is not given by:


(a) C6H5CHO (b) HCHO (c)CCl3-CHO (d)CH3-CHO
9. Aldehydes and ketones are distinguished by which of the following test?
(a) Lucas test (b) Tollen’s test (c) KMnO4 solution (d) None of these
10. Phenylmethyl ketone can be converted into ethylbenzene in one step by which of the following
reagents?
(a) LiAlH4 (b) Zn-Hg/HCl (c) NaBH4 (d) CH3MgI
11. The reagent which does not react with both, acetone and benzaldehyde.
(a) Sodium hydrogen sulphite (b) Phenyl hydrazine (c) Fehling’s solution (d) Grignard reagent
12. Dimerization of carboxylic acids is due to
(a) ionic bond (b) covalent bond (c) coordinate bond (d) intermolecular hydrogen bond
13. In the anion HCOO– the two carbon-oxygen bonds are found to be of equal length. What is the
reason for it?
(a) Electronic orbitals of carbon atom are hybridised
(b) The C=O bond is weaker than the C–C bond
(c) The anion HCOO– has two resonating structures
(d) The anion is obtained by removal of a proton from the acid molecule
14. Which of the following undergoes haloform reaction?
(i) CH3CH2COCH2Cl (ii) C6H5COCH3 (iii) C6H5COCHCl2 (iv) CH3CH2COCCl3
(a) Only (ii) (b) (ii) and (iv) (c) (i), (ii) and (iv) (d) All the four
15. Oxidation of cyclohexene using acidified KMnO4 will give:
(a) adipic acid (b) hexane-1,6-dial (c) cyclohexane carboxylic acid (d) cyclopentane carboxylic acid

Question 1 2 3 4 5 6 7 8 9 10 11 12 13 14 15
Answer b c b d c b d d b b c d c d a

ASSERTION REASON TYPE QUESTIONS(1 MARK)


Note: In the following questions a statement of assertion followed by a statement of reason is
given.Choose the correct answer out of the following choices.
(a) Assertion is correct, reason is correct; reason is a correct explanation for assertion.
(b) Assertion is correct, reason is correct; reason is not a correct explanation for assertion.
(c) Assertion is correct, reason is incorrect.
(d) Assertion is incorrect, reason is correct.
1. Assertion: The boiling points of aldehydes and ketones are higher than hydrocarbons of
comparable molecular masses.
Reason: There is a molecular association in aldehydes and ketones arising out of the dipole-dipole
interactions.
2. Assertion: There are two -NH2 groups in semicarbazide (H2NNHCONH2) any one -NH2group can be
involved in the formation of semicarbazone.
Reason: Its due to resonance effect.
3. Assertion: The molecular mass of acetic acid in benzene is 120 instead of 60.
Reason: The carboxylic acids exist as cyclic dimers in which the two molecules of the acid are held
together by hydrogen bonds.
4. Assertion: Compounds containing –CHO group are easily oxidised to corresponding carboxylic
acids.
Reason: Carboxylic acids can be reduced to alcohols by treatment with LiAlH4.
5. Assertion: Lower aldehyde and ketones are soluble in water but the solubility decreases as
molecular mass increases.
Reason: Aldehydes and ketones can be distinguished by Tollen’s reagent.
77 | Page
Question 1 2 3 4 5
Answer a d a b b
CASE BASED QUESTIONS
The following table has boiling points of different classes of compounds. Study the table and answer
the questions based on the table and related studied concepts.
S.No. Compound Boiling Point (°C)
1. Ethanal 20.2
2. Ethanol 78
3. Acetone 56.2
4. Acetic Acid 118
5. Acetic Anhydride 139.8
6. Diethyl Ether 34.6
7. Acetamide 222
8. Ethyl Acetate 77.1
(a) Diethyl ether has a lower boiling point than ethanol. Why?
Ans. Because diethyl ether has weaker dipole-dipole interactions than ethanol which has
intermolecular hydrogen bonding.
(b) Distinguish Ethanol and Acetone on the basis of a chemical test.
Ans. Acetone gives orange ppt. with 2,4-DNP While ethanol does not or Ethanol gives orange colour
with Ceric ammonium nitrate while Acetone doesn’t or Ethanol gives turbidity with Lucas Reagent
(Conc.HCl and anhydrous AlCl ) while acetone doesn’t.
3

(c) Write the mechanism of the following reaction:

Ans. It is a nucleophilic bimolecular reaction involving an attack of alcohol molecules on protonated


alcohol.

SHORT ANSWER TYPE QUESTIONS (2 MARKS)


Q1. Give reasons:
(a) Chloroacetic acid is a stronger acid than acetic acid.
Ans. Cl is an electron withdrawing group, thus increases the acidity of carboxylic acids by stabilizing
the conjugate base through delocalization of negative charge by inductive effect
(b) PH of a reaction should be carefully controlled while preparing ammonia derivatives of carbonyl
compounds.
Ans. If the medium is too acidic, the ammonia derivatives being basic in nature will form the
respective ammonium salts. Now these ammonium salts are not nucleophilic so reaction will not
occur. However, if the medium is slightly acidic, the protonation of the carbonyl group will not take
place so this will not increase the electron deficiency on carbonyl carbon hence weak nucleophile
ammonia will not react.
Q2 . Arrange the following compounds in increasing order of their property as indicated:
(i) CH COCH , C H COCH , CH CHO (reactivity towards nucleophilic addition reaction)
3 3 6 5 3 3

(ii) Cl—CH —COOH, F—CH —COOH, CH —COOH (acidic character)


2 2 3

Ans. (i) C H COCH < CH COCH < CH CHO


6 5 3 3 3 3

(ii) CH —COOH < Cl—CH —COOH < F—CH —COOH


3 2 2

Q3. Name the reagents used in the following reactions:


(i) Benzyl alcohol to benzoic acid
78 | Page
(ii) Bromination of phenol to 2,4,6- Tribromo phenol.
Ans. (i) Acidified or alkaline KMnO 4

(ii) Aqueous Bromine (Br /H O)


2 2

Q4. Preparation of ethers by acid -catalysed dehydration of secondary and tertiary alcohols is not a
suitable method. Explain.
Ans. Due to steric hindrance in secondary and tertiary alcohols.
Q5. Although phenoxide ion has more number of resonating structures than carboxylate ion,
carboxylic acid is a stronger acid than phenol. Why?
Ans. The electronic charge in the carboxylate ion is more dispersed in comparison to phenate ion.
Carboxylate ion is more stable as compared to phenate ion. The release of H ion is easier from
+

carboxylic acid. It behaves as a stronger acid than phenol.


Q6. How will you prepare the following compounds from benzene? You may use any inorganic reagent
and any organic reagent having not more than one carbon atom.
(i) p-nitrobenzoic acid
(ii) p-nitrobenzaldehyde
Ans (i)

(ii)

Q7. Give reasons:


(i) Oxidation of aldehydes is easier than ketones
(ii) Benzoic acid is a stronger acid than acetic acid
Ans. (i) As aldehydes contain H atoms on the carbonyl group but ketones do not. Cleavage of C—H
bond in aldehydes is easier than cleavage of C—C bond in ketones.
(ii) This is because of greater electronegativity of sp2 hybridised carbonyl to which carboxyl carbon is
attached in benzoic acid.
Q8. An aromatic organic compound ‘A’ with molecular formula C8H8O gives positive Amine derivative
and methyl ketone’s identification tests. It neither gets oxidized by ammoniacal silver nitrate nor does
it decolorize the aqueous solution of the third member of the halogen group. Write the structure of A.
Ans: C6H5COCH3
Q9. While working in the chemistry lab, the teacher poured two types of aliphatic carbonyl
compounds in two separate test tubes. Later on, he forgot to label those test tubes. Now, being a
genius chemistry student, please help him to identify the suitable carbonyl compound in the test
tubes.
Ans: Aliphatic aldehyde when heated with Tollens’ reagent to give silver mirror but ketone does not.
Q10. Your teacher wants to make sanitizer in the lab. One of the chief ingredients of which is isopropyl
alcohol. On searching, he found that none of the alcohol was available in the lab. The teacher decided
to use propanone after treating it under some other reactions. What might have he done?
Ans.

79 | Page
SHORT ANSWER TYPE QUESTIONS (3 MARKS)
Q1. Give reasons:
(i) Alpha hydrogen of aldehydes and ketones are acidic in nature.
(ii) Propanone is less reactive than ethanal towards addition of HCN.
(iii) Benzoic acid does not give Friedel-Crafts reaction.
Ans. (i) due to the strong withdrawing effect of the carbonyl group and resonance stabilisation of the
conjugate base.
(ii) the presence of two methyl groups in propanone hinders more the approach of nucleophile to
carbonyl carbon than in ethanal having one methyl group and two methyl groups reduce the positivity
of the carbonyl carbon more effectively in propanone than in ethanal.
(iii) Carboxyl group is strongly deactivated and catalyst AlCl is a lewis acid that gets bonded to the
3

carboxyl group strongly.


Q2. Which of the following compounds will undergo aldol condensation, which the Cannizzaro
reaction and which neither?
(i) Methanal (ii) 2-Methylpentanal (iii) Benzophenone
Ans. (i)Methanal does not contain α-hydrogen and hence undergoes Cannizzaro reaction.
(ii) 2-Methylpentanal, contains α-hydrogens and hence undergoes aldol condensation.
(iii)Benzophenone is a ketone having no α-hydrogen. It neither undergoes aldol condensation nor
Cannizzaro reaction.
Q3.(a ) What is IUPAC name of-

(b) Write the structural formula of Isoamyl acetate.


(c) Convert benzaldehyde to Cinnamaldehyde?
Ans -1 (a) 4–hydroxy–3–methoxy-benzaldehyde
(b)

(c)
Q4.(a) Complete the following reactions:

Ans-

80 | Page
Q5.(a)Arrange the following compounds in increasing order of their boiling points.
CH CHO, CH CH OH, CH OCH , CH CH CH
3 3 2 3 3 3 2 3

(b)Arrange the following compounds in increasing order of their reactivity in nucleophilic addition
reactions
(i)Ethanal, Propanal, Propanone, Butanone.
(ii)Benzaldehyde, p-Tolualdehyde, p-Nitrobenzaldehyde, Acetophenone.
Ans-(a) CH CH CH < CH OCH < CH CHO < CH CH OH
3 2 3 3 3 3 3 2

(b)(i) Ethanal < Propanal < Propanone < Butanone


(ii) Acetophenone < p-tolualdehyde < Benzaldehyde < p-Nitrobenzaldehyde

Long Answer Type Questions (5 marks each):


Q.1 (a) An organic compound ‘A’ having molecular formula C5H10O gives negative Tollens’ test, forms
n-pentane on Clemmensen reduction but doesn’t give iodoform test.
Identify ‘A’ and give all the reactions involved.
(b) Carry out the following conversions:
(i) Propanoic acid to 2-Bromopropanoic acid
(ii) Benzoyl chloride to benzaldehyde
(c) How will you distinguish between benzaldehyde and acetaldehyde?
Ans- (a) Since, the compound (A) gives a positive iodoform test but negative Tollen's test, so it must
be a methyl ketone. From the given molecular formula C H O(A) can be 3-pentanone
5 10

(b) (i)

(ii)

(c) Distinction between acetaldehyde and benzaldehyde: - Acetaldehyde and benzaldehyde can be
distinguished by Fehling solution. Acetaldehyde gives red coloured precipitate with Fehling
solution while benzaldehyde does not. (Iodoform test - Acetaldehyde)
Q.2 (a) What happens when 2 moles of acetone are condensed in presence of Ba(OH) ? 2

Write chemical equation


(b)What happens when acetic acid is heated with P O ? 2 5

(c) Why is alpha hydrogen of carbonyl compounds acidic in nature?


(d) What happens when salicylic acid is heated with zinc dust?
(e) Why is p-hydroxy benzoic acid more acidic than p-methoxy benzoic acid?
Ans-(a)

(b)

(c) In a carbonyl group, the oxygen is extremely electronegative and it attracts the electron cloud
towards itself developing a partial positive charge on the α-carbon. To reduce the positive charge, α-
carbon loses its hydrogen readily and makes it acidic in nature.
(d)

81 | Page
(e) It is because —OH has more —I effect than —OCH group. 3

Q.3 (a) Carry out the following conversions:


(i) P-nitrotoluene to 2-bromobenzoic acid
(ii) Propanoic acid to acetic acid
(b) An alkene with molecular formula C H on ozonolysis gives a mixture of two compounds, B and C.
5 10

Compound B gives positive Fehling test and also reacts with iodine and NaOH solution. Compound C
does not give Fehling solution test but forms iodoform. Identify the compounds A, B and C.
Ans- (a) (i)

(ii)

(b)

B - Positive Fehling test - aldehyde group - CH CHO (Positive Iodoform test)


3

C - Negative Fehling test- ketone group - CH COCH (Positive Iodoform test)


3 3

REVISION CHART: 1
Reactants Conditions Products
RCH2OH Cu/573 K→ ?
CH3CH2=CH3 O3/Zn+H2O→ ?
RCOCl + H2 ?
Pd + BaSO4→
CH3 CH2CN ?
CH3MgBr &H3O+→
C6H5 CH3 CrO2Cl2→ ?
C6H6 Gattermann koch reaction→ ?
CH3CO CH3 HCN→ ?
C6H5 CHO Conc. KOH & Δ→ ?
CH3COOCH3 DIBAL-H & Δ + Water→ ?
CH3CH2CH2CN SnCl2+HCl/H3O+→ ?

82 | Page
REVISION CHART: 2
Reactants Conditions Products
C6H5 CH3 ?→ C6H5COOH
? CH3MgBr+ H3O+→ CH3COOH
CH3CH2CH2OH ?→ CH3CH2COOH
? + CH3OH H+→ CH3CH2COOCH3
? NH3+ Δ→ C6H5 CONH2
? Friedel Craft Acylation→ C6H5COCH3
? NH2OH→ CH3CH=NOH
C6H5CH2CH2CH3 ?→ C6H5COOH
? NH2NH2+ KOH→ CH3CH2CH3
? NaHSO3→ CH3CH(OH)SO3Na
? C6H5MgBr+Dry Ether + H3O+→ C6H5COCH2CH3 + Mg(NH2)Br
ANSWERS OF REVISION CHART: 1
Reactants Conditions Products
RCH2OH Cu/573 K→ RCHO
CH3CH2=CH3 O3/Zn+H2O→ CH3CHO +HCHO
RCOCl + H2 RCHO
Pd + BaSO4→
CH3 CH2CN CH3MgBr &H3O+→ CH3CH2COCH3

C6H5 CH3 CrO2Cl2→ C6H5CHO


C6H6 Gattermann koch reaction→ C6H5CHO
CH3COCH3 HCN→ CH3C(OH)(CN)CH3
C6H5CHO Conc. KOH & Δ→ C6H5CH2OH + C6H5COOK
CH3COOCH3 DIBAL-H & Δ + Water→ CH3CHO
CH3CH2CH2CN SnCl2+HCl/ H3O+→ CH3CH2CH2CHO + NH4Cl

ANSWERS OF REVISION CHART: 2


Reactants Conditions Products
C H CH 6 5 3 KMnO4+KOH+ Δ / H3O+→ C H COOH 6 5

CO 2 CH3MgBr+H3O+→ CH COOH 3

CH CH CH OH
3 2 2 Alk. KMnO4 + H3O+→ CH CH COOH 3 2

CH COOH + CH OH
3 3 H+→ CH CH COOCH
3 2 3

C H COOH
6 5 NH3+ Δ→ C H CONH
6 5 2

CH 6 6 Friedel Craft Acylation→ C H COCH 6 5 3

CH CHO 3 NH2OH→ CH CH=NOH 3

C H CH CH CH
6 5 2 2 3 KMnO4+KOH+ Δ / H3O+→ C H COOH 6 5

CH COCH 3 3 NH2NH2+ KOH→ CH CH CH 3 2 3

CH CHO 3 NaHSO3→ CH CH(OH)SO Na


3 3

CH CH CN 3 2 C6H5MgBr+Dry Ether+ H3O++→ C6H5COCH2CH3 + Mg(NH2)Br

83 | Page
Amines
QUICK REVISION POINTS:
Amines are alkyl / aryl derivatives of ammonia. In amines nitrogen atom is sp3 hybridised and
contains one lone pair.
Primary amine (10) Secondary amine (20) Tertiary amine (30)
One- H atom is replaced by R/Ar. Two-H atoms are Three-H atoms are replaced
replaced by R/Ar. by R/Ar.

IUPAC Name: -
Aliphatic amine: Alkanamines
Aromatic amine: Benzenamine/Aniline
CH3-NH2 Methanamine CH3-NH- CH3 (CH3)3N
CH3-CH- CH3 N-Methylmethanamine N, N-Dimethylmethanamine
NH2 Propan-2-amine
CH3-NH- CH2- CH3
N-Methylethanamine
CH3 -CH2-NH- CH2- CH3
N-Ethylethanamine
Aniline/benzenamine N, N- Dimethylbenzenamine
Preparation of Amines:
Reduction of Nitro Compounds gives aliphatic
and aromatic primary amines. (Reducing
Agents): -
Hydrogen and nickel/ palladium or platinum.
Iron scrap and hydrochloric acid (Fe + HCl).
Tin and hydrochloric acid (Sn + HCl).
Ammonolysis of alkyl halides: - NH3 RX RX RX
RX → RNH2 R 2 NH R 3 N R 4 N+X −
Reagent: Ethanolic solution of ammonia. −HX
Disadvantage: Mixture of amines formed.
Reduction of nitriles Reducing agents: - R − CN
H 2 /Ni or Na (Hg )/C 2 H 5 OH
R − CH2 NH2
LiAlH4/Catalytic hydrogenation.
Reduction of Amides: LiAll4
CH3 CONH2 → CH3 CH2 NH2
H2 O
Hoffmann bromamide degradation Reaction: An amide is heated with Bromine in aq. solution of
NaOH/KOH to gives primary amine.
R − CONH2 + Br2 + 4NaOH → R − NH2 + Na2 CO3 + 2NaBr + 2H2 O
The amine formed contains one carbon less than that present in the amide.
Gabriel Phthalimide Synthesis: Only aliphatic primary amines are prepared by this method.
Aromatic primary amines cannot be prepared because aryl halides do not undergo nucleophilic
substitution with the anion formed by phthalimide.

84 | Page
Physical properties Basic Nature of Amines

In Tertiary amines H-Bonding is not Amines are Lewis bases due to presence of lone pair.
possible due to absence of H atoms. Larger the value of Kb or smaller the value of pKb,
Hydrogen bonding 30<20<10 stronger is the base.
Solubility in water 30<20<10 NH3 < 10 < 20 < 30 (gas phase) (due to +I effect)
Boiling Point 30<20<10 NH3< C2H5NH2< (C2H5 )3N<(C2H5)2NH (aqueous phase)
Boiling point Alkane < Amine < Alcohol NH3 < (CH3 )3N <CH3NH2 < (CH3 )2NH (aqueous phase)

Ethylamine is more soluble than aniline in water due to the formation of more hydrogen bonding with
water whereas aniline due to large hydrocarbon part, the extent H–Bonding decreases.
Aliphatic amines are stronger base than ammonia due to +I effect of alkyl groups.
Aromatic amines are weaker bases than ammonia due to the electron withdrawing nature of the aryl
group.
Chemical reaction:
Acetylation:(replacement of H-atom of –NH2 />N–H (C2H5)2NH + CH3COCl  CH3CON(C2H5)2 + HCl
group by the acyl group).
Reacting agent: stronger base.
Benzoylation:(replacement of H-atom of –NH2 />N–H CH3NH2 + C6H5COCl C6H5 CONH-CH3 + HCl
group by the benzoyl group).
Carbylamine reaction (Isocyanide Test) RNH2 + CHCl3 + 3KOH
HEAT
RNC + 3KCl + 3H2 O
Primary amines react with chloroform and KOH give Secondary and tertiary amines do not show this
isocyanides (Carbylamines) which has unpleasent smell reaction. It is used as a test for primary amines.
(foul smell).
Reaction with nitrous acid (HNO2) RNH2 +HNO2
NaN O 2 +HCl
RN2+Cl−
H2O
ROH + N2 + HCl
Primary aliphatic amines react with nitrous acid to form
aliphatic diazonium salts which is unstable, and liberates
nitrogen gas to give alcohols. 278 K
Aromatic amines at low temperatures (273-278 K) forms C6 H5 NH2 + HNO2 C6 H5 N2+Cl− + NaCl + 2H2 𝑂
stable diazonium salts.
Reaction with Benzenesulphonyl chloride (Hinsberg's test) (C6H5SO2Cl)
Primary Amine Secondary Amine Tertiary Amine
Pramiry amines form Secondary amines form Tertiary amines do
N-alkylbenzenesulphonamide N, N-dialkylbenzenesulphonamide not react with
which is soluble in alkali due to which is not soluble in alkali due to Hinsberg reagent.
presence of acidic hydrogen at N- absence of hydrogen atom at N-
atom. atom.

Electrophilic substitution:
–NH2 group is ortho and para directing and a powerful activating group.
In direct Nitration meta product is formed due to formation of anilinium ion which is meta
directing.
NH 2 NH 2 NH 2 NH 2
No2 N
H N
H2
2 B
r

HONO2 / H 2 SO4
B
r
NO2 
3B
r2
B
r
2

/H
2

O
 3
HBr
NO2
51% 47% 2% B
r
 

N H2 N H 3 HSO4  NH 3 N H3
453 473K

H 2 SO4
  

Anilinum SO3 H SO3


hydrogen sulphate Sulphanilic acid Zwitter ion

85 | Page
Electrophilic substitution (Controlled and at desired position)
Aniline is converted into acetanilide by acetylation with acetic anhydride and then the desired
substitution is carried out followed by hydrolysis. In this case, the lone pair on nitrogen is less
available for donation to benzene ring by resonance. Therefore, activating effect of –NHCOCH3
group is less than that of amino group.
NH 2 NHCOCH 3 NHCOCH 3 NH 2
CH CO  O  

3 2
Pyridine

HNO3 , H 2 SO4
288 K
 
OH or H

Aniline acetanilide
NO2 NO2
P-Nitro P-Nitroaniline
acetanilide

Benzene Diazonium Chloride Preparation and Reactions:

MULTIPLE CHOICE QUESTIONS (1 MARK)


1 Aniline on treatment with excess of bromine water gives 1
(a) Aniline bromide (b)o-bromoaniline
(c) p-bromoaniline (d)2,4,6 tribromoaniline
2 Primary amines can be distinguished from secondary and tertiary amines by 1
reacting with
(a) Chloroform and alc. KOH (b)Methyl iodide
(c) Chloroform alone (d)Zinc dust
3 KCN react readily to give a cyanide with 1
(a) Ethyl alcohol (b) Ethyl bromide (c)Bromobenzene (d)Chlorobenzene
4 Nitration of aniline also gives m-nitro aniline, in strong acidic medium because- 1
(a)In electrophilic substitution reaction amino group is meta directive

86 | Page
(b)In spite of substituents nitro group always goes to m- position
(c)In strong acidic medium, nitration of aniline is a nucleophilic substitution
reaction
(d)In strong acidic medium aniline is present as anilinium ion
5 Amines are Lewis base due to 1
(a)Higher electronegativity (b)Unavailability of d subshell
(c)Presence of lone pair (d)Smaller Size
6 The correct order of increasing basicity in aqueous medium is – 1
NH3 CH3–CH2–NH2 (CH3–CH2)2–NH (CH3–CH2)3-N
I II III IV
(a) III< I < IV < II (b) II < I < III < IV (c) I < IV< II < III (d) I > II > IV >III
7 The conversion of benzene diazonium chloride to bromobenzene can be 1
accomplished by
(a) Reimer-Tiemann reaction (b) Friedel-Crafts reaction
(c) Gattermann reaction (d) Azo-Coupling reaction
8 Molecular formula of compound (X) is C7H7NO.When (X) is treated with Br2 and 1
NaOH; it gives an amine (Y), which gives positive carbylamine test. Compound Y is
(a) C6H5CONH2 (b) C6H5NHCH3 (c) C6H5 NH2 (d) o-C6H4(NH2)CHO
9 The correct IUPAC name for CH2 = CHCH2 NHCH3 is 1
(a) Allylmethylamine (b) 2-amino-4-pentene
(c) 4-aminopent-1-ene (d) N-methylprop-2-en-1-amine
10 Which one of the following is most basic? 1
(a) Benzylamine (b) p-Methoxyaniline (c) p-nitroaniline (d) Aniline
11 When n-propyl amine is heated with alc. KOH and chloroform an offensive 1
smelling compound (X) is formed. The compound (X) is
(a) CH3CH2CN (b) CH3CH2NC (c) CH3CH2CH2NC (d) CH3CH2CH2CN
12 The source of nitrogen in Gabriel synthesis of amines is ___________. 1
(a) Sodium azide, NaN3 (b) Sodium nitrite, NaNO2
(c) Potassium cyanide, KCN (d) Potassium phthalimide, C6H4(CO)2N-K+
13 The best reagent for converting 2-Phenylpropanamideinto 2-Phenylpropanamine 1
is .
(a) excess H2 (b) Br2 in aqueous NaOH (c) I2 / Red P (d) LiAlH4 in ether
14 Which of the following statements about primary amines is ‘false’? 1
(a) Alkylamines are stronger bases than arylamines.
(b) Alkylamines are stronger bases than ammonia.
(c) Alkylamines react with nitrous acid to produce alcohols.
(d) Arylamines react with nitrous acid to produce phenols
15 Amongst the given set of reactants, the most appropriate for preparing 2° amine is 1
____________.
(a) 2° R—Br + NH3
(b) 2° R—Br + NaCN followed by H2/Pt
(c) 1° R—NH2 + RCHO followed by H2/Pt
(d)1°R—Br (2 mol)+potassium phthalimide followed by H3O+/heat
Ans 1- d, 2- a, 3-b, 4-d, 5-c, 6-c, 7-c, 8-c, 9-d, 10-a, 11-c, 12-d, 13-d, 14-d, 15-c

REASON ASSERSION TYPES QUESTIONS


Given below are two statements labelled as Assertion (A) and Reason (R). Select the most appropriate
answer from the options given below:
(a) Both A and R are true and R is the correct explanation of A
(b) Both A and R are true but R is not the correct explanation of A.
(c) A is true but R is false.
(d) A is false but R is true.
1. Assertion: The major product of reaction Ph–C≡C-N-(CH3)2 with H⊕/H2O will contain amide as
only functional group.
87 | Page
Reason: The major product obtained will be formed due to formation of benzylic carbocation which
to stabilised due to +M–effect of benzene ring.
2 .Assertion: Solubility of amines in water increases with increase in molar mass.
Reason: Intermolecular hydrogen bonding becomes weaker in higher amines due to increase in
hydrophobic effect of carbon.
3 Assertion: Ammonolysis of alkyl halides is not a suitable method for the preparation of pure
primary amines.
Reason: Ammonolysis of alkyl halides yields mainly secondary amines.
4. Assertion: Carbylamine reaction involves chemical reaction between 1° amine and chloroform in
basic medium.
Reason: In carbylamine reaction, –NH2 group changes into –NC group.
5. Assertion: In strongly acidic solutions, aniline becomes more reactive towards electrophilic
reagent.
Reason: The amino group being completely protonated in strongly acidic solution, the lone pair
of electrons on the nitrogen is no longer available for resonance.
Ans 1- c 2- a 3-c 4-b 5-d
SHORT ANSWER TYPE QUESTIONS (2 MARKS)
Q 1 How would you account for the following:
(a) Electrophilic susbstitution in case of aromatic amines takes place more readily than benzene.
(b) Ethanamide is a weaker base than ethanamine.
Answer: (a) Aniline exists as a resonance hybrid of the following five structures:

The electron density is maximum at ortho and para positions to the – NH2 group. But in benzene there
is no delocalisation of electron at any position and hence electrophilic substitution in case of aromatic
amines takes place more readily than benzene.
(b) In ethanamide, the lone pair of e- of N-atom is not available due to resonance structure. So, it is a
weaker base.

Q2: Name the IUPAC names of following.


(a) The amide which gives propanamine by Hoffmann bromamide reaction.
(b) The amine produced by the Hoffmann degradation of benzamide.
Answer: (a) Butanamide (b) Aniline.
Q3: Identify ‘A’ and ‘B’ In the following reaction:

Answer:

Q4: Explain the following reactions:


(a) Gabriel Phthalimide reaction (b) Sandmeyer’s reaction
Answer: (a) Gabriel Phthalimide reaction

88 | Page
(b) Sandmeyer’s reaction

Q5: Write the chemical equations involved when aniline is treated with the following reagents: (a)
Br2 water (b) CHCl3 + KOH
Answer: (a) (b)

Q6: How will you convert the following:


(a) Ethanoic acid to methanamine (b) Benzene diazonium chloride to phenol
Answer: (a) Ethanoic acid to methanamine
𝑁𝐻3 𝐵𝑟2 /𝐾𝑂𝐻
𝐶𝐻3 𝐶𝑂𝑂𝐻 𝐶𝐻3 𝐶𝑂𝑁𝐻2 𝐶𝐻3 𝑁𝐻2
∆ 𝐻𝑜𝑓𝑓𝑚𝑎𝑛𝑛 𝑏𝑟𝑜𝑚𝑖𝑑𝑒 𝑑𝑒𝑔𝑟𝑎𝑑𝑎𝑡𝑖𝑜𝑛
(b)Benzene diazonium chloride to phenol
𝐶6 𝐻5 𝑁2+𝐶𝑙 − + 𝐻2 𝑂 → 𝐶6 𝐻5 𝑂𝐻 + 𝑁2 + 𝐻𝐶𝑙
Q7: Give the chemical tests to distinguish between the following pairs of compounds:
(a)Ethylamine and Aniline (b) Aniline and Benzylamine
Answer: (a) Ethylamine and aniline: Azo-dye test, Aromatic amines (aniline) give the test while
aliphatic amines (ethylamine) do not give the test.
(b) Aniline and benzyl amine: By Nitrous acid test, Benzylamine do not give this test.
Q8: Complete the following chemical reactions:

Answer:

Q9: Give the structures of A and B

Answer:

Q10: Arrange the following


(a) decreasing order of solubility in water: (C2H5)2NH, C2H5NH2, C6H5NH2
(b) increasing order of their boiling point:C2H5NH2, C2H5OH, (CH3)3N
Answer: (a) C2H5NH2> (C2H5)2NH > C6H5NH2
(b) (CH3)3N < C2H5NH2 < C2H2OH
SHORT ANSWER TYPE QUESTIONS (3 MARKS)
1. How will you convert:
(a) Aniline into Fluorobenzene.
(b) Benzamide into Benzylamine.
(c) Ethanamine into N, N-Diethylethanamine.
89 | Page
Answer:

Q2. Mr. Krishna, a chemistry teacher gave his class students a sample of an amine “P” having
molecular formula C4H11N to test, identify and write the reaction. The students observed that it reacts
with Hinsberg’s reagent to form an alkali soluble product. A student Ranu wrote the reaction as
follows.

Identify the mistakes committed by Ranu. Write the type of amine in sample “P” and correct chemical
equation.
Answer: The formula of Hinsberg’s reagent and sample of P were written by Ranu are incorrect. The
Sample “P” belongs to primary amine. Therefore, sample “P” is Butanamine. The correct chemical
equation is as follows:

Q3. Account for the following:


(a) Gabriel phthalimide synthesis is preferred for synthesizing primary amines.
(b) Diazonium salts of aromatic amines are more stable than those of aliphatic amines
(c) Acylation of aniline is carried out in the presence of pyridine.
Answer:
(a) Gabriel phthalimide synthesis results in the formation of primary amines only. Secondary and
tertiary amines are not formed in this synthesis. Thus, a pure primary amine can be obtained.
(b) The diazonium salts of aromatic amines are more stable than those of aliphatic amines due to
dispersal of the positive charge on the benzene ring.
(c) During the acylation of aniline, stronger base pyridine is added. This done in order to remove the
HCl so formed during the reaction and to shift the equilibrium to the right hand side.
Q4. Arrange the following:
(a) In decreasing order of the pKb values:
C2H5NH2, C6H5NHCH3, (C2H5)2NH and C6H5NH2
(b) In decreasing order of basic strength: C6H5NH2, C6H5N (CH3)2, (C2H5)2NH and CH3NH2
(c) In increasing order of basic strength: Aniline, p-nitroaniline and p-toluidine
Ans. (a) Stronger the base lesser the pKb values. C6H5NH2> C6H5NHCH3> C2H5NH2> (C2H5)2NH
(b) (C2H5)2NH > CH3NH2 > C6H5N (CH3)2> C6H5NH2
(c) p-nitro aniline < Aniline < p-toluidine
Q5. An aromatic compound ‘A’ on treatment with aqueous ammonia and heating forms compound ‘B’
which on heating with Br2 and KOH forms a compound ‘C’ of molecular formula C6H7N. Write the
structures and IUPAC names of compounds A, B and C.
Answer:

90 | Page
LONG ANSWER TYPE QUESTIONS (5 MARKS)
1. An aromatic compound A of molecular formula C7H7ON undergoes a series of reactions as shown
below. Write the structures of A, B, C, D and E in the following reactions :

Answer:

2. Give reasons for the following (Attempt any five):


(a) Aniline does not undergo Friedal-Crafts reaction.
(b) (CH3)2NH is more basic than (CH3)3N in an aqueous solution.
(c) Primary amines have higher boiling point than tertiary amines.
(d) CH3NH2 is more basic than C6H5NH2.
(e) Although —NH2 is o-p directing group, yet aniline on nitration gives a significant amount of m-
nitroaniline.
(f) pKb of aniline is more than that of methylamine
(g) Ethylamine is soluble in water whereas aniline is not.
Answer: (a) Aniline being a Lewis base reacts with Lewis acid AlCl3 to form a salt.
As a result, N of aniline acquires positive charge and hence it acts as a strong deactivating group for
electrophilic substitution reaction. Consequently, aniline does not undergo Freidel Crafts reaction.
(b) In (CH3)3N there is maximum steric hindrance and least solvation but in (CH3)2NH the solvation is
more and the steric hindrance is less than in (CH3)3NH; although + I effect is less, since there are two
methyl groups; di-methyl amine is still a stronger base than tri-methyl.
(c) Due to presence of two H-atoms on N-atom of primary amines, they undergo extensive
intermolecular H-bonding while tertiary amines due to the absence of a H-atom on the N-atom, do not
undergo H- bonding. As a result, primary amines have higher boiling points than 3° amines.
(d)CH3NH2 is more basic than aniline due to availability of lone pair of electrons for donation while in
aniline lone pair of electrons on the nitrogen atom is delocalised over benzene ring and thus
unavailable for donation.
(e) Because of nitration in an acidic medium, aniline gets protonated to give anilinium ion which is
indirecting.
(f) In aniline, the lone pair of electrons on the N-atom is delocalised over the benzene ring.
As a result, electron density on the nitrogen atom decreases. Whereas in CH3NH2,+ I-effect of -CH3
group increases the electron density on the N-atom. Therefore, aniline is a weaker base than
methylamine and hence its pKb value is higher than that of methylamine.
(g) Ethylamine dissolves in water due to intermolecular H-bonding. However, in case of aniline, due to
the large hydrophobic part, i.e., hydrocarbon part, the extent of H-bonding is very less therefore
aniline is insoluble in water.
CASE BASED QUESTIONS
Read the passage given below and answer the following questions:-
Amines are derivatives of ammonia in which one or more of the hydrogens has been replaced by an
alkyl or aryl group. For the naming of Amines in the IUPAC system: the “e” ending of the alkane name
for the longest chain is replaced with –amine. The amine group is located by the position number.
91 | Page
Groups that are attached to the nitrogen atom are located using “N” as the position number. More
complex primary amines are named with —NH2 as the amino substituent. Aromatic amines: named as
derivatives of the parent compound aniline. Substituents attached to the nitrogen are indicated by
using “N-” as the location number.
(a) Write the IUPAC name of H2N-(CH2)5-NH2?
(b) Complete the following reaction equations:
(i) C6H5N2CI + H3PO2 + H20 →
(c)Classify the following amines as primary, secondary and tertiary:

OR
(c) Out of CH3—NH2 and (CH3)3N, which one has higher boiling point? Why?
(a)Pentan-1,5-diamine
(b)

(c) (i) 1° (ii) 3° (iii) 1° (iv) 2°


OR
(c) CH3—NH2 has higher boiling point than (CH3)3N because of more H-bonding in primary amines.

BIOMOLECULES
QUICK REVISION POINTS:
KEY POINTS EXPLANATIONS
Monosaccharides Cannot be hydrolysed further. eg- glucose, fructose, ribose
Disaccharides Sucrose (C1 of α-D- glucose + C2 of β-D-fructose), Maltose (C1 of α-D-
glucose + C4 of α-D- glucose), Lactose (C1 of β-D-galactose + C4 of β-D-
glucose)
Polysaccharides Starch (two components—Amylose and Amylopectin) polymer of
α-D- glucose
Amylose Water soluble, 15-20% of starch, unbranched chain, C1– C4 glycosidic
linkage.
Amylopectin Water insoluble, 80-85% of starch, branched chain polymer,
C1–C4 & C1–C6 glycosidic linkage
Cellulose Straight chain polysaccharide of β-D-glucose units/ joined by C1-C4
glycosidic linkage (β-link), not digestible by human / constituent of cell
wall of plant cells
Glycogen Highly branched polymer of α-D- glucose. found in the liver, muscles
and brain.
reducing sugars Aldehydic/ ketonic groups are free so reduce Fehling’s/ Tollens
solution and. eg- maltose and lactose
Non reducing Aldehydic/ ketonic groups are bonded so cannot reduce Fehling’s
sugars solution and Tollens’ reagent. eg- Sucrose
Anomers. The two cyclic hemiacetal forms of glucose differ only in the
configuration of the hydroxyl group at C1, called anomeric carbon Such
isomers, i.e., α –form and β -form, are called anomers.

92 | Page
Invert sugar Sucrose is dextrorotatory but after hydrolysis gives dextrorotatory
glucose and laevorotatory fructose. Since the laevorotation of fructose
(–92.4°) is more than dextrorotation of glucose (+ 52.5°), the mixture is
laevorotatory. Thus, hydrolysis of sucrose brings about a change in the
sign of rotation, from dextro (+) to laevo (–) and the product is named
as invert sugar
Glycosidic linkage Linkage between two monosaccharides
Importance of Major portion of our food. / used as storage molecules as starch in
Carbohydrates plants and glycogen in animals/. Cell wall of bacteria and plants is
made up of cellulose. /wood and cloth are cellulose / provide raw
materials for many important industries like textiles, paper, lacquers
and breweries.
essential amino which cannot be synthesised in the body and must be obtained through
acids diet, eg- Valine, Leucine (TV MILL PATH- The ten essential Amino
Acids)
Nonessential which can be synthesised in the body, eg - Glycine, Alanine
amino acids
Zwitter ion. In aqueous solution, amino acids exist as a dipolar ion known as Zwitter
ion. [+H3N-CH2-COO-]
peptide linkage peptide linkage is an amide formed between –COOH group and –NH2
group of two successive amino acids in peptide chain.
1 - str. of proteins: sequence of amino acids that is said to be the primary structure of
0

protein
20- str. of proteins: The secondary structure of protein refers to the shape in which a long
polypeptide chain can exist.
They are found to exist in two types of structures viz. α -helix and β -
pleated sheet structure.
Tertiary structure Further folding of the secondary structure. It gives rise to two major
of proteins: molecular shapes viz. fibrous and globular.
Fibrous proteins Polypeptide chains run parallel, held together by hydrogen and
disulphide bonds, fibre– like structure. Water insoluble. eg- are keratin
(in hair, wool, silk) and myosin (present in muscles).
Globular proteins Chains of polypeptides coil around to give a spherical shape. water
soluble. Eg-Insulin and albumins
Stab.forces 2°& 3° Hydrogen bonds, disulphide linkages, van der Waals and electrostatic
forces of attraction.
Denaturation of When a protein is subjected to physical change like change in
Proteins temperature or chemical change like change in pH, the hydrogen bonds
are disturbed. Due to this, globules unfold and the helix gets uncoiled
and the protein loses its biological activity. This is called denaturation
of protein. (During denaturation 2° and 3° structures are destroyed but
1º structure remains intact.)
eg- The coagulation of egg white on boiling, curdling of milk
Enzymes Enzymes are essential biological catalysts which are required to
catalyse biological reactions, e.g., maltase, lactase, invertase, etc. Almost
all the enzymes are globular proteins
Fat soluble vit These are vitamins K, E, D, A (कीड़ा). They are stored in liver and
adipose (fat storing) tissues
Water soluble vit B, C. these vitamins must be supplied regularly in diet because they are
readily excreted in urine
Vitamins –sources- Vit- A (Fish liver oil, carrots)- Night blindness
Deficiency diseases Vitamin B1 (Yeast, milk,)- Beri beri
93 | Page
Vit-B2 (Milk, egg white)- Cheilosis
Vit- B6 (Yeast, milk,)- Convulsions
Vit- B12 (Meat, fish,)- anaemia
Vit C (Citrus fruits)- Scurvy,
Vit D (Exposure to sunlight, fish and egg yolk)- Rickets, osteomalacia
Vit E (wheat oil, sunflower oil)- fragility of RBCs
Vit K (leafy vegetables)- Increased blood clotting time

DNA Pentose sugar (D-2-deoxyribose) + phosphoric acid (at C3 or C5 of


pentose sugar) + nitrogenous bases (at C1 of pentose sugar) (A, G, C,
T). Hydrogen bond (H-bond) binds the nitrogen bases between the
two strands of DNA. There are two H-bonds between A and T; three H-
bonds between G and C.
RNA pentose sugar (ribose) + phosphoric acid + nitrogenous bases (A, G,
C, U )
Nucleoside / tides Nucleoside = sugar + base
Nucleotides = sugar + base + phosphate
Phosphodiester Linkage between two nucleotides in polynucleotides
link
Functions of DNA reserves genetic information, maintains the identity of different
Nucleic Acids species, is capable of self-duplication during cell division, and
synthesises protein in the cell.

Multiple Choice Questions


Q1 Which of the following vitamin B can be stored in our body?
(a) Vitamin B1. (b) Vitamin B2 (c) Vitamin B6 (d) Vitamin B12
Q2 Which of the following is essential amino acid?
(a) Valine (b) Glycine. (c) Proline (d) Tyrosine
Q3 Deficiency of Vitamin B6, cause the disease
(a) Convulsion (b) Beri-Beri (c) Cheilosis (d)sterility.
Q4 The two components of DNA are 2-deoxyribose and a nitrogenous base, the third
component is?
(a) D-ribose (b) Thymine (c) Guanine (d) Phosphoric acid
Q5 Which of the following is not true about amino acids?
(a) They are constituent of all proteins (b) Alanine has one -NH2 and one -COOH group
(c) All amino acids have D-configuration (d) Glycine is only optically inactive
Q6 Nucleic acids are the polymer of ____________
(a) Nucleosides (b) Nucleotides (c) Bases (d) Sugar
Q7 The linkage repeated in a protein molecule is-
(a) An ester linkage (b) A glycosidic linkage (c) A peptide linkage (d) An ether linkage
Q8 Amino acids which must be obtained through diet are known as-
(a) Acidic amino acid (b) Essential amino acids
(c) Basic amino acid (d) Non-essential amino acid
Q9 Which of the following polymers is stored in the liver of animals?
(a) Amylose (b) Cellulose (c) Amylopectin (d) Glycogen
Q10 Which of the following is fibrous protein?
(a) Albumin (b) Keratin (c) Insulin (d) Globulin
Q11 Lactose is formed by
(a) C-1 of galactose and C-4 of glucose (b) C-4 of galactose and C-4 of glucose
(c) C-1 of galactose and C-6 of glucose (d) C-6 of galactose and C-1 of glucose
Q12 Glucose on acetylation form Penta acetate which conform the presence of –
(a) -CHO group (b) -COOH group (c) Five -OH group (d) A straight chain
Q13 Enantiomers differ only in
94 | Page
(a) Boiling point (b) Rotation of polarised light (c) Melting point (d) Solubility
Q14 Which of the following is a disaccharide?
(a) Glucose (b) Starch (c) Cellulose (d) Lactose
Q15 Which of the following sugar is known as dextrose?
(a) Glucose (b) Fructose (c) Ribose (d) Sucrose

Q.No. 1 2 3 4 5 6 7 8 9 10 11 12 13 14 15
Ans. d a a d c b c b d b a c b d a

Assertion and reason type Questions


Given below are two statements labelled as Assertion (A) and Reason (R). Select the most appropriate
answer from the options given below for questions 1-5:
(a) Both A and R are true and R is the correct explanation of A
(b) Both A and R are true but R is not the correct explanation of A.
(c) A is true but R is false.
(d) A is false but R is true.
Q1 Assertion (A): D (+)-Glucose is dextrorotatory in nature.
Reason(R): ‘D’ represents its dextrorotatory nature.
Q2 Assertion (A): Sucrose is a non-reducing sugar.
Reason(R): in sucrose, the aldehydic group of glucose and ketonic group of fructose are not free.
Q3 Assertion (A): Vitamin D can be stored in our body.
Reason(R): Vitamin D is fat soluble vitamin
Q4 Assertion (A): Globular proteins are soluble in water.
Reason(R): Keratin is a fibrous protein.
Q5 Assertion (A): Glycine must be taken through diet.
Reason(R): It is a non-essential amino acid

Q.No. 1 2 3 4 5
Ans. c a a b d

SHORT ANSWER TYPE QUESTIONS (2 MARKS)


Q1. What are the products of hydrolysis of maltose?
Ans. The product of hydrolysis of maltose are two molecules of α−D Glucose.
Q2. What is meant by the denaturation of a protein?
Ans. Due to coagulation of globular protein under the influence of change in temperature, change in
pH etc., the native shape of the protein is destroyed and biological activity is lost, this is called
denaturation of protein.
Q3. What do you understand by the term peptide linkage?
Ans. Peptide linkage is an amide formed between the -COOH group and the -NH2 group of two
successive amino acids.
Q4. What happens when D-glucose is treated with the following reagents? (any two)
(i) Hydroxylamine (ii) Bromine water (iii) Acetic anhydride
Ans. (i) oxime (ii)Gluconic acid (iii) Glucose pentaacetate
Q5. Name the only vitamin which can be synthesised in our body. Name the disease caused due to the
deficiency of this vitamin.
Ans. Vitamin which can be synthesised in our body: Vitamin A, its deficiency causes Xerophthalmia.
Q6. Enumerate any one reaction of D-glucose which cannot be explained by its open chain structure.
Ans. The pentaacetate of glucose does not react with hydroxylamine indicating absence of —CHO
group.
Q7. Write one difference between α-helix and β-pleated sheet structures of protein.
Ans. In the α-helix structure of proteins, the polypeptide chains are stabilised by intramolecular
hydrogen bonding whereas β-pealed sheet structure of proteins is stabilised by intermolecular
hydrogen bonding.

95 | Page
Q8. What is meant by ‘reducing sugars’?
Ans. Reducing sugar contains aldehydic or ketonic groups in the hemiacetal and hemiketal forms and
can reduce Tollen’s reagent or Fehling’s solution.
Q9. What is the difference between a nucleoside and a nucleotide?
Ans. A nucleoside can be represented as: Sugar–Base.
A nucleotide can be represented as: Sugar–Base - phosphoric acid group.
Q10. Write the name of nitrogenous bases found in DNA.
Ans. Purine Bases: Adenine (A), Guanine(G) Pyrimidine Bases: Cytosine (C), Thymine (T)

SHORT ANSWER TYPE QUESTIONS (3 MARKS)


Q1. (i) Which of the following biomolecules is insoluble in water? Justify. Insulin, Haemoglobin,
Keratin.
(ii) Why cannot vitamin C be stored in our body?
(iii) Write a chemical reaction to show that glucose contains aldehyde as a carbonyl group.
ANS: (i) Keratin being a fibrous protein insoluble in water
(ii) As vitamin C is water soluble, therefore, it is readily excreted in urine and hence cannot be stored
in the body.
(iii) Glucose reduces Tollens’ reagent to metallic silver.
Q2. Write the main structural difference between DNA and RNA.
Ans.

DNA RNA
1. The sugar present in DNA is 2-deoxy-(-) 1. The sugar present in RNA is D-(-) ribose.
ribose.
2. DNA contains cytosine and thymine as 2. RNA contains cytosine and uracil as pyrimidine
pyrimidine bases. bases.
3. DNA has double standard α-helix 3. RNA has single stranded α-helix structure.
structure.
Q3. Write down the structures and names of the products formed when D-glucose is treated with
(i) Bromine water (ii) Hydrogen Iodide (Prolonged heating). (iii) HNO3
Ans.

Q4. (i) Give one example for fibrous protein and globular protein
(ii). What deficiency disease is caused due to lack of vitamin B1?
(iii) Define Phosphodiester linkage.
Ans. (i) Fibrous protein: Myosin, keratin, collagen, etc. Globular protein: Insulin, haemoglobin, etc
(ii) Beri-Beri
(iii) Linkage between two polynucleotide units.

Q5. Define the following terms:


(i) Nucleotide (ii) Anomers (iii) Essential amino acids
Ans. (i) Nucleotide: A nucleotide contains phosphoric acid, pentose sugar and nitrogenous base.
Structure of Nucleotide:

96 | Page
(ii) Anomers: A pair of stereoisomers which differ in configuration only at C1 are called anomers. eg-
α−D Glucose and β-D- glucose.
(iii) Essential amino acids: Amino acids which are not synthesised in the human body and must be
taken in a diet called essential amino acids. Example: Valine, leucine etc.

LONG ANSWER TYPE QUESTIONS (5 MARKS)


Q.1 (a) What is the difference between native protein and denatured protein?
(b) What type of linkage is present in carbohydrates?
(c) Write the name of the vitamin responsible for the coagulation of blood.
(d)Write the structural difference between starch and cellulose.
(e) Differentiate amylose and amylopectin
Ans. (a) Protein found in a biological system with unique three-dimensional structure and biological
activity is called native protein. When a protein in its native form is subjected to physical and chemical
change, its 2° and 3° structures are destroyed and it loses its biological activity. The protein thus
formed is called denatured protein.
(b) glycosidic linkages
(c) Vitamin K.
(d) Starch contains the α−D Glucose while cellulose contains β-D- glucose as monomer units.
(e) Amylose is a linear polymer, whereas amylopectin is a branched polymer.

Q.2. (a) Give one example each for fibrous protein and globular protein.
(b) Amino acids may be acidic, alkaline or neutral. How does this happen?
(c) Name one oil soluble vitamin which is a powerful antioxidant and give its one natural source.
Ans. (a) Globular protein: All enzymes and hormones like insulin.
Fibrous protein: Keratin in skin.
(b) Amino acids can be broadly classified into the classes depending on the number of —NH2 group
and — COOH group.
Acidic amino acids: Those amino acids such as aspartic acid, asparagine and glutamic acid which
contain two -COOH groups and one -NH2 group are called acidic amino acids.
Alkaline or Basic amino acids: Those amino acids such as lysine, arginine and histidine which
contain two -NH2 groups and one -COOH group, are called basic amino acids.
Neutral amino acids: Those amino acids such as glycine, alanine, valine etc. which contain one -NH2
and one – COOH group, are called neutral amino acids.
(c) Oil soluble Vitamin: Vitamin D
Natural source: Fish liver oil, butter, milk, eggs etc.

CASE BASED QUESTIONS (4 MARKS)


Glucose has an aldehyde group (-CHO), one primary alcoholic group (-CH2OH) and four secondary
alcoholic groups (-CHOH) in their structure. Due to the presence of five hydroxyl groups (-OH),
glucose undergoes acetylation, oxidation with mild oxidising agents like bromine water as well as
with strong oxidising agents like nitric acid. Since glucose is readily oxidised, it acts as a strong
reducing agent and reduces Tollen's reagent and Fehling solution. The two crystalline forms of
glucose: α-D-glucose and β-D glucose undergo mutarotation in aqueous solution.
(a) Which reaction proves the presence of five hydroxyl groups?

97 | Page
(b) On oxidation with nitric acid, glucose as well as gluconic acid both yield the same product, name
the product.
(c) Which sugar is known as invert sugar and why?
Or
(c) Which disaccharide is known as non-reducing sugar and why?
Ans: (a) Acetylation of glucose with acetic anhydride gives glucose pentaacetate which confirms the
presence of five –𝑂𝐻 groups as given in the reaction.
(b) Saccharic acid
(c) Sucrose is dextrorotatory but on hydrolysis gives dextrorotatory glucose and laevorotatory
fructose. Since the laevorotation of fructose (−92.4o) is more than dextrorotation of glucose (+52.5o),
the mixture is laevorotatory. Thus, the product is named as inverted sugar.
Or
(d) Sucrose is considered as Non-Reducing sugar. Two monosaccharide units are held together by a
glycosidic linkage between C1 of α-glucose and C2 of β-fructose. Since the reducing groups of
glucose and fructose are involved in glycosidic bond formation, sucrose is a non-reducing sugar.

98 | Page

You might also like